Med Surg II Exam 2 Practice Questions

अब Quizwiz के साथ अपने होमवर्क और परीक्षाओं को एस करें!

A patient with MS has been admitted to the hospital following an acute exacerbation. When planning the patient's care, the nurse addresses the need to enhance the patient's bladder control. What aspect of nursing care is most likely to meet this goal? A) Establish a timed voiding schedule. B) Avoid foods that change the pH of urine. C) Perform intermittent catheterization q6h. D) Administer anticholinergic drugs as ordered.

A) Establish a timed voiding schedule.

The nurse is developing a plan of care for a patient with Guillain-Barré syndrome. Which of the following interventions should the nurse prioritize for this patient? A) Using the incentive spirometer as prescribed B) Maintaining the patient on bed rest C) Providing aids to compensate for loss of vision D) Assessing frequently for loss of cognitive function

A) Using the incentive spirometer as prescribed

The nurse is providing care for a patient who has benefited from a cochlear implant. The nurse should understand that this patient's health history likely includes which of the following? Select all that apply. A) The patient was diagnosed with sensorineural hearing loss. B) The patient's hearing did not improve appreciably with the use of hearing aids. C) The patient has deficits in peripheral nervous function. D) The patient's hearing deficit is likely accompanied by a cognitive deficit. E) The patient is unable to lip-read.

A,B

The nurse is caring for a client who has undergone supratentorial removal of a pituitary mass. What medication would the nurse expect to administer prophylactically to prevent seizures in this client?

Phenytoin

Hepatitis type Source: blood/bodily fluids Route of transmission: percutaneous permucosal (needles/sex) chronic Prevent by pre/post-exposure immunization and risk behavior modification Only people with Hep B at risk for it

Hepatitis D

A patient with a new diagnosis of ischemic stroke is deemed to be a candidate for treatment with tissue plasminogen activator (t-PA) and has been admitted to the ICU. In addition to closely monitoring the patients cardiac and neurologic status, the nurse monitors the patient for signs of what complication? A) Acute pain B) Septicemia C) Bleeding D) Seizures

C

A patient with suspected Creutzfeldt-Jakob disease (CJD) is being admitted to the unit. The nurse would expect what diagnostic test to be ordered for this patient? A) Cerebral angiography B) ABG analysis C) CT D) EEG

D) EEG

The nurse is caring for a client who has developed SIADH. What intervention is most appropriate?

Fluid restriction

Hepatitis type Source: feces Route of transmission: fecal-oral Not chronic Prevent by pre/post-exposure immunization

Hepatitis A

A patient diagnosed with Bell's palsy is being cared for on an outpatient basis. During health education, the nurse should promote which of the following actions? A) Applying a protective eye shield at night B) Chewing on the affected side to prevent unilateral neglect C) Avoiding the use of analgesics whenever possible D) Avoiding brushing the teeth

A) Applying a protective eye shield at night

Following a motorcycle accident, a 17-year-old man is brought to the ED. What physical assessment findings related to the ear should be reported by the nurse immediately? A. Clear, watery fluid is draining from the patient's ear. B. The malleus can be visualized during otoscopic examination. C. Tenderness is reported by the patient when the mastoid area is palpated. D. The tympanic membrane is pearly gray.

A. Clear, watery fluid is draining from the patient's ear.

When caring for a patient with a neurologic impairment and his or her family, what are the mutual goals? A) Achieve as high a level of function as possible. B) Enhance the quantity of life. C) Teach the family proper care of the patient. D) Provide community assistance.

ANS: A The goals are to achieve as high a level of function as possible and to enhance the quality of life for the patient with neurologic impairment and his or her family. It is not a goal to enhance the quantity of the patient's life or provide community assistance. The scenario does not indicate that the patient needs to be taken care of by the family.

When caring for a patient with increased ICP the nurse must monitor for possible secondary complications. One possible complication of increased ICP is SIADH. What nursing interventions would the nurse initiate if the patient developed SIADH? A) Fluid restriction B) Fluid replacement C) Electrolyte replacement D) Electrolyte restriction

ANS: A The nurse also assesses for complications of increased ICP, including diabetes insipidus and SIADH. SIADH requires fluid restriction and monitoring of serum electrolyte levels. You do not "restrict" electrolytes with patients, you monitor them. Diabetes insipidus requires fluid and electrolyte replacement.

When caring for a patient with a neurologic dysfunction, what complications must the nurse monitor for? (Mark all that apply.) A) Contractures B) Interrupted family processes C) Pressure ulcer D) DVT E) Pneumonia

ANS: A, C, D, E Based on the assessment data, potential complications may include respiratory distress or failure, pneumonia, aspiration, pressure ulcer, deep vein thrombosis (DVT), and contractures. Interrupted family processes is a nursing diagnosis, not a possible complication.

A school nurse is called to the playground where a 6-year-old girl has fallen off the slide. When the nurse gets to the playground the girl is exhibiting jerking motions in her left arm and leg. The girl is unconscious. How would the nurse document the girl's activity in her chart at school? A) Simple partial seizure B) Complex partial seizure C) Complex generalized seizure D) Simple generalized seizure

ANS: B In a simple partial seizure, consciousness remains intact, whereas in a complex partial seizure, consciousness is impaired.

A patient is being admitted to the Neuro ICU following an acute head injury. The patient has cerebral edema. The nurse would expect to administer what priority medications to reduce cerebral edema? A) Hydrochlorothiazide (HydroDIURIL) B) Lasix (Furosemide) C) Mannitol (Osmitrol) D) Spirolactone (Aldactone)

ANS: C The osmotic diuretic mannitol is given to dehydrate the brain tissue and reduce cerebral edema. This drug acts by reduces the volume of brain and extracellular fluid. Spirolactone, Lasix, and Hydrochorothiazide are used in the treatment of CHF and hypertension.

The nurse is caring for a patient with increased intracranial pressure (ICP) caused by a traumatic brain injury. Which of the following clinical manifestations would suggest that the patient may be experiencing increased brain compression causing brain stem damage? A) Hyperthermia B) Tachycardia C) Hypertension D) Bradypnea

Ans: A Feedback: Signs of increasing ICP include slowing of the heart rate (bradycardia), increasing systolic BP, and widening pulse pressure. As brain compression increases, respirations become rapid, BP may decrease, and the pulse slows further. A rapid rise in body temperature is regarded as unfavorable. Hyperthermia increases the metabolic demands of the brain and may indicate brain stem damage.

The school nurse is giving a presentation on preventing spinal cord injuries (SCI). What should the nurse identify as prominent risk factors for SCI? Select all that apply. A) Young age B) Frequent travel C) African American race D) Male gender E) Alcohol or drug use

Ans: A, D, E Feedback: The predominant risk factors for SCI include young age, male gender, and alcohol and drug use. Ethnicity and travel are not risk factors.

A preceptor is discussing stroke with a new nurse on the unit. The preceptor would tell the new nurse which cardiac dysrhythmia is associated with cardiogenic embolic strokes? A) Ventricular tachycardia B) Atrial fibrillation C) Supraventricular tachycardia D) Bundle branch block

Ans: B Feedback: Cardiogenic embolic strokes are associated with cardiac dysrhythmias, usually atrial fibrillation. The other listed dysrhythmias are less commonly associated with this type of stroke.

The ED nurse is caring for a patient who has been brought in by ambulance after sustaining a fall at home. What physical assessment finding is suggestive of a basilar skull fracture? A) Epistaxis B) Periorbital edema C) Bruising over the mastoid D) Unilateral facial numbness

Ans: C Feedback: An area of ecchymosis (bruising) may be seen over the mastoid (Battle's sign) in a basilar skull fracture. Numbness, edema, and epistaxis are not directly associated with a basilar skull fracture.

A 16-year-old presents at the emergency department complaining of right lower quadrant pain and is subsequently diagnosed with appendicitis. When planning this patients nursing care, the nurse should prioritize what nursing diagnosis? A) Imbalanced Nutrition: Less Than Body Requirements Related to Decreased Oral Intake B) Risk for Infection Related to Possible Rupture of Appendix C) Constipation Related to Decreased Bowel Motility and Decreased Fluid Intake D) Chronic Pain Related to Appendicitis

Ans: B Feedback: The patient with a diagnosis of appendicitis has an acute risk of infection related to the possibility of rupture. This immediate physiologic risk is a priority over nutrition and constipation, though each of these concerns should be addressed by the nurse. The pain associated with appendicitis is acute, not chronic.

22. A patient with liver disease has developed jaundice; the nurse is collaborating with the patient to develop a nutritional plan. The nurse should prioritize which of the following in the patient's plan? A) Increased potassium intake B) Fluid restriction to 2 L per day C) Reduction in sodium intake D) High-protein, low-fat diet

Ans: C Feedback: Patients with ascites require a sharp reduction in sodium intake. Potassium intake should not be correspondingly increased. There is no need for fluid restriction or increased protein intake.

The nurse is caring for a patient recovering from an ischemic stroke. What intervention best addresses a potential complication after an ischemic stroke? A) Providing frequent small meals rather than three larger meals B) Teaching the patient to perform deep breathing and coughing exercises C) Keeping a urinary catheter in situ for the full duration of recovery D) Limiting intake of insoluble fiber

B

To alleviate pain associated with trigeminal neuralgia, a patient is taking Tegretol (carbamazepine). What health education should the nurse provide to the patient before initiating this treatment? A) Concurrent use of calcium supplements is contraindicated. B) Blood levels of the drug must be monitored. C) The drug is likely to cause hyperactivity and agitation. D) Tegretol can cause tinnitus during the first few days of treatment.

B) Blood levels of the drug must be monitored.

A patient with MS has developed dysphagia as a result of cranial nerve dysfunction. What nursing action should the nurse consequently perform? A) Arrange for the patient to receive a low residue diet. B) Position the patient upright during feeding. C) Suction the patient following each meal. D) Withhold liquids until the patient has finished eating.

B) Position the patient upright during feeding.

The nurse is caring for a patient with multiple sclerosis (MS). The patient tells the nurse the hardest thing to deal with is the fatigue. When teaching the patient how to reduce fatigue, what action should the nurse suggest? A) Taking a hot bath at least once daily B) Resting in an air-conditioned room whenever possible C) Increasing the dose of muscle relaxants D) Avoiding naps during the day

B) Resting in an air-conditioned room whenever possible

A male patient with multiple injuries is brought to the ED by ambulance. He has had his airway stabilized and is breathing on his own. The ED nurse does not see any active bleeding, but should suspect internal hemorrhage based on what finding? A. Sudden diaphoresis B. Rapid pulse and decreased capillary refill C. Absence of bruising at contusion sites D. Increased BP with narrowed pulse pressure

B. Rapid pulse and decreased capillary refill

A patient is admitted through the ED with suspected St. Louis encephalitis. The unique clinical feature of St. Louis encephalitis will make what nursing action a priority? A) Serial assessments of hemoglobin levels B) Blood glucose monitoring C) Close monitoring of fluid balance D) Assessment of pain along dermatomes

C) Close monitoring of fluid balance

A patient is being admitted to the neurologic ICU with suspected herpes simplex virus encephalitis. What nursing action best addresses the patient's complaints of headache? A) Initiating a patient-controlled analgesia (PCA) of morphine sulfate B) Administering hydromorphone (Dilaudid) IV as needed C) Dimming the lights and reducing stimulation D) Distracting the patient with activity

C) Dimming the lights and reducing stimulation

The nurse is administering eye drops to a patient with glaucoma. After instilling the patient's first medication, how long should the nurse wait before instilling the patient's second medication into the same eye? A) 30 seconds B) 1 minute C) 3 minutes D) 5 minutes

D

What should be included in the patients care plan when establishing an exercise program for a patient affected by a stroke? A) Schedule passive range of motion every other day. B) Keep activity limited, as the patient may be over stimulated. C) Have the patient perform active range-of-motion (ROM) exercises once a day. D) Exercise the affected extremities passively four or five times a day.

D

A nurse is giving an educational class to members of the local disaster team. What should the nurse instruct members of the disaster team to do in a chemical bioterrorist attack? A. Crawl to an exit. B. Cover their eyes. C. Put on a personal protective equipment mask. D. Stand up.

D. Stand up.

A client has experienced a seizure in which she became rigid and then experienced alternating muscle relaxation and contraction. What type of seizure does the nurse recognize?

Generalized seizure

Hepatitis type Source: blood/bodily fluids Route of transmission: percutaneous permucosal (needles/sex) chronic Prevent by blood donor screening and risk behavior modification Most common blood-borne infection

Hepatitis C

When obtaining the vital signs of a client with multiple traumatic injuries, a nurse detects bradycardia, bradypnea, and systolic hypertension. The nurse must notify the physician immediately because these findings may reflect which complication?

Increased intracranial pressure (ICP)

What does the nurse recognize as the earliest sign of serious impairment of brain circulation related to increasing ICP?

Lethargy and stupor

A community health nurse is performing a home visit to a patient with amyotrophic lateral sclerosis (ALS). The nurse should prioritize assessments related to which of the following?

Respiratory function

A client is receiving hypothermic treatment for uncontrolled fever related to increased intracranial pressure (ICP). Which assessment finding requires immediate intervention?

Shivering

A nurse is caring for a client with a diagnosis of trigeminal neuralgia. Which activity is altered as a result of this diagnosis? a. chewing b. swallowing c. smelling d. tasting

a Trigeminal neuralgia is a painful condition that involves the fifth (V) cranial nerve (the trigeminal nerve) and is important to chewing.

The nurse is taking care of a client with a headache. In addition to administering medications, the nurse takes which measure to assist the client in reducing the pain associated with the headache? a. Apply warm or cool cloths to the forehead or back of the neck. b. Maintain hydration by drinking eight glasses of fluid a day. c. Perform the Heimlich maneuver. d. Use pressure-relieving pads or a similar type of mattress.

a Warmth promotes vasodilation; cool stimuli reduce blood flow.

An osmotic diuretic such as mannitol is given to the client with increased intracranial pressure (ICP) to a. control fever. b. control shivering. c. dehydrate the brain and reduce cerebral edema. d. reduce cellular metabolic demand.

c Osmotic diuretics draw water across intact membranes, thereby reducing the volume of brain and extracellular fluid. Antipyretics and a cooling blanket are used to control fever in the client with increased ICP. Chlorpromazine may be prescribed to control shivering in the client with increased ICP. Medications such as barbiturates are given to the client with increased ICP to reduce cellular metabolic demands.

A nurse assesses the patient's level of consciousness using the Glasgow Coma Scale. What score indicates severe impairment of neurologic function?

3

The nurse is assessing a patient with a suspected stroke. What assessment finding is most suggestive of a stroke? A) Facial droop B) Dysrhythmias C) Periorbital edema D) Projectile vomiting

Ans: A Feedback: Facial drooping or asymmetry is a classic abnormal finding on a physical assessment that may be associated with a stroke. Facial edema is not suggestive of a stroke and patients less commonly experience dysrhythmias or vomiting.

A nurse is participating in the emergency care of a patient who has just developed variceal bleeding. What intervention should the nurse anticipate? A) Infusion of intravenous heparin B) IV administration of albumin C) STAT administration of vitamin K by the intramuscular route D) IV administration of octreotide (Sandostatin)

Ans: D Feedback: Octreotide (Sandostatin)—a synthetic analog of the hormone somatostatin—is effective in decreasing bleeding from esophageal varices, and lacks the vasoconstrictive effects of vasopressin. Because of this safety and efficacy profile, octreotide is considered the preferred treatment regimen for immediate control of variceal bleeding. Vitamin K and albumin are not administered and heparin would exacerbate, not alleviate, bleeding.

The nurse is reviewing the medication administration record of a female patient who possesses numerous risk factors for stroke. Which of the womans medications carries the greatest potential for reducing her risk of stroke? A) Naproxen 250 PO b.i.d. B) Calcium carbonate 1,000 mg PO b.i.d. C) Aspirin 81 mg PO o.d. D) Lorazepam 1 mg SL b.i.d. PRN

C

A nurse is planning the care of a 28-year-old woman hospitalized with a diagnosis of myasthenia gravis. What approach would be most appropriate for the care and scheduling of diagnostic procedures for this patient? A) All at one time, to provide a longer rest period B) Before meals, to stimulate her appetite C) In the morning, with frequent rest periods D) Before bedtime, to promote rest

C) In the morning, with frequent rest periods

A client with a traumatic brain injury has already displayed early signs of increasing intracranial pressure (ICP). Which of the following would be considered late signs of increasing ICP?

Decerebrate posturing and loss of corneal reflex

Hepatitis type Source: feces Route of transmission: fecal-oral Not chronic Prevent by ensuring safe drinking water Only people with Hep A are at risk for it

Hepatitis E

When educating a patient about the use of antiseizure medication, what should the nurse inform the patient is a result of long-term use of the medication in women?

Osteoporosis

A patient with generalized seizure disorder has just had a seizure. The nurse would assess for what characteristic associated with the postictal state?

Urinary incontinence

A nurse caring for a patient with head trauma will be monitoring the patient for Cushing's triad. What will the nurse recognize as the symptoms associated with Cushing's triad? Select all that apply. a. Bradycardia b. Bradypnea c. Hypertension d. Tachycardia e. Pupillary constriction

a, b, c At a certain point as intracranial pressure increases due to an injury, the brain's ability to autoregulate becomes ineffective and decompensation (ischemia and infarction) begins. When this occurs, the patient exhibits significant changes in mental status and vital signs. The bradycardia, hypertension, and bradypnea associated with this deterioration are known as Cushing's triad, which is a grave sign.

When should the nurse plan the rehabilitation of a patient who is having an ischemic stroke? a. The day before the patient is discharged b. After the patient has passed the acute phase of the stroke c. After the nurse has received the discharge orders d. The day the patient has the stroke

b Although rehabilitation begins on the day the patient has the stroke, the process is intensified during convalescence and requires a coordinated team effort.

A nurse is monitoring a client for increasing intracranial pressure (ICP). Early signs of increased ICP include: a. pupillary changes. b. diminished responsiveness. d. decreasing blood pressure. d. elevated temperature.

b Usually, diminished responsiveness is the first sign of increasing ICP. Pupillary changes occur later. Increased ICP causes systolic blood pressure to rise. Temperature changes vary and may not occur even with a severe decrease in responsiveness.

A client is sitting in a chair and begins having a tonic-clonic seizure. The most appropriate nursing response is to:

carefully move the client to a flat surface and turn him on his side.

The nurse is caring for a patient having a hemorrhagic stroke. What position in the bed will the nurse maintain this patient? a. High-Fowler's b. Prone c. Supine d. Semi-Fowler's

d The head of the bed is elevated 15 to 30 degrees (semi-Fowler's position) to promote venous drainage and decrease intracranial pressure.

A patient comes to the ophthalmology clinic for an eye examination. The patient tells the nurse that he often sees floaters in his vision. How should the nurse best interpret this subjective assessment finding? A) This is a normal aging process of the eye. B) Glasses will minimize this phenomenon. C) The patient may be exhibiting signs of glaucoma. D) This may be a result of weakened ciliary muscles.

A

A patient has had a sudden loss of vision after head trauma. How should the nurse best describe the placement of items on the dinner tray? A) Explain the location of items using clock cues. B) Explain that each of the items on the tray is clearly separated. C) Describe the location of items from the bottom of the plate to the top. D) Ask the patient to describe the location of items before confirming their location.

A

A patient who just suffered a suspected ischemic stroke is brought to the ED by ambulance. On what should the nurses primary assessment focus? A) Cardiac and respiratory status B) Seizure activity C) Pain D) Fluid and electrolyte balance

A

A patient who suffered an ischemic stroke now has disturbed sensory perception. What principle should guide the nurses care of this patient? A) The patient should be approached on the side where visual perception is intact. B) Attention to the affected side should be minimized in order to decrease anxiety. C) The patient should avoid turning in the direction of the defective visual field to minimize shoulder subluxation. D) The patient should be approached on the opposite side of where the visual perception is intact to promote recovery.

A

The nurse has taken shift report on her patients and has been told that one patient has an ocular condition that has primarily affected the rods in his eyes. Considering this information, what should the nurse do while caring for the patient? A) Ensure adequate lighting in the patient's room. B) Provide a dimly lit room to aid vision by limiting contrast. C) Carefully point out color differences for the patient. D) Carefully point out fine details for the patient.

A

The nurse should recognize the greatest risk for the development of blindness in which of the following patients? A) A 58-year-old Caucasian woman with macular degeneration B) A 28-year-old Caucasian man with astigmatism C) A 58-year-old African American woman with hyperopia D) A 28-year-old African American man with myopia

A

The patient has been diagnosed with aphasia after suffering a stroke. What can the nurse do to best make the patients atmosphere more conducive to communication? A) Provide a board of commonly used needs and phrases. B) Have the patient speak to loved ones on the phone daily. C) Help the patient complete his or her sentences. D) Speak in a loud and deliberate voice to the patient.

A

The nurse caring for a client in a persistent vegetative state is regularly assessing for potential complications. The nurse should assess for which complications? Select all that apply. A) Contractures B) Hemorrhage C) Pressure ulcers D) Venous thromboembolism E) Pneumonia

A) Contractures C) Pressure ulcers D) Venous thromboembolism E) Pneumonia

Level C personal protective equipment has been deemed necessary in the response to an unknown substance. The nurse is aware that the equipment will include what? A. An air-purified respirator B. A uniform only C. A self-contained breathing apparatus D. A vapor-tight, chemical-resistant suit

A. An air-purified respirator

A client is recovering from intracranial surgery performed approximately 24 hours ago and is complaining of a headache that the client rates at 8 on a 10-point pain scale. What nursing action is most appropriate?

Administer morphine sulfate as prescribed.

A patient who is being treated in the hospital for a spinal cord injury is advocating for the removal of his urinary catheter, stating that he wants to try to resume normal elimination. What principle should guide the care team's decision regarding this intervention? A) Urinary retention can have serious consequences in patients with SCIs. B) Urinary function is permanently lost following an SCI. C) Urinary catheters should not remain in place for more than 7 days. D) Overuse of urinary catheters can exacerbate nerve damage.

Ans: A Feedback: Bladder distention, a major cause of autonomic dysreflexia, can also cause trauma. For this reason, removal of a urinary catheter must be considered with caution. Extended use of urinary catheterization is often necessary following SCI. The effect of a spinal cord lesion on urinary function depends on the level of the injury. Catheter use does not cause nerve damage, although it is a major risk factor for UTIs.

The nurse is caring for a patient who exhibits abnormal results of the Weber test and Rinne test. The nurse should suspect dysfunction involving what cranial nerve? A. Trigeminal B. Acoustic C. Trochlear D. Hypoglossal

B. Acoustic

A patient is admitted to an inpatient unit after undergoing paracentesis for ascites. Which lab value is MOST relevant for the nurse monitor, in addition to the patient's clinical presentation, after this treatment: A. Serum sodium B. Serum albumin C. C-reactive protein D. Serum potassium

B. Serum albumin

A patient with Guillain-Barré syndrome has experienced a sharp decline in vital capacity. What is the nurse's most appropriate action? A) Administer bronchodilators as ordered. B) Remind the patient of the importance of deep breathing and coughing exercises. C) Prepare to assist with intubation. D) Administer supplementary oxygen by nasal cannula.

C) Prepare to assist with intubation.

The nurse is discharging a patient home after mastoid surgery. What should the nurse include in discharge teaching? A) "Try to induce a sneeze every 4 hours to equalize pressure." B) "Be sure to exercise to reduce fatigue." C) "Avoid sleeping in a side-lying position." D) "Don't blow your nose for 2 to 3 weeks."

D

The registered nurse taking shift report learns that an assigned patient is blind. How should the nurse best communicate with this patient? A) Provide instructions in simple, clear terms. B) Introduce herself in a firm, loud voice at the doorway of the room. C) Lightly touch the patient's arm and then introduce herself. D) State her name and role immediately after entering the patient's room.

D

The nurse caring for a patient in ICU diagnosed with Guillain-Barré syndrome should prioritize monitoring for what potential complication? A) Impaired skin integrity B) Cognitive deficits C) Hemorrhage D) Autonomic dysfunction

D) Autonomic dysfunction

A middle-aged woman has sought care from her primary care provider and undergone diagnostic testing that has resulted in a diagnosis of MS. What sign or symptom is most likely to have prompted the woman to seek care? A) Cognitive declines B) Personality changes C) Contractures D) Difficulty in coordination

D) Difficulty in coordination

The nurse is creating a plan of care for a patient who has a recent diagnosis of MS. Which of the following should the nurse include in the patient's care plan? A) Encourage patient to void every hour. B) Order a low-residue diet. C) Provide total assistance with all ADLs. D) Instruct the patient on daily muscle stretching.

D) Instruct the patient on daily muscle stretching.

A nurse is assessing a client who has been in a motor vehicle collision. The client directly and accurately answers questions. The nurse notes a contusion to the client's forehead; the client reports a headache. Assessing the client's pupils, what reaction would confirm increasing intracranial pressure? a. unequal response b. equal response c. rapid response d. constricted response

a In increased ICP, the pupil response is unequal. One pupil responds more sluggishly than the other or becomes fixed and dilated.

A client who has experienced an ischemic stroke has been admitted to the medical unit. The client's family is adamant that she remain on bed rest to hasten her recovery and to conserve energy. What principle of care should inform the nurse's response to the family? a. The client should mobilize as soon as she is physically able. b. To prevent contractures and muscle atrophy, bed rest should not exceed 4 weeks. c. The client should remain on bed rest until she expresses a desire to mobilize. d. Lack of mobility will greatly increase the client's risk of stroke recurrence.

a As soon as possible, the client is assisted out of bed and an active rehabilitation program is started. Delaying mobility causes complications, but not necessarily stroke recurrence. Mobility should not be withheld until the client initiates.

Which of the following, if left untreated, can lead to an ischemic stroke? a. Atrial fibrillation b. Cerebral aneurysm c. Arteriovenous malformation (AVM) d. Ruptured cerebral arteries

a Atrial fibrillation is the most frequently diagnosed arrhythmia in the United States. If left untreated, it can lead to an ischemic stroke. Cerebral hemorrhage, arteriovenous malformation, and cerebral hemorrhage can lead to a hemorrhagic stroke. Cerebral aneurysm, arteriovenous malformations, and ruptured cerebral arteries can lead to hemorrhagic stroke.

The nurse is caring for a client who is in status epilepticus. What medication should the nurse anticipated administering to halt the seizure immediately? a. Intravenous phenobarbital b. Intravenous diazepam c. Oral lorazepam d. Oral phenytoin

b Medical management of status epilepticus includes IV diazepam and IV lorazepam given slowly in an attempt to halt seizures immediately. Other medications (phenytoin, phenobarbital) are given later to maintain a seizure-free state. Oral medications are not given during status epilepticus.

A school nurse is called to the playground where a 6-year-old girl has been found sitting unresponsive and "staring into space," according to the playground supervisor. How would the nurse document the girl's activity in her chart at school? a. Generalized seizure b. Absence seizure c. Focal seizure d. Unclassified seizure

b Staring episodes characterize an absence seizure, whereas focal seizures, generalized seizures, and unclassified seizures involve uncontrolled motor activity.

A patient having an acute stroke with no other significant medical disorders has a blood glucose level of 420 mg/dL. What significance does the hyperglycemia have for this patient? a. The patient has new onset diabetes. b. This is significant for poor neurologic outcomes. c. The patient has developed diabetes insipidus due to the location of the stroke. d. The patient has liver failure.

b Hyperglycemia has been associated with poor neurologic outcomes in acute stroke and should be treated if the blood glucose is above 140 mg/dL (Summers et al., 2009).

A patient with amyotrophic lateral sclerosis (ALS) asks if the nurse has heard of a drug that will prolong the patient's life. The nurse knows that there is a medication that may prolong the life by 3 to 6 months. To which medication is the patient referring? a. Baclofen b. Riluzole c. Dantrolene sodium d. Diazepam

b Riluzole, a glutamate antagonist, has been shown to prolong survival for persons with ALS for 3 to 6 months.

A patient who has suffered a stroke begins having complications regarding spasticity in the lower extremity. What ordered medication does the nurse administer to help alleviate this problem? a. Diphenhydramine (Benadryl) b. Lioresal (Baclofen) c. Heparin d. Pregabalin (Lyrica)

b Spasticity, particularly in the hand, can be a disabling complication after stroke. Botulinum toxin type A injected intramuscularly into wrist and finger muscles has been shown to be effective in reducing this spasticity (although the effect is temporary, typically lasting 2 to 4 months) (Teasell, Foley, Pereira, et al., 2012). Other treatments for spasticity may include stretching, splinting, and oral medications such as baclofen (Lioresal).

After a seizure, the nurse should place the patient in which of the following positions to prevent complications? a. High Fowler's, to prevent aspiration b. Side-lying, to facilitate drainage of oral secretions c. Supine, to rest the muscles of the extremities d. Semi-Fowler's, to promote breathing

b To prevent complications, the patient is placed in the side-lying position to facilitate drainage of oral secretions, and suctioning is performed, if needed, to maintain a patent airway and prevent aspiration.

The nurse is performing stroke risk screenings at a hospital open house. The nurse has identified four clients who might be at risk for a stroke. Which client is likely at the highest risk for a hemorrhagic stroke? a. White female, age 60, with history of excessive alcohol intake b. White male, age 60, with history of uncontrolled hypertension c. Black male, age 60, with history of diabetes d. Black male, age 50, with history of smoking

b Uncontrolled hypertension is the primary cause of a hemorrhagic stroke. Control of hypertension, especially in individuals over 55 years of age, clearly reduces the risk for hemorrhagic stroke. Additional risk factors are increased age, male gender, and excessive alcohol intake. Another high-risk group includes Black people, where the incidence of first stroke is almost twice that as in White people.

A nurse is providing care to a client who has had a stroke. Which symptoms are consistent with right-sided stroke? a. expressive aphasia, defects in the right visual fields, problems with abstract thinking b. impulsive behavior, poor judgment, deficits in left visual fields c. problems with abstract thinking, impairment of short-term memory, poor judgment d. cautious behavior, deficits in left visual fields, misjudgment of distances

b expressive aphasia, defects in the right visual fields, problems with abstract thinking impulsive behavior, poor judgment, deficits in left visual fields problems with abstract thinking, impairment of short-term memory, poor judgment cautious behavior, deficits in left visual fields, misjudgment of distances

One defining characteristic of a complex partial seizure versus a simple partial seizure is the presence of which of the following? a. Sensory symptoms b. Motor symptoms c. Impaired consciousness d. Compound forms

c A complex partial seizure is characterized by complex symptoms with the impairment of consciousness. A simple partial seizure generally occurs without impairment of consciousness.

The nurse is admitting a 55-year-old male patient diagnosed with a retinal detachment in his left eye. While assessing this patient, what characteristic symptom would the nurse expect to find? A) Flashing lights in the visual field B) Sudden eye pain C) Loss of color vision D) Colored halos around lights

A

A nurse is caring for a patient admitted with cluster headaches. The nurse knows that in the early phase of a cluster headache what is required? A) Dim lighting B) Abortive medication therapy C) Quiet D) Rest

ANS: B A migraine or a cluster headache in the early phase requires abortive medication therapy instituted as soon as possible. Dim lighting, quiet, and rest are necessary for migraines; they are not required in the early phase of a cluster headache.

A clinic nurse is caring for a patient diagnosed with migraine headaches. When doing patient teaching, the patient questions the nurse regarding alcohol consumption. What would the nurse be correct in telling the patient about alcohol's effects? A) Alcohol causes hormone fluctuation. B) Alcohol causes vasodilation of the blood vessels. C) Alcohol has an excitatory effect on the CNS. D) Alcohol diminishes endorphins in the brain.

ANS: B Alcohol causes vasodilation of the blood vessels. Alcohol has a depressant effect on the central nervous system (CNS). Alcohol does not cause hormone fluctuations, nor does it decrease endorphins (morphine-like substances produced by the body) in the brain.

The nurse is developing a plan of care for a patient newly diagnosed with Bell's palsy. The nurse's plan of care should address what characteristic manifestation of this disease? A) Tinnitus B) Facial paralysis C) Pain at the base of the tongue D) Diplopia

Ans: B Feedback: Bell's palsy is characterized by facial dysfunction, weakness, and paralysis. It does not result in diplopia, pain at the base of the tongue, or tinnitus.

A patient has just been diagnosed with Parkinson's disease and the nurse is planning the patient's subsequent care for the home setting. What nursing diagnosis should the nurse address when educating the patient's family? A) Risk for infection B) Impaired spontaneous ventilation C) Unilateral neglect D) Risk for injury

Ans: D Feedback: Individuals with Parkinson's disease face a significant risk for injury related to the effects of dyskinesia. Unilateral neglect is not characteristic of the disease, which affects both sides of the body. Parkinson's disease does not directly constitute a risk for infection or impaired respiration.

An ED nurse has just received a call from EMS that they are transporting a 17-year-old man who has just sustained a spinal cord injury (SCI). The nurse recognizes that the most common cause of this type of injury is what? A) Sports-related injuries B) Acts of violence C) Injuries due to a fall D) Motor vehicle accidents

Ans: D Feedback: The most common causes of SCIs are motor vehicle crashes (46%), falls (22%), violence (16%), and sports (12%).

A patient presents at the ED after receiving a chemical burn to the eye. What would be the nurse's initial intervention for this patient? A) Generously flush the affected eye with a dilute antibiotic solution. B) Generously flush the affected eye with normal saline or water. C) Apply a patch to the affected eye. D) Apply direct pressure to the affected eye.

B

An elderly patient is being discharged home. The patient lives alone and has atrophy of his olfactory organs. The nurse tells the patient's family that it is essential that the patient have what installed in the home? A. Nonslip mats B. A smoke detector C. Baseboard heaters D. Grab bars

B. A smoke detector

A patient has a poor prognosis after being involved in a motor vehicle accident resulting in a head injury. As the patient's ICP increases and condition worsens, the nurse knows to assess for indications of approaching death. These indications include which of the following? A. Dry mucous membranes B. Loss of brain stem reflexes C. Hemiplegia D. Signs of internal bleeding

B. Loss of brain stem reflexes

A child goes to the school nurse and complains of not being able to hear the teacher. What test could the school nurse perform that would preliminarily indicate hearing loss? A) Audiometry B) Rinne test C) Whisper test D) Weber test

C

The critical care nurse is admitting a patient in myasthenic crisis to the ICU. The nurse should prioritize what nursing action in the immediate care of this patient? A) Suctioning secretions B) Facilitating ABG analysis C) Providing ventilatory assistance D) Administering tube feedings

C) Providing ventilatory assistance

The nurse is preparing to provide care for a patient diagnosed with myasthenia gravis. The nurse should know that the signs and symptoms of the disease are the result of what? A) Genetic dysfunction B) Upper and lower motor neuron lesions C) Decreased conduction of impulses in an upper motor neuron lesion D) A lower motor neuron lesion

D) A lower motor neuron lesion

A client has a 12-year history of migraine headaches and is frustrated over how these headaches impact lifestyle. What would the nurse indicate to the client is the origin of migraines? a. vascular b. muscular c. light d. endocrine

a Migraine headaches, which are recurrent and severe and last for a day or more, have a vascular origin.

A woman has been brought to the emergency department (ED) by her distraught husband who believes that she has had a stroke. A rapid assessment by the care team confirms that the husband's suspicions are likely accurate, and the woman is being screened for the possible administration of recombinant tissue plasminogen activator (r tPA). Which of the following factors would contraindicate the use of tPA? a. The woman's stroke has a hemorrhagic etiology. b. The woman is older than 80 years of age. c. The woman has previously had a stroke. d. The woman has hypertension and type 1 diabetes.

a tPA is contraindicated in hemorrhagic stroke because it would greatly exacerbate cerebral bleeding. Older age, previous stroke, hypertension, and diabetes do not necessarily contraindicate the use of tPA.

A client with an inoperable brain tumor says to the nurse, "I'm so afraid that I'm going to die alone." What is the nurse's best response? a. "You sound frightened." b. "You are not going to die." c. "There is nothing to be afraid of." d. "It won't be as bad as you think."

a In this scenario, the nurse stating "You sound frightened" is an example of reflective technique; it focuses on the client's feelings and encourages verbalization. The other statements deny the client's feelings.

The nurse is caring for a patient postoperatively after intracranial surgery for the treatment of a subdural hematoma. The nurse observes an increase in the patient's blood pressure from the baseline and a decrease in the heart rate from 86 to 54. The patient has crackles in the bases of the lungs. What does the nurse suspect is occurring? a. Increased ICP b. Exacerbation of uncontrolled hypertension c. Infection d. Increase in cerebral perfusion pressure

a Increased ICP and bleeding are life threatening to the patient who has undergone intracranial surgery. An increase in blood pressure and decrease in pulse with respiratory failure may indicate increased ICP.

While caring for a client with a seizure disorder, the nurse observes a sudden, brief jerking of the client's left arm. Most likely, the client has which type of seizure disorder? a. myoclonic seizure b. tonic-clonic seizure c. partial seizure d. absence seizure

a Myoclonic seizures are characterized by sudden, excessive jerking of the arms, legs, or entire body. The seizures are brief.

The nurse is providing nonpharmacologic interventions for pain relief to a client with a tension headache. Which techniques may the nurse use? Select all that apply. a. Use guided imagery. b. Play soothing music. c. Allow interactions with friends and family members. d. Allow watching TV.

a, b Guided imagery and soothing music can reduce tension and relieve a tension headache. Stimuli should be reduced to help alleviate anxiety and reduce pain.

A client has suffered several migraines per month for the last 4 months. The physician prescribes prophylactic drug therapy. What is the rationale behind this action? Select all that apply. a. possible reduction in frequency of attacks b. possible reduction of migraine intensity c. possible reduction in migraine duration d. prevention of all migraines

a, b, c Prophylactic drug therapy may be necessary if migraine headaches occur several times a month and produce severe impairment, or if acute attacks are not adequately relieved. Although this may not prevent all migraines, it may reduce the duration, frequency, and intensity of attacks.

During assessment of a patient who has been taking dilantin for seizure management for 3 years, the nurse notices one of the side effects that should be reported. What is that side effect? a. Alopecia b. Gingival hyperplasia c. Diplopia d. Ataxia

b Side-effects of dilantin include visual problems, hirsutism, gingival hyperplasia, arrhythmias, dysarthria, and nystagmus.

A client with increased intracranial pressure has a cerebral perfusion pressure (CPP) of 40 mm Hg. How should the nurse interpret the CPP value? a. The CPP is high. b. The CPP is low. c. The CPP is within normal limits. d. The CPP reading is inaccurate.

b The normal CPP is 70 to 100 mm Hg. Therefore, a CPP of 40 mm Hg is low. Changes in intracranial pressure (ICP) are closely linked with cerebral perfusion pressure (CPP). The CPP is calculated by subtracting the ICP from the mean arterial pressure (MAP). Patients with a CPP of less than 50 mm Hg experience irreversible neurologic damage.

A nurse is working with a student nurse who is caring for a client with an acute bleeding cerebral aneurysm. Which action by the student nurse requires further intervention? a. Positioning the client to prevent airway obstruction b. Keeping the client in one position to decrease bleeding c. Administering I.V. fluid as ordered and monitoring the client for signs of fluid volume excess d. Maintaining the client in a quiet environment

b The student nurse shouldn't keep the client in one position. She should carefully reposition the client often (at least every hour). The client needs to be positioned so that a patent airway can be maintained. Fluid administration must be closely monitored to prevent complications such as increased intracranial pressure. The client must be maintained in a quiet environment to decrease the risk of rebleeding.

A community health nurse is conducting a workshop for unlicensed care providers who work in a chain of long-term care facilities. The nurse is teaching the participants about the signs and symptoms of stroke. What signs and symptoms should the nurse identify? Select all that apply. a. Epistaxis (nosebleed) b. Confusion c. Sudden numbness d. Sudden ear pain e. Visual disturbances

b, c, e The most common symptoms of stroke include numbness or weakness of the face, arm, or leg, especially on one side of the body; confusion or change in mental status; trouble speaking or understanding speech; visual disturbances; difficulty walking, dizziness, or loss of balance or coordination; and sudden, severe headache.

The nurse is educating a patient with a seizure disorder. What nutritional approach for seizure management would be beneficial for this patient? a. Low in fat b. Restricts protein to 10% of daily caloric intake c. High in protein and low in carbohydrate d. At least 50% carbohydrate

c A dietary intervention, referred to as the ketogenic diet, may be helpful for control of seizures in some patients. This high-protein, low-carbohydrate, high-fat diet is most effective in children whose seizures have not been controlled with two antiseizure medications, but it is sometimes used for adults who have had poor seizure control (Mosek, Natour, Neufeld, et al., 2009).

Which is a late sign of increased intracranial pressure (ICP)? a. Irritability b. Slow speech c. Altered respiratory patterns d. Headache

c Altered respiratory patterns are late signs of increased ICP and may indicate pressure or damage to the brainstem. Headache, irritability, and any change in LOC are early signs of increased ICP. Speech changes, such as slowed speech or slurring, are also early signs of increased ICP.

A nurse who provides care at a community clinic is in contact with a diverse group of patients. Which of the following individuals most clearly displays risk factors for stroke? a. A man who is receiving oral antibiotics for the treatment of a chlamydial infection b. A woman who has osteoporosis, a history of fractures, and a family history of stroke c. An obese woman with a history of atrial fibrillation and type 2 diabetes d. A 70-year-old man who has benign prostatic hyperplasia and early stage Alzheimer's disease

c Obesity, atrial fibrillation, and type 2 diabetes are all highly significant risk factors for stroke. None of the other listed individuals displays multiple risk factors for stroke.

The nurse practitioner is able to correlate a patient's neurologic deficits with the location in the brain affected by ischemia or hemorrhage. For a patient with a left hemispheric stroke, the nurse would expect to see: a. Spatial-perceptual deficits. b. Left visual field deficit. c. Right-sided paralysis. d. Impulsive behavior.

c A left hemispheric stroke will cause right-sided weakness or paralysis. Because upper motor neurons decussate, a disturbance on one side of the body can cause damage on the opposite side of the brain. Refer to Box 47-2 in the text.

The nurse is discharging home a client who had a stroke. The client has a flaccid right arm and leg and is experiencing urinary incontinence. The nurse makes a referral to a home health nurse because of an awareness of what common client response to a change in body image? a. Confusion b. Uncertainty c. Depression d. Disassociation

c Depression is a common and serious problem in the client who has had a stroke. It can result from a profound disruption in his or her life and changes in total function, leaving the client with a loss of independence. The nurse needs to encourage the client to verbalize feelings to assess the effect of the stroke on self-esteem. Confusion, uncertainty, and disassociation are not the most common client response to a change in body image, although each can occur in some clients.

A client diagnosed with a hemorrhagic stroke has been admitted to the neurologic ICU. The nurse knows that teaching for the client and family needs to begin as soon as the client is settled on the unit and will continue until the client is discharged. What will family education need to include? a. How to differentiate between hemorrhagic and ischemic stroke b. Risk factors for ischemic stroke c. How to correctly modify the home environment d. Techniques for adjusting the client's medication dosages at home

c For a client with a hemorrhagic stroke, teaching addresses the use of assistive devices or modification of the home environment to help the client live with the disability. This is more important to the client's needs than knowing about risk factors for ischemic stroke. It is not necessary for the family to differentiate between different types of strokes. Medication regimens should never be altered without consultation.

Which of the following the are early manifestations of liver cancer? Select all that apply. a) Fever b) Continuous aching in the back c) Vomiting d) Pain e) Increased appetite f) Jaundice

• Pain • Continuous aching in the back Explanation: Early manifestations of liver cancer include pain and continuous dull aching in the right upper quadrant epigastrium or back. Weight loss, anorexia, and anemia may occur. Jaundice is present only if the larger bile ducts are occluded by the pressure of malignant nodules in the hilum of the liver. Fever and vomiting are not associated manifestations.

The nurse is preparing a care plan for a patient with hepatic cirrhosis. Which of the following nursing diagnoses are appropriate? Select all that apply. a) Activity intolerance related to fatigue, general debility, muscle wasting, and discomfort b) Altered nutrition, more than body requirements, related to decreased activity and bed rest c) Risk for injury related to altered clotting mechanisms d) Disturbed body image related to changes in appearance, sexual dysfunction, and role function e) Urinary incontinence related to general debility and muscle wasting

• Risk for injury related to altered clotting mechanisms • Disturbed body image related to changes in appearance, sexual dysfunction, and role function • Activity intolerance related to fatigue, general debility, muscle wasting, and discomfort Correct Explanation: Risks for injury, activity intolerance, and disturbed body image are priority nursing diagnoses. The appropriate nursing diagnosis related to nutrition would be altered nutrition, less than body requirements, related to chronic gastritis, decreased GI motility, and anorexia. Urinary incontinence is not generally a concern with hepatic cirrhosis.

A physician orders therapy with tissue plasminogen activator for a client. The nurse alerts the physician to a potential problem when reviewing the client's chart and seeing that the client had major surgery within the last:

14 An exclusion criterion for therapy with tissue plasminogen activator is major surgery or invasive procedure within the past 14 days.

The nurse is preparing to administer tissue plasminogen activator (t-PA) to a patient who weighs 132 lb. The order reads 0.9 mg/kg t-PA. The nurse understands that 10% of the calculated dose is administered as an IV bolus over 1 minute, and the remaining dose (90%) is administered IV over 1 hour via an infusion pump. How many milligrams IV bolus over 1 minute will the nurse initially administer?

5.4 mg

The nurse is planning discharge education for a patient with trigeminal neuralgia. The nurse knows to include information about factors that precipitate an attack. What would the nurse be correct in teaching the patient to avoid? A) Washing his face B) Exposing his skin to sunlight C) Using artificial tears D) Drinking large amounts of fluids

A) Washing his face

A public health nurse is teaching a health promotion workshop that focuses on vision and eye health. What should this nurse cite as the most common causes of blindness and visual impairment among adults over the age of 40? Select all that apply. A) Diabetic retinopathy B) Trauma C) Macular degeneration D) Cytomegalovirus E) Glaucoma

A,C,E

A patient who has been on long-term phenytoin (Dilantin) therapy is admitted to your unit. You would be correct in assessing for what adverse effect of this therapy? A) Bradycardia B) Diarrhea C) Gingivival hyperplasia D) Weight gain

ANS: C Gingivival hyperplasia (swollen and tender gums) can be associated with long-term phenytoin (Dilantin) use. Bradycardia, diarrhea, and weight gain are not associated with dilantin therapy. Adverse effects noted with dilantin therapy are tachycardia, constipation, and weight loss

A client is transferred to the intensive care unit after evacuation of a subdural hematoma. Which nursing intervention reduces the client's risk of increased intracranial pressure (ICP)?

Administering a stool softener as ordered

A patient is admitted to the neurologic ICU with a suspected diffuse axonal injury. What would be the primary neuroimaging diagnostic tool used on this patient to evaluate the brain structure? A) MRI B) PET scan C) X-ray D) Ultrasound

Ans: A Feedback: CT and MRI scans, the primary neuroimaging diagnostic tools, are useful in evaluating the brain structure. Ultrasound would not show the brain nor would an x-ray. A PET scan shows brain function, not brain structure.

A patient is admitted to the neurologic ICU with a spinal cord injury. When assessing the patient the nurse notes there is a sudden depression of reflex activity in the spinal cord below the level of injury. What should the nurse suspect? A) Epidural hemorrhage B) Hypertensive emergency C) Spinal shock D) Hypovolemia

Ans: C Feedback: In spinal shock, the reflexes are absent, BP and heart rate fall, and respiratory failure can occur. Hypovolemia, hemorrhage, and hypertension do not cause this sudden change in neurologic function.

33. A patient with end-stage liver disease has developed hypervolemia. What nursing interventions would be most appropriate when addressing the patient's fluid volume excess? Select all that apply. A) Administering diuretics B) Administering calcium channel blockers C) Implementing fluid restrictions D) Implementing a 1500 kcal/day restriction E) Enhancing patient positioning

Ans: A, C, E Feedback: Administering diuretics, implementing fluid restrictions, and enhancing patient positioning can optimize the management of fluid volume excess. Calcium channel blockers and calorie restriction do not address this problem.

A nurse is caring for a patient who experiences debilitating cluster headaches. The patient should be taught to take appropriate medications at what point in the course of the onset of a new headache? A) As soon as the patient's pain becomes unbearable B) As soon as the patient senses the onset of symptoms C) Twenty to 30 minutes after the onset of symptoms D) When the patient senses his or her symptoms peaking

Ans: B Feedback:A migraine or a cluster headache in the early phase requires abortive medication therapy instituted as soon as possible. Delaying medication administration would lead to unnecessary pain.

A hospitalized patient with impaired vision must get a picture in his or her mind of the hospital room and its contents in order to mobilize independently and safely. What must the nurse monitor in the patient's room? A) That a commode is always available at the bedside B) That all furniture remains in the same position C) That visitors do not leave items on the bedside table D) That the patient's slippers stay under the bed

B

Cytomegalovirus (CMV) is the most common cause of retinal inflammation in patients with AIDS. What drug, surgically implanted, is used for the acute stage of CMV retinitis? A) Pilocarpine B) Penicillin C) Ganciclovir D) Gentamicin

C

Hepatitis type Source: blood/bodily fluids Route of transmission: percutaneous permucosal (needles/sex) chronic Prevent by pre/post-exposure immunization Transmitted to baby at birth Worldwide cause of liver cirrhosis

Hepatitis B

A stroke victim is experiencing memory loss and impaired learning capacity. The nurse knows that brain damage has most likely occurred in which lobe? a. Frontal b. Occipital c. Parietal d. Temporal

a If damage has occurred to the frontal lobe, learning capacity, memory, or other higher cortical intellectual functions may be impaired. Such dysfunction may be reflected in a limited attention span, difficulties in comprehension, forgetfulness, and a lack of motivation.

A client has a 12-year history of cluster headaches. After the client describes the characteristics of the head pain, the nurse begins to discuss its potential causes. What would the nurse indicate that the origin of the headaches is: a. unknown. b. muscular. c. vasodilating agents. d. endocrine.

a Although cluster headaches can be triggered by vasodilating agents, the cause of cluster headaches is unknown.

A patient presents to the emergency room with complaints of having an "exploding headache" for the last 2 hours. The patient is immediately seen by a triage nurse who suspects the patient is experiencing a stroke. Which of the following is a possible cause based on the characteristic symptom? a. Large artery thrombosis b. Cerebral aneurysm c. Cardiogenic emboli d. Small artery thrombosis

b A cerebral aneurysm is a type of hemorrhagic stroke that is characterized by an exploding headache.

A nurse is caring for a client diagnosed with a hemorrhagic stroke. When creating this client's plan of care, what goal should be prioritized? a. Prevent complications of immobility. b. Maintain and improve cerebral tissue perfusion. c. Relieve anxiety and pain. d. Relieve sensory deprivation.

b Each of the listed goals is appropriate in the care of a client recovering from a stroke. However, promoting cerebral perfusion is a priority physiologic need, on which the client's survival depends.

The nurse is caring for a patient on the neurological unit who is in status epilepticus. What medication does the nurse anticipate being given to halt the seizure? a. IV phenobarbital b. IV diazepam c. IV lidocaine d. Oral phenytoin

b Status epilepticus (acute prolonged seizure activity) is a series of generalized seizures that occur without full recovery of consciousness between attacks. Medical management of status epilepticus includes IV diazepam (Valium) and IV lorazepam (Ativan), given slowly in an attempt to halt seizures immediately. Other medications (phenytoin, phenobarbital) are given later to maintain a seizure-free state.

A patient who has had a previous stroke and is taking warfarin tells the nurse that he started taking garlic to help reduce his blood pressure. The nurse knows that garlic when taken together with warfarin will produce which type of interaction? a. No drug to drug interactions, may be taken together b. Can greatly increase the international normalization ratio (INR), increasing the risk of bleeding c. Can cause platelet aggregation, increasing the risk of blood clotting d. May increase cerebral blood flow, causing migraine headaches

b Taken together warfarin and garlic can greatly increase the INR, increasing the risk of bleeding

The neurologic ICU nurse is admitting a client with increased intracranial pressure. How should the nurse best position the client? a. Position the client supine. b. Maintain head of bed (HOB) elevated at 30 to 45 degrees. c. Position client in prone position. d. Maintain bed in Trendelenburg position.

b The client with increased ICP should be placed with the HOB elevated 30 to 45 degrees, with the neck in neutral alignment. Each of the other listed positions would cause a dangerous elevation in ICP.

Which is the earliest sign of increasing intracranial pressure? a. Vomiting b. Change in level of consciousness c. Headache d. Posturing

b The earliest sign of increasing intracranial pressure (ICP) is a change in level of consciousness. Other manifestations of increasing ICP are vomiting, headache, and posturing.

The nurse is caring for a client who is postoperative following a craniotomy. When writing the plan of care, the nurse identifies a diagnosis of "deficient fluid volume related to fluid restriction and osmotic diuretic use." What is the nurse's most appropriate intervention for this diagnosis? a. Change the client's position as indicated. b. Monitor serum electrolytes. c. Maintain NPO status. d. Monitor arterial blood gas (ABG) values.

b The postoperative fluid regimen depends on the type of neurosurgical procedure and is determined on an individual basis. The volume and composition of fluids are adjusted based on daily serum electrolyte values, along with fluid intake and output. Fluids may have to be restricted in clients with cerebral edema. Changing the client's position, maintaining an NPO status, and monitoring ABG values do not relate to the nursing diagnosis of deficient fluid volume.

A patient is admitted via ambulance to the emergency room of a stroke center at 1:30 p.m. with symptoms that the patient said began at 1:00 p.m. Within 1 hour, an ischemic stroke had been confirmed and the doctor ordered tPA. The nurse knows to give this drug no later than what time? a. 2:00 p.m. b. 3:00 p.m. c. 4:00 p.m. d. 7:00 p.m.

c Tissue plasminogen activator (tPA) must be given within 3 hours after symptom onset. Therefore, since symptom onset was 1:00 pm, the window of opportunity ends at 4:00 pm.

A nurse is collaborating with the interdisciplinary team to help manage a client's recurrent headaches. What aspect of the client's health history should the nurse identify as a potential contributor to the client's headaches? a. The client leads a sedentary lifestyle. b. The client takes vitamin D and calcium supplements. c. The client takes vasodilators for the treatment of angina. d. The client has a pattern of weight loss followed by weight gain.

c Vasodilators are known to contribute to headaches. Weight fluctuations, sedentary lifestyle, and vitamin supplements are not known to have this effect.

The nurse is providing care for a client who is withdrawing from heavy alcohol use. The nurse and other members of the care team are present at the bedside when the client has a seizure. In preparation for documenting this clinical event, the nurse should note which of the following? a. The ability of the client to follow instructions during the seizure. b. The success or failure of the care team to physically restrain the client. c. The client's ability to explain his seizure during the postictal period. d. The client's activities immediately prior to the seizure.

d Before and during a seizure, the nurse observes the circumstances before the seizure, including visual, auditory, or olfactory stimuli; tactile stimuli; emotional or psychological disturbances; sleep; and hyperventilation. Communication with the client is not possible during a seizure and physical restraint is not attempted. The client's ability to explain the seizure is not clinically relevant.

A client diagnosed with a cerebral aneurysm reports a severe headache to the nurse. What action is a priority for the nurse? a. Sit with the client for a few minutes. b. Administer an analgesic. c. Inform the nurse manager. d. Call the health care provider immediately.

d A headache may be an indication that the aneurysm is leaking. The nurse should notify the health care provider immediately. The health care provider will decide whether administration of an analgesic is indicated. Informing the nurse manager is not necessary. Sitting with the client is appropriate, once the health care provider has been notified of the change in the client's condition.

A nurse in the ICU is providing care for a client who has been admitted with a hemorrhagic stroke. The nurse is performing frequent neurologic assessments and observes that the client is becoming progressively more drowsy over the course of the day. What is the nurse's best response to this assessment finding? a. Report this finding to the health care provider as an indication of decreased metabolism. b. Provide more stimulation to the client and monitor the client closely. c. Recognize this as the expected clinical course of a hemorrhagic stroke. d. Report this to the health care provider as a possible sign of clinical deterioration.

d Alteration in LOC often is the earliest sign of deterioration in a client with a hemorrhagic stroke. Drowsiness and slight slurring of speech may be early signs that the LOC is deteriorating. This finding is unlikely to be the result of metabolic changes and it is not expected. Stimulating a client with an acute stroke is usually contraindicated.

A patient who has experienced an ischemic stroke has been admitted to the medical unit. The patients family in adamant that she remain on bed rest to hasten her recovery and to conserve energy. What principle of care should inform the nurses response to the family? A) The patient should mobilize as soon as she is physically able. B) To prevent contractures and muscle atrophy, bed rest should not exceed 4 weeks. C) The patient should remain on bed rest until she expresses a desire to mobilize. D) Lack of mobility will greatly increase the patients risk of stroke recurrence.

A

A patient with glaucoma has presented for a scheduled clinic visit and tells the nurse that she has begun taking an herbal remedy for her condition that was recommended by a work colleague. What instruction should the nurse provide to the patient? A) The patient should discuss this new remedy with her ophthalmologist promptly. B) The patient should monitor her IOP closely for the next several weeks. C) The patient should do further research on the herbal remedy. D) The patient should report any adverse effects to her pharmacist.

A

The nurse is teaching a patient with Guillain-Barré syndrome about the disease. The patient asks how he can ever recover if demyelination of his nerves is occurring. What would be the nurse's best response? A) "Guillain-Barré spares the Schwann cell, which allows for remyelination in the recovery phase of the disease." B) "In Guillain-Barré, Schwann cells replicate themselves before the disease destroys them, so remyelination is possible." C) "I know you understand that nerve cells do not remyelinate, so the physician is the best one to answer your question." D) "For some reason, in Guillain-Barré, Schwann cells become activated and take over the remyelination process."

A) "Guillain-Barré spares the Schwann cell, which allows for remyelination in the recovery phase of the disease."

You are the clinic nurse caring for a patient with a recent diagnosis of myasthenia gravis. The patient has begun treatment with pyridostigmine bromide (Mestinon). What change in status would most clearly suggest a therapeutic benefit of this medication? A) Increased muscle strength B) Decreased pain C) Improved GI function D) Improved cognition

A) Increased muscle strength

A 35-year-old woman is diagnosed with a peripheral neuropathy. When making her plan of care, the nurse knows to include what in patient teaching? Select all that apply. A) Inspect the lower extremities for skin breakdown. B) Footwear needs to be accurately sized. C) Immediate family members should be screened for the disease. D) Assistive devices may be needed to reduce the risk of falls. E) Dietary modifications are likely necessary.

A) Inspect the lower extremities for skin breakdown. B) Footwear needs to be accurately sized. D) Assistive devices may be needed to reduce the risk of falls.

The nurse is working with a patient who is newly diagnosed with MS. What basic information should the nurse provide to the patient? A) MS is a progressive demyelinating disease of the nervous system. B) MS usually occurs more frequently in men. C) MS typically has an acute onset. D) MS is sometimes caused by a bacterial infection.

A) MS is a progressive demyelinating disease of the nervous system.

The nurse is caring for a 77-year-old woman with MS. She states that she is very concerned about the progress of her disease and what the future holds. The nurse should know that elderly patients with MS are known to be particularly concerned about what variables? Select all that apply. A) Possible nursing home placement B) Pain associated with physical therapy C) Increasing disability D) Becoming a burden on the family E) Loss of appetite

A) Possible nursing home placement C) Increasing disability D) Becoming a burden on the family

A patient with diabetes presents to the clinic and is diagnosed with a mononeuropathy. This patient's nursing care should involve which of the following? A) Protection of the affected limb from injury B) Passive and active ROM exercises for the affected limb C) Education about improvements to glycemic control D) Interventions to prevent contractures

A) Protection of the affected limb from injury

The nurse in the neurological unit reviews the plan of care for a patient that presents with a fever after a craniotomy due to an abscess. It is MOST important for the nurse to instruct the nursing assistant to do which of the following? A. "Place a cooling blanket on the patient." B. "Check patient every hour and notify me immediately if a patient presents with a headache." C. "Remember to keep the head of the bed flat." D. "Keep a padded tongue blade promptly available in case of seizures."

A. "Place a cooling blanket on the patient."

A 38 year old male patient is flown to the neurointensive care unit after sustaining a traumatic head injury in a motor vehicle crash. He is intubated on arrival and has an initial GCS of 5 prior to intubation. After imaging is completed, a diffuse axonal injury is identified. Which of the following is the MOST important for the nurse to relay to the provider? A. A pupil which is large (~8mm) and nonreactive to light B. A CN I palsy on cranial nerve examination C. No enteral access is available D. No response to verbal stimulus observed

A. A pupil which is large (~8mm) and nonreactive to light

The physician has ordered a somatosensory evoked responses (SERs) test for a patient for whom the nurse is caring. The nurse is justified in suspecting that this patient may have a history of what type of neurologic disorder? A. Demyelinating disease B. Diabetic neuropathy C. Hypothalamic disorder D. Brainstem deficit

A. Demyelinating disease

A nurse is caring for a patient with an external ventricular drain (EVD). The patient's vital signs are 100/56 (71), 93, 20, and 38.2 C. The patient's ICP is measured as 15 mmHg. Which scheduled medication would the nurse hold and clarify with the provider? A. Metoprolol 25 mg B. Polyethylene glycol (Miralax) 1 tbsp C. Acetaminophen 650 mg D. Levetiracetam (Keppra) 1,000 mg

A. Metoprolol 25 mg

A patient has developed diabetes insipidus after having increased ICP following head trauma. When developing a teaching plan for this patient the nurse should include information about which hormone, commonly lacking in patients with diabetes insipidus? A) Antidiuretic hormone (ADH) B) Thyroid-stimulating hormone (TSH) C) Follicle-stimulating hormone (FSH) D) Luteinizing hormone (LH)

ANS: A ADH is the hormone lacking in diabetes insipidus. The patient's TSH, FSH, and LH levels won't be affected.

Your patient is scheduled for intracranial surgery in the morning. You know that it is important that the patient has adequate preparation for surgery to reduce what? A) Postoperative complications B) Length of time under anesthesia C) Establishing expectations that are too high D) Length of time in the hospital

ANS: A Adequate preparation for surgery, with attention to the patient's physical and emotional status, can reduce the risk of anxiety, fear, and postoperative complications. Adequate preparation for surgery does not reduce the length of time under anesthesia or in the hospital and it does not establish expectations that are too high.

You have a patient with an altered level of consciousness. What would be your first action when assessing this patient? A) Assessing the verbal response B) Assessing if the patient follows commands C) Assessing whether the patient will open their eyes D) Assessing response to pain

ANS: A Assessment of the patient with an altered LOC often starts with assessing the verbal response through determining the patient's orientation to time, person, and place. Therefore options B, C, and D are incorrect.

When the nurse observes that the post-craniotomy patient is unresponsive to and unaware of environmental stimuli, the nurse uses which of the following terms to describe the patient in his documentation? A) Unresponsive B) Comatose C) Demonstrating akinetic mutism D) In a persistent vegetative state

ANS: A Coma is a clinical state of unarousable unresponsiveness in which there are no purposeful responses to internal or external stimuli, although nonpurposeful responses to painful stimuli and brainstem reflexes may be present. Persistent vegetative state is a condition in which the patient is described as wakeful but devoid of conscious content, without cognitive or affective mental function. In unresponsiveness, the patient is unresponsive to and unaware of environmental stimuli. Akinetic mutism is a state of unresponsiveness to the environment in which the patient makes no movement or sound but sometimes opens the eyes.

The nursing instructor is discussing increased intracranial pressure (ICP) with the senior nursing students. What would the instructor be correct in telling the students is an early clinical manifestation of ICP? A) Disorientation and restlessness B) Decreased pulse and respirations C) Projectile vomiting D) Loss of corneal reflex

ANS: A Early indicators of ICP include disorientation and restlessness. Later signs include decreased pulse and respirations, projectile vomiting, and loss of brainstem reflexes such as the corneal reflex.

The nurse is admitting a patient to the unit who is scheduled for removal of an intracranial mass. What diagnostic procedures might be included in this patient's admission orders? (Mark all that apply.) A) Transcranial Doppler flow study B) Cerebral angiography C) MRI D) Cranial radiography E) EMG

ANS: A, B, C Preoperative diagnostic procedures may include a CT scan to demonstrate the lesion and show the degree of surrounding brain edema, the ventricular size, and the displacement. An MRI scan provides information similar to that of a CT scan with improved tissue contrast, resolution, and anatomic definition. Cerebral angiography may be used to study a tumor's blood supply or obtain information about vascular lesions. Transcranial Doppler flow studies are used to evaluate the blood flow within intracranial blood vessels. Regular x-rays of the skull would not be diagnostic for an intracranial mass. An EMG would not be ordered prior to intracranial surgery to remove a mass.

What diagnostic test is contraindicated in a patient exhibiting clinical manifestations of increased intracranial pressure? A) CT scan B) Lumbar puncture C) MRI D) Venous Doppler studies

ANS: B A lumbar puncture in a client with increased intracranial pressure (ICP) may cause the brain to herniate from the withdrawal of fluid and change in pressure during the lumbar puncture. Herniation of the brain is a dire and frequently fatal event. A CT scan, MRI, and venous Doppler are considered noninvasive procedures and would not affect the intracranial pressure itself.

While completing a health history on a newly diagnosed patient with generalized seizure disorder the nurse would assess for what characteristic associated with the post-ictal state? A) Epileptic cry B) Confusion C) Urinary incontinence D) Body rigidity

ANS: B In the post-ictal state (after the seizure), the patient is often confused and hard to arouse and may sleep for hours. The epileptic cry occurs from the simultaneous contractions of the diaphragm and chest muscles which occur during the seizure. Urinary incontinence and intense rigidity of the entire body are followed by alternating muscle relaxation and contraction (generalized tonic-clonic contraction) during the seizure.

A patient exhibiting an altered level of consciousness (LOC) due to blunt-force trauma to the head is admitted to the emergency department. The physician determines the patient's injury is causing increased intracranial pressure (ICP). The priority nursing evaluations, when assessing level of consciousness in this patient, would be based on what? A) Monro-Kellie hypothesis B) Glasgow Coma Scale C) Cranial nerve function D) Mental status exam

ANS: B Level of consciousness (LOC), a sensitive indicator of neurologic function, is assessed based on the criteria in the Glasgow Coma Scale: eye opening, verbal response, and motor response. Refer to Chart 63-4. The Monro-Kellie hypothesis states that because of the limited space for expansion within the skull, an increase in any one of the components (blood, brain tissue, cerebrospinal fluid) causes a change in the volume of the others. Cranial nerve function and the mental status exam would be part of the neurologic examination for this patient, but would not be the priority in evaluating level of consciousness.

When caring for an unconscious patient what nursing intervention takes highest priority? A) Inserting an indwelling urinary catheter B) Maintaining a patent airway C) Putting a nasogastric (NG) tube in place D) Administering an enema daily

ANS: B Maintaining a patent airway always takes top priority. An indwelling urinary catheter and NG tube can be inserted after airway patency has been established. Enemas should be avoided because of the danger of increasing intracranial pressure.

The nurse is caring for a patient on the neurologic unit who is in status epilepticus. What medications does the nurse know may be given to halt the seizure immediately? A) Intravenous phenobarbitol (Luminal) B) Intravenous diazepam (Valium) C) Oral lorazepam (Ativan) D) Oral phenytoin (Dilantin)

ANS: B Status epilepticus (acute prolonged seizure activity) is a series of generalized seizures that occur without full recovery of consciousness between attacks. Medical management of status epilepticus includes intravenous diazepam (Valium) and intravenous lorazepam (Ativan) given slowly in an attempt to halt seizures immediately. Other medications (phenytoin, phenobarbitol) are given later to maintain a seizure-free state.

In what position should the nurse place the patient following a craniotomy with a supratentorial approach? A) Position patient flat B) Maintain HOB elevated at 30 to 45 degrees C) Position patient in prone position D) Maintain bed in Trendelenburg position

ANS: B The patient undergoing a craniotomy with a supratentorial (above the tentorium) approach should be placed with the HOB elevated 30 to 45 degrees, with the neck in neutral alignment.

The nurse is caring for a postop craniotomy patient. When writing the plan of care, the patient has a diagnosis of Deficient fluid volume related to fluid restriction related to osmotic diuretic use. What would be an appropriate intervention for this diagnosis? A) Change the patient's position as indicated B) Monitor serum electrolytes C) Maintain NPO status D) Monitor arterial blood gas values

ANS: B The postoperative fluid regimen depends on the type of neurosurgical procedure and is determined on an individual basis. The volume and composition of fluids are adjusted based on daily serum electrolyte values, along with fluid intake and output. Fluids may have to be restricted in patients with cerebral edema. Changing the patient's position, maintaining an NPO status, and monitoring ABG values do not relate to the nursing diagnosis of Risk for imbalanced fluid volume.

A priority in postoperative management of a patient who has had intracranial surgery is what? A) Reducing pain B) Reducing periorbital edema C) Monitoring ICP D) Preserving seizures

ANS: C Ongoing postoperative management is aimed at detecting and reducing cerebral edema, relieving pain and preventing seizures, and monitoring ICP and neurologic status.

A nurse is admitting a patient with a severe migraine headache. The patient has a history of myocardial infarction in the past year. What migraine medication would the nurse question for this patient? A) Rizatriptan (Maxalt) B) Naratriptan (Amerge) C) Sumatriptan succinate (Imitrex) D) Zolmitriptan (Zomig)

ANS: C Sumatriptan may cause chest pain and is contraindicated in patients with ischemic heart disease. Adverse effects of sumatriptan succinate include angina, chest pressure, and chest tightness. None of the triptan medications should be taken concurrently with medications containing ergotamine (vascular headache suppressant) due to the potential for a prolonged vasoactive reaction. Maxalt, Amerge, and Zomig are triptans used in routine clinical use for the treatment of migraine headaches.

You have admitted a patient to the Neurolog Intensive Care Unit with a brainstem herniation. The patient is now exhibiting an altered level of consciousness. The nurse has determined that the patient's mean arterial pressure (MAP) is 60 with an intracranial pressure (ICP) reading of 5 mm Hg. The nurse would be correct in determining the cerebral perfusion pressure (CPP) as which of the following values? A) Normal B) High C) Low D) Compensating

ANS: C The cerebral perfusion pressure (CPP) is 55 mm Hg, which is considered low. The normal CPP is 70 to 100 mm Hg. Patients with a CPP of less than 50 mm Hg experience irreversible neurologic damage. A lower than normal CPP indicates that the cardiac output is insufficient to maintain adequate cerebral perfusion.

During their pathophysiology class the nursing students study seizures. How might the instructor best describe the cause of a seizure? A) Uncontrolled normal electrical charges throughout the brain B) A dysrhythmia in the motor strip of the brain C) A dysrhythmia in the nerve cells in one section of the brain D) Abnormal, recurring, controlled electrical charges in the brain

ANS: C The underlying cause of a seizure is an electrical disturbance (dysrhythmia) in the nerve cells in one section of the brain; these cells emit abnormal, recurring, uncontrolled electrical discharges. Seizures are not caused by normal electrical charges throughout the brain or controlled electrical charges in the brain. Option B could be correct, but not all seizures arise in the motor strip of the brain.

A patient with a seizure disorder is presenting having a generalized seizure. An appropriate nursing intervention during the seizure would include what? A) Restrain the patient to prevent injury. B) Open the patient's jaws to insert an oral airway. C) Place patient in high Fowler's position. D) Loosen the patient's restrictive clothing.

ANS: D An appropriate nursing intervention would include loosening any restrictive clothing on the patient. No attempt should be made to restrain the patient during the seizure because muscular contractions are strong and restraint can produce injury. Do not attempt to pry open jaws that are clenched in a spasm to insert anything. Broken teeth and injury to the lips and tongue may result from such an action. If possible, place the patient on one side with head flexed forward, which allows the tongue to fall forward and facilitates drainage of saliva and mucus.

The nurse is caring for a patient with increased intracranial pressure (ICP). The patient has a nursing diagnosis of Ineffective cerebral tissue perfusion. What would be an expected outcome that the nurse would document for this diagnosis? A) Copes with sensory deprivation. B) Registers normal body temperature. C) Pays attention to grooming. D) Obeys commands with appropriate motor responses.

ANS: D An expected outcome of the diagnosis of Ineffective tissue perfusion in a patient with increased intracranial pressure (ICP) would include obeying commands with appropriate motor responses. Vitals signs and neurologic status are assessed every 15 minutes to every hour. Coping with sensory deprivation would relate to the nursing diagnosis of Disturbed sensory perception. The outcome of Registers normal body temperature relates to the diagnosis of Potential for ineffective thermoregulation. Body image disturbance would have a potential outcome of Pays attention to grooming.

You are discharging a patient home after supratentorial removal of a pituitary mass. What medication would you expect to have ordered prophylactically for this patient? A) Prednisone B) Dexamethasone C) Cafergot D) Phentoin

ANS: D Antiseizure medication (phenytoin, diazepam) is often prescribed prophylactically for patients who have undergone supratentorial craniotomy because of the high risk of seizures after these procedures. Prednisone and dexamethasone are steroids. Cafergot is used in the treatment of migraines.

A patient is considered terminal after being involved in a motor vehicle accident in which they received massive trauma to the head. As the patient's ICP increases and condition worsens, the family asks you what indications of approaching death will there be. What would be your best response? A) "There is a change in the pattern of their respirations." B) "Projectile vomiting and hemiplegia usually occur just before death." C) "Posturing may develop as pressure on the brainstem increases." D) "Loss of brainstem reflexes is a sign of approaching death."

ANS: D As ICP increases, the patient's condition worsens, as manifested by the following signs and symptoms: the LOC continues to deteriorate until the patient is comatose. The pulse rate and respiratory rate decrease or become erratic, and the blood pressure and temperature increase. The pulse pressure (the difference between the systolic and the diastolic pressures) widens. The pulse fluctuates rapidly, varying from bradycardia to tachycardia. Altered respiratory patterns develop, including Cheyne-Stokes breathing (rhythmic waxing and waning of rate and depth of respirations alternating with brief periods of apnea) and ataxic breathing (irregular breathing with a random sequence of deep and shallow breaths). Projectile vomiting may occur with increased pressure on the reflex center in the medulla. Hemiplegia or decorticate or decerebrate posturing may develop as pressure on the brainstem increases. Bilateral flaccidity occurs before death. Loss of brainstem reflexes, including pupillary, corneal, gag, and swallowing reflexes, is an ominous sign of approaching death.

What should the nurse suspect when hourly assessment of urine output on a postcraniotomy patient exhibits a urine output from a catheter of 1500 mL for 2 consecutive hours? A) Cushing's syndrome. B) Syndrome of inappropriate antidiuretic hormone C) Adrenal crisis. D) Diabetes insipidus.

ANS: D Diabetes insipidus is an abrupt onset of extreme polyuria that commonly occurs in patients after brain surgery. Cushing's syndrome is excessive glucocorticoid secretion resulting in sodium and water retention. SIADH is the result of increased secretion of ADH; the patient becomes volume-overloaded, urine output diminishes, and serum sodium concentration becomes dilute. Adrenal crisis is undersecretion of glucocorticoids resulting in profound hypoglycemia, hypovolemia, and hypotension.

During the examination of an unconscious patient, the nurse notices that the patient's pupils are fixed and dilated. What is the clinical significance of the nurse's finding? A) It suggests onset of metabolic problems. B) It indicates paralysis on the right side of the body. C) It indicates paralysis of cranial nerve X. D) It indicates an injury at the midbrain level.

ANS: D Pupils that are fixed and dilated indicate injury at the midbrain level.

How does the nurse help the patient and family gain control of their lives? A) By providing educational resources in the community B) By offering referrals to community social clubs C) By introducing the patient to other neurologically impaired people in the community D) By collaborating with other members of the health care team

ANS: D The nurse collaborates with other members of the health care team to provide essential care, offer a variety of solutions to problems, help the patient and family gain control of their lives, and explore the educational and supportive resources available in the community. The nurse does not provide educational resources in the community, provide introductions to others who are neurologically impaired, or refer patients to social clubs

The causes of acquired seizures include what? (Mark all that apply.) A) Cerebrovascular disease B) Metabolic and toxic conditions C) Hypernatremia D) Brain tumor E) Drug and alcohol addiction

ANS: D The specific causes of seizures are varied and can be categorized as idiopathic (genetic, developmental defects) and acquired. Causes of acquired seizures include cerebrovascular disease; hypoxemia of any cause, including vascular insufficiency; fever (childhood); head injury; hypertension; central nervous system infections; metabolic and toxic conditions (eg, renal failure, hyponatremia, hypocalcemia, hypoglycemia, pesticide exposure); brain tumor; drug and alcohol withdrawal; and allergies.

An adult patient has sought care for the treatment of headaches that have become increasingly severe and frequent over the past several months. Which of the following questions addresses potential etiological factors? Select all that apply? A) "Are you exposed to any toxins or chemicals at work?" B) "How would you describe your ability to cope with stress?" C) "What medications are you currently taking?" D) "When was the last time you were hospitalized?" E) "Does anyone else in your family struggle with headaches?"

Ans: A, B, C, E Feedback: Headaches are multifactorial, and may involve medications, exposure to toxins, family history, and stress. Hospitalization is an unlikely contributor to headaches.

A patient with a C5 spinal cord injury is tetraplegic. After being moved out of the ICU, the patient complains of a severe throbbing headache. What should the nurse do first? A) Check the patient's indwelling urinary catheter for kinks to ensure patency. B) Lower the HOB to improve perfusion. C) Administer analgesia. D) Reassure the patient that headaches are expected after spinal cord injuries.

Ans: A Feedback: A severe throbbing headache is a common symptom of autonomic dysreflexia, which occurs after injuries to the spinal cord above T6. The syndrome is usually brought on by sympathetic stimulation, such as bowel and bladder distention. Lowering the HOB can increase ICP. Before administering analgesia, the nurse should check the patient's catheter, record vital signs, and perform an abdominal assessment. A severe throbbing headache is a dangerous symptom in this patient and is not expected.

A patient who just suffered a suspected ischemic stroke is brought to the ED by ambulance. On what should the nurse's primary assessment focus? A) Cardiac and respiratory status B) Seizure activity C) Pain D) Fluid and electrolyte balance

Ans: A Feedback: Acute care begins with managing ABCs. Patients may have difficulty keeping an open and clear airway secondary to decreased LOC. Neurologic assessment with close monitoring for signs of increased neurologic deficit and seizure activity occurs next. Fluid and electrolyte balance must be controlled carefully with the goal of adequate hydration to promote perfusion and decrease further brain activity.

A patient who has experienced an ischemic stroke has been admitted to the medical unit. The patient's family in adamant that she remain on bed rest to hasten her recovery and to conserve energy. What principle of care should inform the nurse's response to the family? A) The patient should mobilize as soon as she is physically able. B) To prevent contractures and muscle atrophy, bed rest should not exceed 4 weeks. C) The patient should remain on bed rest until she expresses a desire to mobilize. D) Lack of mobility will greatly increase the patient's risk of stroke recurrence.

Ans: A Feedback: As soon as possible, the patient is assisted out of bed and an active rehabilitation program is started. Delaying mobility causes complications, but not necessarily stroke recurrence. Mobility should not be withheld until the patient initiates.

The school nurse has been called to the football field where player is immobile on the field after landing awkwardly on his head during a play. While awaiting an ambulance, what action should the nurse perform? A) Ensure that the player is not moved. B) Obtain the player's vital signs, if possible. C) Perform a rapid assessment of the player's range of motion. D) Assess the player's reflexes.

Ans: A Feedback: At the scene of the injury, the patient must be immobilized on a spinal (back) board, with the head and neck maintained in a neutral position, to prevent an incomplete injury from becoming complete. This is a priority over determining the patient's vital signs. It would be inappropriate to test ROM or reflexes.

The nurse is caring for a patient whose spinal cord injury has caused recent muscle spasticity. What medication should the nurse expect to be ordered to control this? A) Baclofen (Lioresal) B) Dexamethasone (Decadron) C) Mannitol (Osmitrol) D) Phenobarbital (Luminal)

Ans: A Feedback: Baclofen is classified as an antispasmodic agent in the treatment of muscles spasms related to spinal cord injury. Decadron is an anti-inflammatory medication used to decrease inflammation in both SCI and head injury. Mannitol is used to decrease cerebral edema in patients with head injury. Phenobarbital is an anticonvulsant that is used in the treatment of seizure activity.

A patient with a T2 injury is in spinal shock. The nurse will expect to observe what assessment finding? A) Absence of reflexes along with flaccid extremities B) Positive Babinski's reflex along with spastic extremities C) Hyperreflexia along with spastic extremities D) Spasticity of all four extremities

Ans: A Feedback: During the period immediately following a spinal cord injury, spinal shock occurs. In spinal shock, all reflexes are absent and the extremities are flaccid. When spinal shock subsides, the patient demonstrates a positive Babinski's reflex, hyperreflexia, and spasticity of all four extremities

When preparing to discharge a patient home, the nurse has met with the family and warned them that the patient may exhibit unexpected emotional responses. The nurse should teach the family that these responses are typically a result of what cause? A) Frustration around changes in function and communication B) Unmet physiologic needs C) Changes in brain activity during sleep and wakefulness D) Temporary changes in metabolism

Ans: A Feedback: Emotional problems associated with stroke are often related to the new challenges around ADLs and communication. These challenges are more likely than metabolic changes, unmet physiologic needs, or changes in brain activity, each of which should be ruled out.

The nurse is planning the care of a patient with a T1 spinal cord injury. The nurse has identified the diagnosis of "risk for impaired skin integrity." How can the nurse best address this risk? A) Change the patient's position frequently. B) Provide a high-protein diet. C) Provide light massage at least daily. D) Teach the patient deep breathing and coughing exercises.

Ans: A Feedback: Frequent position changes are among the best preventative measures against pressure ulcers. A high-protein diet can benefit wound healing, but does not necessarily prevent skin breakdown. Light massage and deep breathing do not protect or restore skin integrity.

After a major ischemic stroke, a possible complication is cerebral edema. Nursing care during the immediate recovery period from an ischemic stroke should include which of the following? A) Positioning to avoid hypoxia B) Maximizing PaCO2 C) Administering hypertonic IV solution D) Initiating early mobilization

Ans: A Feedback: Interventions during this period include measures to reduce ICP, such as administering an osmotic diuretic (e.g., mannitol), maintaining the partial pressure of carbon dioxide (PaCO2) within the range of 30 to 35 mm Hg, and positioning to avoid hypoxia. Hypertonic IV solutions are not used unless sodium depletion is evident. Mobilization would take place after the immediate threat of increased ICP has past.

A patient with a new diagnosis of amyotrophic lateral sclerosis (ALS) is overwhelmed by his diagnosis and the known complications of the disease. How can the patient best make known his wishes for care as his disease progresses? A) Prepare an advance directive. B) Designate a most responsible physician (MRP) early in the course of the disease. C) Collaborate with representatives from the Amyotrophic Lateral Sclerosis Association. D) Ensure that witnesses are present when he provides instruction.

Ans: A Feedback: Patients with ALS are encouraged to complete an advance directive or "living will" to preserve their autonomy in decision making. None of the other listed actions constitutes a legally binding statement of end-of-life care.

A patient who suffered an ischemic stroke now has disturbed sensory perception. What principle should guide the nurse's care of this patient? A) The patient should be approached on the side where visual perception is intact. B) Attention to the affected side should be minimized in order to decrease anxiety. C) The patient should avoid turning in the direction of the defective visual field to minimize shoulder subluxation. D) The patient should be approached on the opposite side of where the visual perception is intact to promote recovery.

Ans: A Feedback: Patients with decreased field of vision should first be approached on the side where visual perception is intact. All visual stimuli should be placed on this side. The patient can and should be taught to turn the head in the direction of the defective visual field to compensate for this loss. The nurse should constantly remind the patient of the other side of the body and should later stand at a position that encourages the patient to move or turn to visualize who and what is in the room.

The patient has been diagnosed with aphasia after suffering a stroke. What can the nurse do to best make the patient's atmosphere more conducive to communication? A) Provide a board of commonly used needs and phrases. B) Have the patient speak to loved ones on the phone daily. C) Help the patient complete his or her sentences. D) Speak in a loud and deliberate voice to the patient.

Ans: A Feedback: The inability to talk on the telephone or answer a question or exclusion from conversation causes anger, frustration, fear of the future, and hopelessness. A common pitfall is for the nurse or other health care team member to complete the thoughts or sentences of the patient. This should be avoided because it may cause the patient to feel more frustrated at not being allowed to speak and may deter efforts to practice putting thoughts together and completing a sentence. The patient may also benefit from a communication board, which has pictures of commonly requested needs and phrases. The board may be translated into several languages.

Following diagnostic testing, a patient has been admitted to the ICU and placed on cerebral aneurysm precautions. What nursing action should be included in patient's plan of care? A) Supervise the patient's activities of daily living closely. B) Initiate early ambulation to prevent complications of immobility. C) Provide a high-calorie, low-protein diet. D) Perform all of the patient's hygiene and feeding.

Ans: A Feedback: The patient is placed on immediate and absolute bed rest in a quiet, nonstressful environment, because activity, pain, and anxiety elevate BP, which increases the risk for bleeding. As such, independent ADLs and ambulation are contraindicated. There is no need for a high-calorie or low-protein diet.

The nurse is providing health education to a patient who has a C6 spinal cord injury. The patient asks why autonomic dysreflexia is considered an emergency. What would be the nurse's best answer? A) "The sudden increase in BP can raise the ICP or rupture a cerebral blood vessel." B) "The suddenness of the onset of the syndrome tells us the body is struggling to maintain its normal state." C) "Autonomic dysreflexia causes permanent damage to delicate nerve fibers that are healing." D) "The sudden, severe headache increases muscle tone and can cause further nerve damage."

Ans: A Feedback: The sudden increase in BP may cause a rupture of one or more cerebral blood vessels or lead to increased ICP. Autonomic dysreflexia does not directly cause nerve damage.

A patient is brought by ambulance to the ED after suffering what the family thinks is a stroke. The nurse caring for this patient is aware that an absolute contraindication for thrombolytic therapy is what? A) Evidence of hemorrhagic stroke B) Blood pressure of ³ 180/110 mm Hg C) Evidence of stroke evolution D) Previous thrombolytic therapy within the past 12 months

Ans: A Feedback: Thrombolytic therapy would exacerbate a hemorrhagic stroke with potentially fatal consequences. Stroke evolution, high BP, or previous thrombolytic therapy does not contraindicate its safe and effective use.

An 82-year-old man is admitted for observation after a fall. Due to his age, the nurse knows that the patient is at increased risk for what complication of his injury? A) Hematoma B) Skull fracture C) Embolus D) Stroke

Ans: A Feedback: Two major factors place older adults at increased risk for hematomas. First, the dura becomes more adherent to the skull with increasing age. Second, many older adults take aspirin and anticoagulants as part of routine management of chronic conditions. Because of these factors, the patient's risk of a hematoma is likely greater than that of stroke, embolism, or skull fracture.

When caring for a patient with increased ICP the nurse knows the importance of monitoring for possible secondary complications, including syndrome of inappropriate antidiuretic hormone (SIADH). What nursing interventions would the nurse most likely initiate if the patient developed SIADH? A) Fluid restriction B) Transfusion of platelets C) Transfusion of fresh frozen plasma (FFP) D) Electrolyte restriction

Ans: A Feedback: The nurse also assesses for complications of increased ICP, including diabetes insipidus, and SIADH. SIADH requires fluid restriction and monitoring of serum electrolyte levels. Transfusions are unnecessary.

A patient is recovering from intracranial surgery performed approximately 24 hours ago and is complaining of a headache that the patient rates at 8 on a 10-point pain scale. What nursing action is most appropriate? A) Administer morphine sulfate as ordered. B) Reposition the patient in a prone position. C) Apply a hot pack to the patient's scalp. D) Implement distraction techniques.

Ans: A Feedback: The patient usually has a headache after a craniotomy as a result of stretching and irritation of nerves in the scalp during surgery. Morphine sulfate may also be used in the management of postoperative pain in patients who have undergone a craniotomy. Prone positioning is contraindicated due to the consequent increase in ICP. Distraction would likely be inadequate to reduce pain and a hot pack may cause vasodilation and increased pain.

A patient with spinal cord injury is ready to be discharged home. A family member asks the nurse to review potential complications one more time. What are the potential complications that should be monitored for in this patient? Select all that apply. A) Orthostatic hypotension B) Autonomic dysreflexia C) DVT D) Salt-wasting syndrome E) Increased ICP

Ans: A, B, C Feedback: For a spinal cord-injured patient, based on the assessment data, potential complications that may develop include DVT, orthostatic hypotension, and autonomic dysreflexia. Salt-wasting syndrome or increased ICP are not typical complications following the immediate recovery period.

During a patient's recovery from stroke, the nurse should be aware of predictors of stroke outcome in order to help patients and families set realistic goals. What are the predictors of stroke outcome? Select all that apply. A) National Institutes of Health Stroke Scale (NIHSS) score B) Race C) LOC at time of admission D) Gender E) Age

Ans: A, C, E Feedback: It is helpful for clinicians to be knowledgeable about the relative importance of predictors of stroke outcome (age, NIHSS score, and LOC at time of admission) to provide stroke survivors and their families with realistic goals. Race and gender are not predictors of stroke outcome.

When caring for a patient who had a hemorrhagic stroke, close monitoring of vital signs and neurologic changes is imperative. What is the earliest sign of deterioration in a patient with a hemorrhagic stroke of which the nurse should be aware? A) Generalized pain B) Alteration in level of consciousness (LOC) C) Tonic-clonic seizures D) Shortness of breath

Ans: B Feedback: Alteration in LOC is the earliest sign of deterioration in a patient after a hemorrhagic stroke, such as mild drowsiness, slight slurring of speech, and sluggish papillary reaction. Sudden headache may occur, but generalized pain is less common. Seizures and shortness of breath are not identified as early signs of hemorrhagic stroke.

An elderly woman found with a head injury on the floor of her home is subsequently admitted to the neurologic ICU. What is the best rationale for the following physician orders: elevate the HOB; keep the head in neutral alignment with no neck flexion or head rotation; avoid sharp hip flexion? A) To decrease cerebral arterial pressure B) To avoid impeding venous outflow C) To prevent flexion contractures D) To prevent aspiration of stomach contents

Ans: B Feedback: Any activity or position that impedes venous outflow from the head may contribute to increased volume inside the skull and possibly increase ICP. Cerebral arterial pressure will be affected by the balance between oxygen and carbon dioxide. Flexion contractures are not a priority at this time. Stomach contents could still be aspirated in this position.

A nurse is caring for a critically ill patient with autonomic dysreflexia. What clinical manifestations would the nurse expect in this patient? A) Respiratory distress and projectile vomiting B) Bradycardia and hypertension C) Tachycardia and agitation D) Third-spacing and hyperthermia

Ans: B Feedback: Autonomic dysreflexia is characterized by a pounding headache, profuse sweating, nasal congestion, piloerection ("goose bumps"), bradycardia, and hypertension. It occurs in cord lesions above T6 after spinal shock has resolved; it does not result in vomiting, tachycardia

A patient has been admitted to the ICU after being recently diagnosed with an aneurysm and the patient's admission orders include specific aneurysm precautions. What nursing action will the nurse incorporate into the patient's plan of care? A) Elevate the head of the bed to 45 degrees. B) Maintain the patient on complete bed rest. C) Administer enemas when the patient is constipated. D) Avoid use of thigh-high elastic compression stockings.

Ans: B Feedback: Cerebral aneurysm precautions are implemented for the patient with a diagnosis of aneurysm to provide a nonstimulating environment, prevent increases in ICP, and prevent further bleeding. The patient is placed on immediate and absolute bed rest in a quiet, nonstressful environment because activity, pain, and anxiety elevate BP, which increases the risk for bleeding. Visitors, except for family, are restricted. The head of the bed is elevated 15 to 30 degrees to promote venous drainage and decrease ICP. Some neurologists, however, prefer that the patient remains flat to increase cerebral perfusion. No enemas are permitted, but stool softeners and mild laxatives are prescribed. Thigh-high elastic compression stockings or sequential compression boots may be ordered to decrease the patient's risk for deep vein thrombosis (DVT).

A patient has recently begun mobilizing during the recovery from an ischemic stroke. To protect the patient's safety during mobilization, the nurse should perform what action? A) Support the patient's full body weight with a waist belt during ambulation. B) Have a colleague follow the patient closely with a wheelchair. C) Avoid mobilizing the patient in the early morning or late evening. D) Ensure that the patient's family members do not participate in mobilization.

Ans: B Feedback: During mobilization, a chair or wheelchair should be readily available in case the patient suddenly becomes fatigued or feels dizzy. The family should be encouraged to participate, as appropriate, and the nurse should not have to support the patient's full body weight. Morning and evening activity are not necessarily problematic.

A nurse is caring for a patient diagnosed with a hemorrhagic stroke. When creating this patient's plan of care, what goal should be prioritized? A) Prevent complications of immobility. B) Maintain and improve cerebral tissue perfusion. C) Relieve anxiety and pain. D) Relieve sensory deprivation.

Ans: B Feedback: Each of the listed goals is appropriate in the care of a patient recovering from a stroke. However, promoting cerebral perfusion is a priority physiologic need, on which the patient's survival depends.

When caring for a patient who has had a stroke, a priority is reduction of ICP. What patient position is most consistent with this goal? A) Head turned slightly to the right side B) Elevation of the head of the bed C) Position changes every 15 minutes while awake D) Extension of the neck

Ans: B Feedback: Elevation of the head of the bed promotes venous drainage and lowers ICP; the nurse should avoid flexing or extending the neck or turning the head side to side. The head should be in a neutral midline position. Excessively frequent position changes are unnecessary.

A patient is brought to the trauma center by ambulance after sustaining a high cervical spinal cord injury 1½ hours ago. Endotracheal intubation has been deemed necessary and the nurse is preparing to assist. What nursing diagnosis should the nurse associate with this procedure? A) Risk for impaired skin integrity B) Risk for injury C) Risk for autonomic dysreflexia D) Risk for suffocation

Ans: B Feedback: If endotracheal intubation is necessary, extreme care is taken to avoid flexing or extending the patient's neck, which can result in extension of a cervical injury. Intubation does not directly cause autonomic dysreflexia and the threat to skin integrity is a not a primary concern. Intubation does not carry the potential to cause suffocation.

Following a spinal cord injury a patient is placed in halo traction. While performing pin site care, the nurse notes that one of the traction pins has become detached. The nurse would be correct in implementing what priority nursing action? A) Complete the pin site care to decrease risk of infection. B) Notify the neurosurgeon of the occurrence. C) Stabilize the head in a lateral position. D) Reattach the pin to prevent further head trauma.

Ans: B Feedback: If one of the pins became detached, the head is stabilized in neutral position by one person while another notifies the neurosurgeon. Reattaching the pin as a nursing intervention would not be done due to risk of increased injury. Pin site care would not be a priority in this instance. Prevention of neurologic injury is the priority.

A rehabilitation nurse caring for a patient who has had a stroke is approached by the patient's family and asked why the patient has to do so much for herself when she is obviously struggling. What would be the nurse's best answer? A) "We are trying to help her be as useful as she possibly can." B) "The focus on care in a rehabilitation facility is to help the patient to resume as much self-care as possible." C) "We aren't here to care for her the way the hospital staff did; we are here to help her get better so she can go home." D) "Rehabilitation means helping patients do exactly what they did before their stroke."

Ans: B Feedback: In both acute care and rehabilitation facilities, the focus is on teaching the patient to resume as much self-care as possible. The goal of rehabilitation is not to be "useful," nor is it to return patients to their prestroke level of functioning, which may be unrealistic.

A patient with spinal cord injury has a nursing diagnosis of altered mobility and the nurse recognizes the increased the risk of deep vein thrombosis (DVT). Which of the following would be included as an appropriate nursing intervention to prevent a DVT from occurring? A) Placing the patient on a fluid restriction as ordered B) Applying thigh-high elastic stockings C) Administering an antifibrinolyic agent D) Assisting the patient with passive range of motion (PROM) exercises

Ans: B Feedback: It is important to promote venous return to the heart and prevent venous stasis in a patient with altered mobility. Applying elastic stockings will aid in the prevention of a DVT. The patient should not be placed on fluid restriction because a dehydrated state will increase the risk of clotting throughout the body. Antifibrinolytic agents cause the blood to clot, which is absolutely contraindicated in this situation. PROM exercises are not an effective protection against the development of DVT.

A patient exhibiting an altered level of consciousness (LOC) due to blunt-force trauma to the head is admitted to the ED. The physician determines the patient's injury is causing increased intracranial pressure (ICP). The nurse should gauge the patient's LOC on the results of what diagnostic tool? A) Monro-Kellie hypothesis B) Glasgow Coma Scale C) Cranial nerve function D) Mental status examination

Ans: B Feedback: LOC, a sensitive indicator of neurologic function, is assessed based on the criteria in the Glasgow Coma Scale: eye opening, verbal response, and motor response. The Monro-Kellie hypothesis states that because of the limited space for expansion within the skull, an increase in any one of the components (blood, brain tissue, cerebrospinal fluid) causes a change in the volume of the others. Cranial nerve function and the mental status examination would be part of the neurologic examination for this patient, but would not be the priority in evaluating LOC.

The staff educator is precepting a nurse new to the critical care unit when a patient with a T2 spinal cord injury is admitted. The patient is soon exhibiting manifestations of neurogenic shock. In addition to monitoring the patient closely, what would be the nurse's most appropriate action? A) Prepare to transfuse packed red blood cells. B) Prepare for interventions to increase the patient's BP. C) Place the patient in the Trendelenberg position. D) Prepare an ice bath to lower core body temperature.

Ans: B Feedback: Manifestations of neurogenic shock include decreased BP and heart rate. Cardiac markers would be expected to rise in cardiogenic shock. Transfusion, repositioning, and ice baths are not indicated interventions.

A neurologic flow chart is often used to document the care of a patient with a traumatic brain injury. At what point in the patient's care should the nurse begin to use a neurologic flow chart? A) When the patient's condition begins to deteriorate B) As soon as the initial assessment is made C) At the beginning of each shift D) When there is a clinically significant change in the patient's condition

Ans: B Feedback: Neurologic parameters are assessed initially and as frequently as the patient's condition requires. As soon as the initial assessment is made, the use of a neurologic flowchart is started and maintained. A new chart is not begun at the start of every shift.

A patient who has sustained a nondepressed skull fracture is admitted to the acute medical unit. Nursing care should include which of the following? A) Preparation for emergency craniotomy B) Watchful waiting and close monitoring C) Administration of inotropic drugs D) Fluid resuscitation

Ans: B Feedback: Nondepressed skull fractures generally do not require surgical treatment; however, close observation of the patient is essential. A craniotomy would not likely be needed if the fracture is nondepressed. Even if treatment is warranted, it is unlikely to include inotropes or fluid resuscitation.

A family member brings the patient to the clinic for a follow-up visit after a stroke. The family member asks the nurse what he can do to decrease his chance of having another stroke. What would be the nurse's best answer? A) "Have your heart checked regularly." B) "Stop smoking as soon as possible." C) "Get medication to bring down your sodium levels." D) "Eat a nutritious diet."

Ans: B Feedback: Smoking is a modifiable and highly significant risk factor for stroke. The significance of smoking, and the potential benefits of quitting, exceed the roles of sodium, diet, and regular medical assessments.

A nurse is reviewing the trend of a patient's scores on the Glasgow Coma Scale (GCS). This allows the nurse to gauge what aspect of the patient's status? A) Reflex activity B) Level of consciousness C) Cognitive ability D) Sensory involvement

Ans: B Feedback: The Glasgow Coma Scale (GCS) examines three responses related to LOC: eye opening, best verbal response, and best motor response.

The nurse is performing stroke risk screenings at a hospital open house. The nurse has identified four patients who might be at risk for a stroke. Which patient is likely at the highest risk for a hemorrhagic stroke? A) White female, age 60, with history of excessive alcohol intake B) White male, age 60, with history of uncontrolled hypertension C) Black male, age 60, with history of diabetes D) Black male, age 50, with history of smoking

Ans: B Feedback: Uncontrolled hypertension is the primary cause of a hemorrhagic stroke. Control of hypertension, especially in individuals over 55 years of age, clearly reduces the risk for hemorrhagic stroke. Additional risk factors are increased age, male gender, and excessive alcohol intake. Another high-risk group includes African Americans, where the incidence of first stroke is almost twice that as in Caucasians.

The nurse is caring for a patient who is in status epilepticus. What medication does the nurse know may be given to halt the seizure immediately? A) Intravenous phenobarbital (Luminal) B) Intravenous diazepam (Valium) C Oral lorazepam (Ativan) D) Oral phenytoin (Dilantin)

Ans: B Feedback: Medical management of status epilepticus includes IV diazepam (Valium) and IV lorazepam (Ativan) given slowly in an attempt to halt seizures immediately. Other medications (phenytoin, phenobarbital) are given later to maintain a seizure-free state. Oral medications are not given during status epilepticus.

A school nurse is called to the playground where a 6-year-old girl has been found unresponsive and "staring into space," according to the playground supervisor. How would the nurse document the girl's activity in her chart at school? A) Generalized seizure B) Absence seizure C) Focal seizure D) Unclassified seizure

Ans: B Feedback: Staring episodes characterize an absence seizure, whereas focal seizures, generalized seizures, and unclassified seizures involve uncontrolled motor activity.

The nurse is caring for a patient who is rapidly progressing toward brain death. The nurse should be aware of what cardinal signs of brain death? Select all that apply. A) Absence of pain response B) Apnea C) Coma D) Absence of brain stem reflexes E) Absence of deep tendon reflexes

Ans: B, C, D Feedback: The three cardinal signs of brain death upon clinical examination are coma, the absence of brain stem reflexes, and apnea. Absences of pain response and deep tendon reflexes are not necessarily indicative of brain death.

As a member of the stroke team, the nurse knows that thrombolytic therapy carries the potential for benefit and for harm. The nurse should be cognizant of what contraindications for thrombolytic therapy? Select all that apply. A) INR above 1.0 B) Recent intracranial pathology C) Sudden symptom onset D) Current anticoagulation therapy E) Symptom onset greater than 3 hours prior to admission

Ans: B, D, E Feedback: Some of the absolute contraindications for thrombolytic therapy include symptom onset greater than 3 hours before admission, a patient who is anticoagulated (with an INR above 1.7), or a patient who has recently had any type of intracranial pathology (e.g., previous stroke, head injury, trauma)

A patient is admitted to the neurologic ICU with a C4 spinal cord injury. When writing the plan of care for this patient, which of the following nursing diagnoses would the nurse prioritize in the immediate care of this patient? A) Risk for impaired skin integrity related to immobility and sensory loss B) Impaired physical mobility related to loss of motor function C) Ineffective breathing patterns related to weakness of the intercostal muscles D) Urinary retention related to inability to void spontaneously

Ans: C Feedback: A nursing diagnosis related to breathing pattern would be the priority for this patient. A C4 spinal cord injury will require ventilatory support, due to the diaphragm and intercostals being affected. The other nursing diagnoses would be used in the care plan, but not designated as a higher priority than ineffective breathing patterns.

A patient has had an ischemic stroke and has been admitted to the medical unit. What action should the nurse perform to best prevent joint deformities? A) Place the patient in the prone position for 30 minutes/day. B) Assist the patient in acutely flexing the thigh to promote movement. C) Place a pillow in the axilla when there is limited external rotation. D) Place patient's hand in pronation.

Ans: C Feedback: A pillow in the axilla prevents adduction of the affected shoulder and keeps the arm away from the chest. The prone position with a pillow under the pelvis, not flat, promotes hyperextension of the hip joints, essential for normal gait. To promote venous return and prevent edema, the upper thigh should not be flexed acutely. The hand is placed in slight supination, not pronation, which is its most functional position.

A patient with Parkinson's disease is undergoing a swallowing assessment because she has recently developed adventitious lung sounds. The patient's nutritional needs should be met by what method? A) Total parenteral nutrition (TPN) B) Provision of a low-residue diet C) Semisolid food with thick liquids D) Minced foods and a fluid restriction

Ans: C Feedback: A semisolid diet with thick liquids is easier for a patient with swallowing difficulties to consume than is a solid diet. Low-residue foods and fluid restriction are unnecessary and counterproductive to the patient's nutritional status. The patient's status does not warrant TPN.

A community health nurse is giving an educational presentation about stroke and heart disease at the local senior citizens center. What nonmodifiable risk factor for stroke should the nurse cite? A) Female gender B) Asian American race C) Advanced age D) Smoking

Ans: C Feedback: Advanced age, male gender, and race are well-known nonmodifiable risk factors for stroke. High-risk groups include people older than 55 years of age; the incidence of stroke more than doubles in each successive decade. Men have a higher rate of stroke than that of women. Another high-risk group is African Americans; the incidence of first stroke in African Americans is almost twice that as in Caucasian Americans; Asian American race is not a risk factor. Smoking is a modifiable risk.

A patient is brought to the ED by her family after falling off the roof. A family member tells the nurse that when the patient fell she was "knocked out," but came to and "seemed okay." Now she is complaining of a severe headache and not feeling well. The care team suspects an epidural hematoma, prompting the nurse to prepare for which priority intervention? A) Insertion of an intracranial monitoring device B) Treatment with antihypertensives C) Emergency craniotomy D) Administration of anticoagulant therapy

Ans: C Feedback: An epidural hematoma is considered an extreme emergency. Marked neurologic deficit or respiratory arrest can occur within minutes. Treatment consists of making an opening through the skull to decrease ICP emergently, remove the clot, and control the bleeding. Antihypertensive medications would not be a priority. Anticoagulant therapy should not be ordered for a patient who has a cranial bleed. This could further increase bleeding activity. Insertion of an intracranial monitoring device may be done during the surgery, but is not priority for this patient.

A patient with a new diagnosis of ischemic stroke is deemed to be a candidate for treatment with tissue plasminogen activator (t-PA) and has been admitted to the ICU. In addition to closely monitoring the patient's cardiac and neurologic status, the nurse monitors the patient for signs of what complication? A) Acute pain B) Septicemia C) Bleeding D) Seizures

Ans: C Feedback: Bleeding is the most common side effect of t-PA administration, and the patient is closely monitored for any bleeding. Septicemia, pain, and seizures are much less likely to result from thrombolytic therapy.

The nurse is discharging home a patient who suffered a stroke. He has a flaccid right arm and leg and is experiencing problems with urinary incontinence. The nurse makes a referral to a home health nurse because of an awareness of what common patient response to a change in body image? A) Denial B) Fear C) Depression D) Disassociation

Ans: C Feedback: Depression is a common and serious problem in the patient who has had a stroke. It can result from a profound disruption in his or her life and changes in total function, leaving the patient with a loss of independence. The nurse needs to encourage the patient to verbalize feelings to assess the effect of the stroke on self-esteem. Denial, fear, and disassociation are not the most common patient response to a change in body image, although each can occur in some patients.

A patient diagnosed with a hemorrhagic stroke has been admitted to the neurologic ICU. The nurse knows that teaching for the patient and family needs to begin as soon as the patient is settled on the unit and will continue until the patient is discharged. What will family education need to include? A) How to differentiate between hemorrhagic and ischemic stroke B) Risk factors for ischemic stroke C) How to correctly modify the home environment D) Techniques for adjusting the patient's medication dosages at home

Ans: C Feedback: For a patient with a hemorrhagic stroke, teaching addresses the use of assistive devices or modification of the home environment to help the patient live with the disability. This is more important to the patient's needs than knowing about risk factors for ischemic stroke. It is not necessary for the family to differentiate between different types of strokes. Medication regimens should never be altered without consultation.

A patient has experienced a seizure in which she became rigid and then experienced alternating muscle relaxation and contraction. What type of seizure does the nurse recognize? A) Unclassified seizure B) Absence seizure C) Generalized seizure D) Focal seizure

Ans: C Feedback: Generalized seizures often involve both hemispheres of the brain, causing both sides of the body to react. Intense rigidity of the entire body may occur, followed by alternating muscle relaxation and contraction (generalized tonic-clonic contraction). This pattern of rigidity does not occur in patients who experience unclassified, absence, or focal seizures.

The nurse is caring for a patient diagnosed with an ischemic stroke and knows that effective positioning of the patient is important. Which of the following should be integrated into the patient's plan of care? A) The patient's hip joint should be maintained in a flexed position. B) The patient should be in a supine position unless ambulating. C) The patient should be placed in a prone position for 15 to 30 minutes several times a day. D) The patient should be placed in a Trendelenberg position two to three times daily to promote cerebral perfusion.

Ans: C Feedback: If possible, the patient is placed in a prone position for 15 to 30 minutes several times a day. A small pillow or a support is placed under the pelvis, extending from the level of the umbilicus to the upper third of the thigh. This helps to promote hyperextension of the hip joints, which is essential for normal gait, and helps prevent knee and hip flexion contractures. The hip joints should not be maintained in flexion and the Trendelenberg position is not indicated.

Paramedics have brought an intubated patient to the RD following a head injury due to acceleration-deceleration motor vehicle accident. Increased ICP is suspected. Appropriate nursing interventions would include which of the following? A) Keep the head of the bed (HOB) flat at all times. B) Teach the patient to perform the Valsalva maneuver. C) Administer benzodiazepines on a PRN basis. D) Perform endotracheal suctioning every hour.

Ans: C Feedback: If the patient with a brain injury is very agitated, benzodiazepines are the most commonly used sedatives and do not affect cerebral blood flow or ICP. The HOB should be elevated 30 degrees. Suctioning should be done a limited basis, due to increasing the pressure in the cranium. The Valsalva maneuver is to be avoided. This also causes increased ICP.

A patient with Parkinson's disease is experiencing episodes of constipation that are becoming increasingly frequent and severe. The patient states that he has been achieving relief for the past few weeks by using OTC laxatives. How should the nurse respond? A) "It's important to drink plenty of fluids while you're taking laxatives." B) "Make sure that you supplement your laxatives with a nutritious diet." C) "Let's explore other options, because laxatives can have side effects and create dependency." D) "You should ideally be using herbal remedies rather than medications to promote bowel function."

Ans: C Feedback: Laxatives should be avoided in patients with Parkinson's disease due to the risk of adverse effects and dependence. Herbal bowel remedies are not necessarily less risky.

The nurse recognizes that a patient with a SCI is at risk for muscle spasticity. How can the nurse best prevent this complication of an SCI? A) Position the patient in a high Fowler's position when in bed. B) Support the knees with a pillow when the patient is in bed. C) Perform passive ROM exercises as ordered. D) Administer NSAIDs as ordered.

Ans: C Feedback: Passive ROM exercises can prevent muscle spasticity following SCI. NSAIDs are not used for this purpose. Pillows and sitting upright do not directly address the patient's risk of muscle spasticity.

A patient is admitted to the neurologic ICU with a spinal cord injury. In writing the patient's care plan, the nurse specifies that contractures can best be prevented by what action? A) Repositioning the patient every 2 hours B) Initiating range-of-motion exercises (ROM) as soon as the patient initiates C) Initiating (ROM) exercises as soon as possible after the injury D) Performing ROM exercises once a day

Ans: C Feedback: Passive ROM exercises should be implemented as soon as possible after injury. It would be inappropriate to wait for the patient to first initiate exercises. Toes, metatarsals, ankles, knees, and hips should be put through a full ROM at least four, and ideally five, times daily. Repositioning alone will not prevent contractures.

The nurse is reviewing the medication administration record of a female patient who possesses numerous risk factors for stroke. Which of the woman's medications carries the greatest potential for reducing her risk of stroke? A) Naproxen 250 PO b.i.d. B) Calcium carbonate 1,000 mg PO b.i.d. C) Aspirin 81 mg PO o.d. D) Lorazepam 1 mg SL b.i.d. PRN

Ans: C Feedback: Research findings suggest that low-dose aspirin may lower the risk of stroke in women who are at risk. Naproxen, lorazepam, and calcium supplements do not have this effect.

Splints have been ordered for a patient who is at risk of developing footdrop following a spinal cord injury. The nurse caring for this patient knows that the splints are removed and reapplied when? A) At the patient's request B) Each morning and evening C) Every 2 hours D) One hour prior to mobility exercises

Ans: C Feedback: The feet are prone to footdrop; therefore, various types of splints are used to prevent footdrop. When used, the splints are removed and reapplied every 2 hours.

A patient diagnosed with transient ischemic attacks (TIAs) is scheduled for a carotid endarterectomy. The nurse explains that this procedure will be done for what purpose? A) To decrease cerebral edema B) To prevent seizure activity that is common following a TIA C) To remove atherosclerotic plaques blocking cerebral flow D) To determine the cause of the TIA

Ans: C Feedback: The main surgical procedure for select patients with TIAs is carotid endarterectomy, the removal of an atherosclerotic plaque or thrombus from the carotid artery to prevent stroke in patients with occlusive disease of the extracranial arteries. An endarterectomy does not decrease cerebral edema, prevent seizure activity, or determine the cause of a TIA.

The nurse planning the care of a patient with head injuries is addressing the patient's nursing diagnosis of "sleep deprivation." What action should the nurse implement? A) Administer a benzodiazepine at bedtime each night. B) Do not disturb the patient between 2200 and 0600. C) Cluster overnight nursing activities to minimize disturbances. D) Ensure that the patient does not sleep during the day.

Ans: C Feedback: To allow the patient longer times of uninterrupted sleep and rest, the nurse can group nursing care activities so that the patient is disturbed less frequently. However, it is impractical and unsafe to provide no care for an 8-hour period. The use of benzodiazepines should be avoided.

The nurse caring for a patient with a spinal cord injury notes that the patient is exhibiting early signs and symptoms of disuse syndrome. Which of the following is the most appropriate nursing action? A) Limit the amount of assistance provided with ADLs. B) Collaborate with the physical therapist and immobilize the patient's extremities temporarily. C) Increase the frequency of ROM exercises. D) Educate the patient about the importance of frequent position changes.

Ans: C Feedback: To prevent disuse syndrome, ROM exercises must be provided at least four times a day, and care is taken to stretch the Achilles tendon with exercises. The patient is repositioned frequently and is maintained in proper body alignment whether in bed or in a wheelchair. The patient must be repositioned by caregivers, not just taught about repositioning. It is inappropriate to limit assistance for the sole purpose of preventing disuse syndrome.

A patient with a spinal cord injury has experienced several hypotensive episodes. How can the nurse best address the patient's risk for orthostatic hypotension? A) Administer an IV bolus of normal saline prior to repositioning. B) Maintain bed rest until normal BP regulation returns. C) Monitor the patient's BP before and during position changes. D) Allow the patient to initiate repositioning.

Ans: C Feedback: To prevent hypotensive episodes, close monitoring of vital signs before and during position changes is essential. Prolonged bed rest carries numerous risks and it is not possible to provide a bolus before each position change. Following the patient's lead may or may not help regulate BP.

A patient diagnosed with a cerebral aneurysm reports a severe headache to the nurse. What action is a priority for the nurse? A) Sit with the patient for a few minutes. B) Administer an analgesic. C) Inform the nurse-manager. D) Call the physician immediately.

Ans: D Feedback: A headache may be an indication that the aneurysm is leaking. The nurse should notify the physician immediately. The physician will decide whether administration of an analgesic is indicated. Informing the nurse-manager is not necessary. Sitting with the patient is appropriate, once the physician has been notified of the change in the patient's condition.

The ED is notified that a 6-year-old is in transit with a suspected brain injury after being struck by a car. The child is unresponsive at this time, but vital signs are within acceptable limits. What will be the primary goal of initial therapy? A) Promoting adequate circulation B) Treating the child's increased ICP C) Assessing secondary brain injury D) Preserving brain homeostasis

Ans: D Feedback: All therapy is directed toward preserving brain homeostasis and preventing secondary brain injury, which is injury to the brain that occurs after the original traumatic event. The scenario does not indicate the child has increased ICP or a secondary brain injury at this point. Promoting circulation is likely secondary to the broader goal of preserving brain homeostasis.

A nurse in the ICU is providing care for a patient who has been admitted with a hemorrhagic stroke. The nurse is performing frequent neurologic assessments and observes that the patient is becoming progressively more drowsy over the course of the day. What is the nurse's best response to this assessment finding? A) Report this finding to the physician as an indication of decreased metabolism. B) Provide more stimulation to the patient and monitor the patient closely. C) Recognize this as the expected clinical course of a hemorrhagic stroke. D) Report this to the physician as a possible sign of clinical deterioration.

Ans: D Feedback: Alteration in LOC often is the earliest sign of deterioration in a patient with a hemorrhagic stroke. Drowsiness and slight slurring of speech may be early signs that the LOC is deteriorating. This finding is unlikely to be the result of metabolic changes and it is not expected. Stimulating a patient with an acute stroke is usually contraindicated.

A patient with a documented history of seizure disorder experiences a generalized seizure. What nursing action is most appropriate? A) Restrain the patient to prevent injury. B) Open the patient's jaws to insert an oral airway. C) Place patient in high Fowler's position. D) Loosen the patient's restrictive clothing.

Ans: D Feedback: An appropriate nursing intervention would include loosening any restrictive clothing on the patient. No attempt should be made to restrain the patient during the seizure because muscular contractions are strong and restraint can produce injury. Do not attempt to pry open jaws that are clenched in a spasm to insert anything. Broken teeth and injury to the lips and tongue may result from such an action. If possible, place the patient on one side with head flexed forward, which allows the tongue to fall forward and facilitates drainage of saliva and mucus.

The nurse has implemented interventions aimed at facilitating family coping in the care of a patient with a traumatic brain injury. How can the nurse best facilitate family coping? A) Help the family understand that the patient could have died. B) Emphasize the importance of accepting the patient's new limitations. C) Have the members of the family plan the patient's inpatient care. D) Assist the family in setting appropriate short-term goals.

Ans: D Feedback: Helpful interventions to facilitate coping include providing family members with accurate and honest information and encouraging them to continue to set well-defined, short-term goals. Stating that a patient's condition could be worse downplays their concerns. Emphasizing the importance of acceptance may not necessarily help the family accept the patient's condition. Family members cannot normally plan a patient's hospital care, although they may contribute to the care in some ways.

A 13-year-old was brought to the ED, unconscious, after being hit in the head by a baseball. When the child regains consciousness, 5 hours after being admitted, he cannot remember the traumatic event. MRI shows no structural sign of injury. What injury would the nurse suspect the patient has? A) Diffuse axonal injury B) Grade 1 concussion with frontal lobe involvement C) Contusion D) Grade 3 concussion with temporal lobe involvement

Ans: D Feedback: In a grade 3 concussion there is a loss of consciousness lasting from seconds to minutes. Temporal lobe involvement results in amnesia. Frontal lobe involvement can cause uncharacteristic behavior and a grade 1 concussion does not involve loss of consciousness. Diagnostic studies may show no apparent structural sign of injury, but the duration of unconsciousness is an indicator of the severity of the concussion. Diffuse axonal injury (DAI) results from widespread shearing and rotational forces that produce damage throughout the brain—to axons in the cerebral hemispheres, corpus callosum, and brain stem. In cerebral contusion, a moderate to severe head injury, the brain is bruised and damaged in a specific area because of severe acceleration-deceleration force or blunt trauma.

A nurse on the neurologic unit is providing care for a patient who has spinal cord injury at the level of C4. When planning the patient's care, what aspect of the patient's neurologic and functional status should the nurse consider? A) The patient will be unable to use a wheelchair. B) The patient will be unable to swallow food. C) The patient will be continent of urine, but incontinent of bowel. D) The patient will require full assistance for all aspects of elimination

Ans: D Feedback: Patients with a lesion at C4 are fully dependent for elimination. The patient is dependent for feeding, but is able to swallow. The patient will be capable of using an electric wheelchair.

The public health nurse is planning a health promotion campaign that reflects current epidemiologic trends. The nurse should know that hemorrhagic stroke currently accounts for what percentage of total strokes in the United States? A) 43% B) 33% C) 23% D) 13%

Ans: D Feedback: Strokes can be divided into two major categories: ischemic (87%), in which vascular occlusion and significant hypoperfusion occur, and hemorrhagic (13%), in which there is extravasation of blood into the brain or subarachnoid space.

What should be included in the patient's care plan when establishing an exercise program for a patient affected by a stroke? A) Schedule passive range of motion every other day. B) Keep activity limited, as the patient may be over stimulated. C) Have the patient perform active range-of-motion (ROM) exercises once a day. D) Exercise the affected extremities passively four or five times a day.

Ans: D Feedback: The affected extremities are exercised passively and put through a full ROM four or five times a day to maintain joint mobility, regain motor control, prevent development of a contracture in the paralyzed extremity, prevent further deterioration of the neuromuscular system, and enhance circulation. Active ROM exercises should ideally be performed more than once per day.

A patient with a cerebral aneurysm exhibits signs and symptoms of an increase in intracranial pressure (ICP). What nursing intervention would be most appropriate for this patient? A) Range-of-motion exercises to prevent contractures B) Encouraging independence with ADLs to promote recovery C) Early initiation of physical therapy D) Absolute bed rest in a quiet, nonstimulating environment

Ans: D Feedback: The patient is placed on immediate and absolute bed rest in a quiet, nonstressful environment because activity, pain, and anxiety elevate BP, which increases the risk for bleeding. Visitors are restricted. The nurse administers all personal care. The patient is fed and bathed to prevent any exertion that might raise BP.

After a subarachnoid hemorrhage, the patient's laboratory results indicate a serum sodium level of less than 126 mEq/L. What is the nurse's most appropriate action? A) Administer a bolus of normal saline as ordered. B) Prepare the patient for thrombolytic therapy as ordered. C) Facilitate testing for hypothalamic dysfunction. D) Prepare to administer 3% NaCl by IV as ordered.

Ans: D Feedback: The patient may be experiencing syndrome of inappropriate antidiuretic hormone (SIADH) or cerebral salt-wasting syndrome. The treatment most often is the use of IV hypertonic 3% saline. A normal saline bolus would exacerbate the problem and there is no indication for tests of hypothalamic function or thrombolytic therapy.

A nursing student is writing a care plan for a newly admitted patient who has been diagnosed with a stroke. What major nursing diagnosis should most likely be included in the patient's plan of care? A) Adult failure to thrive B) Post-trauma syndrome C) Hyperthermia D) Disturbed sensory perception

Ans: D Feedback: The patient who has experienced a stroke is at a high risk for disturbed sensory perception. Stroke is associated with multiple other nursing diagnoses, but hyperthermia, adult failure to thrive, and post-trauma syndrome are not among these.

A female patient is diagnosed with a right-sided stroke. The patient is now experiencing hemianopsia. How might the nurse help the patient manage her potential sensory and perceptional difficulties? A) Keep the lighting in the patient's room low. B) Place the patient's clock on the affected side. C) Approach the patient on the side where vision is impaired. D) Place the patient's extremities where she can see them.

Ans: D Feedback: The patient with homonymous hemianopsia (loss of half of the visual field) turns away from the affected side of the body and tends to neglect that side and the space on that side; this is called amorphosynthesis. In such instances, the patient cannot see food on half of the tray, and only half of the room is visible. It is important for the nurse to remind the patient constantly of the other side of the body, to maintain alignment of the extremities, and if possible, to place the extremities where the patient can see them. Patients with a decreased field of vision should be approached on the side where visual perception is intact. All visual stimuli (clock, calendar, and television) should be placed on this side. The patient can be taught to turn the head in the direction of the defective visual field to compensate for this loss. Increasing the natural or artificial lighting in the room and providing eyeglasses are important in increasing vision. There is no reason to keep the lights dim.

A patient recovering from a stroke has severe shoulder pain from subluxation of the shoulder and is being cared for on the unit. To prevent further injury and pain, the nurse caring for this patient is aware of what principle of care? A) The patient should be fitted with a cast because use of a sling should be avoided due to adduction of the affected shoulder. B) Elevation of the arm and hand can lead to further complications associated with edema. C) Passively exercising the affected extremity is avoided in order to minimize pain. D) The patient should be taught to interlace fingers, place palms together, and slowly bring scapulae forward to avoid excessive force to shoulder.

Ans: D Feedback: To prevent shoulder pain, the nurse should never lift a patient by the flaccid shoulder or pull on the affected arm or shoulder. The patient is taught how to move and exercise the affected arm/shoulder through proper movement and positioning. The patient is instructed to interlace the fingers, place the palms together, and push the clasped hands slowly forward to bring the scapulae forward; he or she then raises both hands above the head. This is repeated throughout the day. The use of a properly worn sling when the patient is out of bed prevents the paralyzed upper extremity from dangling without support. Range-of-motion exercises are still vitally important in preventing a frozen shoulder and ultimately atrophy of subcutaneous tissues, which can cause more pain. Elevation of the arm and hand is also important in preventing dependent edema of the hand.

A patient with a head injury has been increasingly agitated and the nurse has consequently identified a risk for injury. What is the nurse's best intervention for preventing injury? A) Restrain the patient as ordered. B) Administer opioids PRN as ordered. C) Arrange for friends and family members to sit with the patient. D) Pad the side rails of the patient's bed.

Ans: D Feedback: To protect the patient from self-injury, the nurse uses padded side rails. The nurse should avoid restraints, because straining against them can increase ICP or cause other injury. Narcotics used to control restless patients should be avoided because these medications can depress respiration, constrict the pupils, and alter the patient's responsiveness. Visitors should be limited if the patient is agitated.

The nurse is caring for a patient with increased intracranial pressure (ICP). The patient has a nursing diagnosis of "ineffective cerebral tissue perfusion." What would be an expected outcome that the nurse would document for this diagnosis? A) Copes with sensory deprivation. B) Registers normal body temperature. C) Pays attention to grooming. D) Obeys commands with appropriate motor responses.

Ans: D Feedback: An expected outcome of the diagnosis of ineffective cerebral tissue perfusion in a patient with increased intracranial pressure (ICP) would include obeying commands with appropriate motor responses. Vitals signs and neurologic status are assessed every 15 minutes to every hour. Coping with sensory deprivation would relate to the nursing diagnosis of "disturbed sensory perception." The outcome of "registers normal body temperature" relates to the diagnosis of "potential for ineffective thermoregulation." Body image disturbance would have a potential outcome of "pays attention to grooming."

The nurse is providing care for a patient who is withdrawing from heavy alcohol use. The nurse and other members of the care team are present at the bedside when the patient has a seizure. In preparation for documenting this clinical event, the nurse should note which of the following? A) The ability of the patient to follow instructions during the seizure. B) The success or failure of the care team to physically restrain the patient. C) The patient's ability to explain his seizure during the postictal period. D) The patient's activities immediately prior to the seizure.

Ans: D Feedback: Before and during a seizure, the nurse observes the circumstances before the seizure, including visual, auditory, or olfactory stimuli; tactile stimuli; emotional or psychological disturbances; sleep; and hyperventilation. Communication with the patient is not possible during a seizure and physical restraint is not attempted. The patient's ability to explain the seizure is not clinically relevant.

A nurse is providing care for a patient who has a diagnosis of irritable bowel syndrome (IBS). When planning this patients care, the nurse should collaborate with the patient and prioritize what goal? A) Patient will accurately identify foods that trigger symptoms. B) Patient will demonstrate appropriate care of his ileostomy. C) Patient will demonstrate appropriate use of standard infection control precautions. D) Patient will adhere to recommended guidelines for mobility and activity.

Ans: A Feedback: A major focus of nursing care for the patient with IBS is to identify factors that exacerbate symptoms. Surgery is not used to treat this health problem and infection control is not a concern that is specific to this diagnosis. Establishing causation likely is more important to the patient than managing physical activity.

A nurse is caring for a patient admitted with symptoms of an anorectal infection; cultures indicate that the patient has a viral infection. The nurse should anticipate the administration of what drug? A) Acyclovir (Zovirax) B) Doxycycline (Vibramycin) C) Penicillin (penicillin D) Metronidazole (Flagyl)

Ans: A Feedback: Acyclovir (Zovirax) is often given to patients with viral anorectal infections. Doxycycline (Vibramycin) and penicillin (penicillin G) are drugs of choice for bacterial infections. Metronidazole (Flagyl) is used for other infections with a bacterial etiology; it is ineffective against viruses

A nurse is caring for a patient with cancer of the liver whose condition has required the insertion of a percutaneous biliary drainage system. The nurse's most recent assessment reveals the presence of dark green fluid in the collection container. What is the nurse's best response to this assessment finding? A) Document the presence of normal bile output. B) Irrigate the drainage system with normal saline as ordered. C) Aspirate a sample of the drainage for culture. D) Promptly report this assessment finding to the primary care provider.

Ans: A Feedback: Bile is usually a dark green or brownish-yellow color, so this would constitute an expected assessment finding, with no other action necessary.

A 35-year-old male patient presents at the emergency department with symptoms of a small bowel obstruction. In collaboration with the primary care provider, what intervention should the nurse prioritize? A) Insertion of a nasogastric tube Test Bank - Brunner & Suddarth's Textbook of Medical-Surgical Nursing 14e (Hinkle 2017) 895 B) Insertion of a central venous catheter C) Administration of a mineral oil enema D) Administration of a glycerin suppository and an oral laxative

Ans: A Feedback: Decompression of the bowel through a nasogastric tube is necessary for all patients with small bowel obstruction. Peripheral IV access is normally sufficient. Enemas, suppositories, and laxatives are not indicated if an obstruction is present.

A group of nurses have attended an inservice on the prevention of occupationally acquired diseases that affect healthcare providers. What action has the greatest potential to reduce a nurse's risk of acquiring hepatitis C in the workplace? A) Disposing of sharps appropriately and not recapping needles B) Performing meticulous hand hygiene at the appropriate moments in care C) Adhering to the recommended schedule of immunizations D) Wearing an N95 mask when providing care for patients on airborne precautions

Ans: A Feedback: HCV is bloodborne. Consequently, prevention of needlestick injuries is paramount. Hand hygiene, immunizations and appropriate use of masks are important aspects of overall infection control, but these actions do not directly mitigate the risk of HCV.

A patient with a liver mass is undergoing a percutaneous liver biopsy. What action should the nurse perform when assisting with this procedure? A) Position the patient on the right side with a pillow under the costal margin after the procedure. B) Administer 1 unit of albumin 90 minutes before the procedure as ordered. C) Administer at least 1 unit of packed red blood cells as ordered the day before the scheduled procedure. D) Confirm that the patient's electrolyte levels have been assessed prior to the procedure.

Ans: A Feedback: Immediately after a percutaneous liver biopsy, assist the patient to turn onto the right side and place a pillow under the costal margin. Prior administration of albumin or PRBCs is unnecessary. Coagulation tests should be performed, but electrolyte analysis is not necessary.

A patient has developed hepatic encephalopathy secondary to cirrhosis and is receiving care on the medical unit. The patient's current medication regimen includes lactulose (Cephulac) four times daily. What desired outcome should the nurse relate to this pharmacologic intervention? A) Two to 3 soft bowel movements daily B) Significant increase in appetite and food intake C) Absence of nausea and vomiting D) Absence of blood or mucus in stool

Ans: A Feedback: Lactulose (Cephulac) is administered to reduce serum ammonia levels. Two or three soft stools per day are desirable; this indicates that lactulose is performing as intended. Lactulose does not address the patient's appetite, symptoms of nausea and vomiting, or the development of blood and mucus in the stool.

A teenage patient with a pilonidal cyst has been brought for care by her mother. The nurse who is contributing to the patients care knows that treatment will be chosen based on what risk? A) Risk for infection B) Risk for bowel incontinence C) Risk for constipation D) Risk for impaired tissue perfusion

Ans: A Feedback: Pilonidal cysts frequently develop into an abscess, necessitating surgical repair. These cysts do not contribute to bowel incontinence, constipation, or impaired tissue perfusion.

A nurse is presenting an educational event to a local community group. When speaking about colorectal cancer, what risk factor should the nurse cite? A) High levels of alcohol consumption B) History of bowel obstruction C) History of diverticulitis D) Longstanding psychosocial stress

Ans: A Feedback: Risk factors include high alcohol intake; cigarette smoking; and high-fact, high-protein, low-fiber diet. Diverticulitis, obstruction, and stress are not noted as risk factors for colorectal cancer.

A triage nurse in the emergency department is assessing a patient who presented with complaints of general malaise. Assessment reveals the presence of jaundice and increased abdominal girth. What assessment question best addresses the possible etiology of this patient's presentation? A) "How many alcoholic drinks do you typically consume in a week?" B) "To the best of your knowledge, are your immunizations up to date?" C) "Have you ever worked in an occupation where you might have been exposed to toxins?" D) "Has anyone in your family ever experienced symptoms similar to yours?"

Ans: A Feedback: Signs or symptoms of hepatic dysfunction indicate a need to assess for alcohol use. Immunization status, occupational risks, and family history are also relevant considerations, but alcohol use is a more common etiologic factor in liver disease.

The nurse is providing care for a patient whose inflammatory bowel disease has necessitated hospital treatment. Which of the following would most likely be included in the patients medication regimen? A) Anticholinergic medications 30 minutes before a meal B) Antiemetics on a PRN basis C) Vitamin B12 injections to prevent pernicious anemia D) Beta adrenergic blockers to reduce bowel motility

Ans: A Feedback: The nurse administers anticholinergic medications 30 minutes before a meal as prescribed to decrease intestinal motility and administers analgesics as prescribed for pain. Antiemetics, vitamin B12 injections and beta blockers do not address the signs, symptoms, or etiology of inflammatory bowel disease

A nurse is caring for a patient with hepatic encephalopathy. While making the initial shift assessment, the nurse notes that the patient has a flapping tremor of the hands. The nurse should document the presence of what sign of liver disease? A) Asterixis B) Constructional apraxia C) Fetor hepaticus D) Palmar erythema

Ans: A Feedback: The nurse will document that a patient exhibiting a flapping tremor of the hands is demonstrating asterixis. While constructional apraxia is a motor disturbance, it is the inability to reproduce a simple figure. Fetor hepaticus is a sweet, slightly fecal odor to the breath and not associated with a motor disturbance. Skin changes associated with liver dysfunction may include palmar erythema, which is a reddening of the palms, but is not a flapping tremor.

A nurse is preparing to provide care for a patient whose exacerbation of ulcerative colitis has required hospital admission. During an exacerbation of this health problem, the nurse would anticipate that the patients stools will have what characteristics? A) Watery with blood and mucus B) Hard and black or tarry C) Dry and streaked with blood D) Loose with visible fatty streaks

Ans: A Feedback: The predominant symptoms of ulcerative colitis are diarrhea and abdominal pain. Stools may be bloody and contain mucus. Stools are not hard, dry, tarry, black or fatty in patients who have ulcerative colitis.

A 55-year-old female patient with hepatocellular carcinoma (HCC) is undergoing radiofrequency ablation. The nurse should recognize what goal of this treatment? A) Destruction of the patient's liver tumor B) Restoration of portal vein patency C) Destruction of a liver abscess D) Reversal of metastasis

Ans: A Feedback: Using radiofrequency ablation, a tumor up to 5 cm in size can be destroyed in one treatment session. This technique does not address circulatory function or abscess formation. It does not allow for the reversal of metastasis.

A patient is admitted to the medical unit with a diagnosis of intestinal obstruction. When planning this patients care, which of the following nursing diagnoses should the nurse prioritize? A) Ineffective Tissue Perfusion Related to Bowel Ischemia B) Imbalanced Nutrition: Less Than Body Requirements Related to Impaired Absorption C) Anxiety Related to Bowel Obstruction and Subsequent Hospitalization D) Impaired Skin Integrity Related to Bowel Obstruction

Ans: A Feedback: When the bowel is completely obstructed, the possibility of strangulation and tissue necrosis (i.e., tissue death) warrants surgical intervention. As such, this immediate physiologic need is a nursing priority. Nutritional support and management of anxiety are necessary, but bowel ischemia is a more immediate threat. Skin integrity is not threatened.

A patient has developed diabetes insipidus after having increased ICP following head trauma. What nursing assessment best addresses this complication? A) Vigilant monitoring of fluid balance B) Continuous BP monitoring C) Serial arterial blood gases (ABGs) D) Monitoring of the patient's airway for patency

Ans: A Feedback:Diabetes insipidus requires fluid and electrolyte replacement, along with the administration of vasopressin, to replace and slow the urine output. Because of these alterations in fluid balance, careful monitoring is necessary. None of the other listed assessments directly addresses the major manifestations of diabetes insipidus.

A nurse is caring for a patient who has been admitted to the hospital with diverticulitis. Which of the following would be appropriate nursing diagnoses for this patient? Select all that apply. A) Acute Pain Related to Increased Peristalsis and GI Inflammation B) Activity Intolerance Related to Generalized Weakness C) Bowel Incontinence Related to Increased Intestinal Peristalsis D) Deficient Fluid Volume Related to Anorexia, Nausea, and Diarrhea E) Impaired Urinary Elimination Related to GI Pressure on the Bladder

Ans: A, B, D Feedback: Patients with diverticulitis are likely to experience pain and decreased activity levels, and are at risk of fluid volume deficit. The patient is unlikely to experience fecal incontinence and urinary function is not directly influenced.

A patient's physician has ordered a "liver panel" in response to the patient's development of jaundice. When reviewing the results of this laboratory testing, the nurse should expect to review what blood tests? Select all that apply. A) Alanine aminotransferase (ALT) B) C-reactive protein (CRP) C) Gamma-glutamyl transferase (GGT) D) Aspartate aminotransferase (AST) E) B-type natriuretic peptide (BNP)

Ans: A, C, D Feedback: Liver function testing includes GGT, ALT, and AST. CRP addresses the presence of generalized inflammation and BNP is relevant to heart failure; neither is included in a liver panel.

A patient with a diagnosis of colon cancer is 2 days postoperative following bowel resection and anastomosis. The nurse has planned the patients care in the knowledge of potential complications. What Test Bank - Brunner & Suddarth's Textbook of Medical-Surgical Nursing 14e (Hinkle 2017) 909 assessment should the nurse prioritize? A) Close monitoring of temperature B) Frequent abdominal auscultation C) Assessment of hemoglobin, hematocrit, and red blood cell levels D) Palpation of peripheral pulses and leg girth

Ans: B Feedback: After bowel surgery, it is important to frequently assess the abdomen, including bowel sounds and abdominal girth, to detect bowel obstruction. The resumption of bowel motility is a priority over each of the other listed assessments, even though each should be performed by the nurse.

A patient with a diagnosis of esophageal varices has undergone endoscopy to gauge the progression of this complication of liver disease. Following the completion of this diagnostic test, what nursing intervention should the nurse perform? A) Keep patient NPO until the results of test are known. B) Keep patient NPO until the patient's gag reflex returns. C) Administer analgesia until post-procedure tenderness is relieved. D) Give the patient a cold beverage to promote swallowing ability.

Ans: B Feedback: After the examination, fluids are not given until the patient's gag reflex returns. Lozenges and gargles may be used to relieve throat discomfort if the patient's physical condition and mental status permit. The result of the test is known immediately. Food and fluids are contraindicated until the gag reflex returns.

A nurse caring for a patient with colorectal cancer is preparing the patient for upcoming surgery. The nurse administers cephalexin (Keflex) to the patient and explains what rationale? A) To treat any undiagnosed infections B) To reduce intestinal bacteria levels C) To reduce bowel motility D) To reduce abdominal distention postoperatively

Ans: B Feedback: Antibiotics such a kanamycin (Kantrex), neomycin (Mycifradin), and cephalexin (Keflex) are administered orally the day before surgery to reduce intestinal bacterial. Preoperative antibiotics are not given to treat undiagnosed infections, reduce motility, or prevent abdominal distention.

A nurse is caring for a patient who has been admitted for the treatment of advanced cirrhosis. What assessment should the nurse prioritize in this patient's plan of care? A) Measurement of abdominal girth and body weight B) Assessment for variceal bleeding C) Assessment for signs and symptoms of jaundice D) Monitoring of results of liver function testing

Ans: B Feedback: Esophageal varices are a major cause of mortality in patients with uncompensated cirrhosis. Consequently, this should be a focus of the nurse's assessments and should be prioritized over the other listed assessments, even though each should be performed.

A nurse is providing care for a patient whose recent colostomy has contributed to a nursing diagnosis of Disturbed Body Image Related to Colostomy. What intervention best addresses this diagnosis? A) Encourage the patient to conduct online research into colostomies. B) Engage the patient in the care of the ostomy to the extent that the patient is willing. C) Emphasize the fact that the colostomy was needed to alleviate a much more serious health problem. D) Emphasize the fact that the colostomy is temporary measure and is not permanent

Ans: B Feedback: For some patients, becoming involved in the care of the ostomy helps to normalize it and enhance familiarity. Emphasizing the benefits of the intervention is unlikely to improve the patients body image, since the benefits are likely already known. Online research is not likely to enhance the patients body image and some ostomies are permanent.

A patient has had an ileostomy created for the treatment of irritable bowel disease and the patient is now preparing for discharge. What should the patient be taught about changing this device in the home Test Bank - Brunner & Suddarth's Textbook of Medical-Surgical Nursing 14e (Hinkle 2017) 894 setting? A) Apply antibiotic ointment as ordered after cleaning the stoma. B) Apply a skin barrier to the peristomal skin prior to applying the pouch. C) Dispose of the clamp with each bag change. D) Cleanse the area surrounding the stoma with alcohol or chlorhexidine.

Ans: B Feedback: Guidelines for changing an ileostomy appliance are as follows. Skin should be washed with soap and water, and dried. A skin barrier should be applied to the peristomal skin prior to applying the pouch. Clamps are supplied one per box and should be reused with each bag change. Topical antibiotics are not utilized, but an antifungal spray or powder may be used.

During a patients scheduled home visit, an older adult patient has stated to the community health nurse that she has been experiencing hemorrhoids of increasing severity in recent months. The nurse should recommend which of the following? A) Regular application of an OTC antibiotic ointment B) Increased fluid and fiber intake C) Daily use of OTC glycerin suppositories D) Use of an NSAID to reduce inflammation

Ans: B Feedback: Hemorrhoid symptoms and discomfort can be relieved by good personal hygiene and by avoiding excessive straining during defecation. A high-residue diet that contains fruit and bran along with an Test Bank - Brunner & Suddarth's Textbook of Medical-Surgical Nursing 14e (Hinkle 2017) 905 increased fluid intake may be all the treatment that is necessary to promote the passage of soft, bulky stools to prevent straining. Antibiotics, regular use of suppositories, and NSAIDs are not recommended, as they do not address the etiology of the health problem.

A nursing instructor is discussing hemorrhoids with the nursing class. Which patients would the nursing instructor identify as most likely to develop hemorrhoids? A) A 45-year-old teacher who stands for 6 hours per day B) A pregnant woman at 28 weeks gestation C) A 37-year-old construction worker who does heavy lifting D) A 60-year-old professional who is under stress

Ans: B Feedback: Hemorrhoids commonly affect 50% of patients after the age of 50. Pregnancy may initiate hemorrhoids or aggravate existing ones. This is due to increased constipation during pregnancy. The significance of pregnancy is greater than that of standing, lifting, or stress in the development of hemorrhoids.

While completing a health history on a patient who has recently experienced a seizure, the nurse would assess for what characteristic associated with the postictal state? A)Epileptic cry B) Confusion C) Urinary incontinence D) Body rigidity

Ans: B Feedback: In the postictal state (after the seizure), the patient is often confused and hard to arouse and may sleep for hours. The epileptic cry occurs from the simultaneous contractions of the diaphragm and chest muscles that occur during the seizure. Urinary incontinence and intense rigidity of the entire body are followed by alternating muscle relaxation and contraction (generalized tonic-clonic contraction) during the seizure.

A nurse caring for a patient with a newly created ileostomy assesses the patient and notes that the patient has had not ostomy output for the past 12 hours. The patient also complains of worsening nausea. What is the nurses priority action? A) Facilitate a referral to the wound-ostomy-continence (WOC) nurse. B) Report signs and symptoms of obstruction to the physician. C) Encourage the patient to mobilize in order to enhance motility. D) Contact the physician and obtain a swab of the stoma for culture

Ans: B Feedback: It is important to report nausea and abdominal distention, which may indicate intestinal obstruction. This requires prompt medical intervention. Referral to the WOC nurse is not an appropriate short-term response, since medical treatment is necessary. Physical mobility will not normally resolve an obstruction. There is no need to collect a culture from the stoma, because infection is unrelated to this problem.

A nurse is caring for a patient with constipation whose primary care provider has recommended senna (Senokot) for the management of this condition. The nurse should provide which of the following education points? A) Limit your fluid intake temporarily so you dont get diarrhea. B) Avoid taking the drug on a long-term basis. C) Make sure to take a multivitamin with each dose. D) Take this on an empty stomach to ensure maximum effect.

Ans: B Feedback: Laxatives should not be taken on an ongoing basis in order to reduce the risk of dependence. Fluid should be increased, not limited, and there is no need to take each dose with a multivitamin. Senna does not need to be taken on an empty stomach.

A nurse is caring for a patient with cirrhosis secondary to heavy alcohol use. The nurse's most recent assessment reveals subtle changes in the patient's cognition and behavior. What is the nurse's most appropriate response? A) Ensure that the patient's sodium intake does not exceed recommended levels. B) Report this finding to the primary care provider due to the possibility of hepatic encephalopathy. C) Inform the primary care provider that the patient should be assessed for alcoholic hepatitis. D) Implement interventions aimed at ensuring a calm and therapeutic care environment.

Ans: B Feedback: Monitoring is an essential nursing function to identify early deterioration in mental status. The nurse monitors the patient's mental status closely and reports changes so that treatment of encephalopathy can be initiated promptly. This change in status is likely unrelated to sodium intake and would not signal the onset of hepatitis. A supportive care environment is beneficial, but does not address the patient's physiologic deterioration.

A patient admitted with inflammatory bowel disease asks the nurse for help with menu selections. What menu selection is most likely the best choice for this patient? A) Spinach B) Tofu C) Multigrain bagel D) Blueberries

Ans: B Feedback: Nutritional management of inflammatory bowel disease requires ingestion of a diet that is bland, lowresidue, high-protein, and high-vitamin. Tofu meets each of the criteria. Spinach, multigrain bagels, and blueberries are not low-residue.

A nurse at an outpatient surgery center is caring for a patient who had a hemorrhoidectomy. What discharge education topics should the nurse address with this patient? A) The appropriate use of antibiotics to prevent postoperative infection B) The correct procedure for taking a sitz bath C) The need to eat a low-residue, low-fat diet for the next 2 weeks Test Bank - Brunner & Suddarth's Textbook of Medical-Surgical Nursing 14e (Hinkle 2017) 910 D) The correct technique for keeping the perianal region clean without the use of water

Ans: B Feedback: Sitz baths are usually indicated after perianal surgery. A low-residue, low-fat diet is not necessary and water is used to keep the region clean. Postoperative antibiotics are not normally prescribed.

A patient has been diagnosed with advanced stage breast cancer and will soon begin aggressive treatment. What assessment findings would most strongly suggest that the patient may have developed liver metastases? A) Persistent fever and cognitive changes B) Abdominal pain and hepatomegaly C) Peripheral edema unresponsive to diuresis D) Spontaneous bleeding and jaundice

Ans: B Feedback: The early manifestations of malignancy of the liver include pain—a continuous dull ache in the right upper quadrant, epigastrium, or back. Weight loss, loss of strength, anorexia, and anemia may also occur. The liver may be enlarged and irregular on palpation. Jaundice is present only if the larger bile ducts are occluded by the pressure of malignant nodules in the hilum of the liver. Fever, cognitive changes, peripheral edema, and bleeding are atypical signs.

An older adult who resides in an assisted living facility has sought care from the nurse because of recurrent episodes of constipation. Which of the following actions should the nurse first perform? A) Encourage the patient to take stool softener daily. Test Bank - Brunner & Suddarth's Textbook of Medical-Surgical Nursing 14e (Hinkle 2017) 898 B) Assess the patients food and fluid intake. C) Assess the patients surgical history. D) Encourage the patient to take fiber supplements.

Ans: B Feedback: The nurse should follow the nursing process and perform an assessment prior to interventions. The patients food and fluid intake is more likely to affect bowel function than surgery.

A patient admitted with acute diverticulitis has experienced a sudden increase in temperature and complains of a sudden onset of exquisite abdominal tenderness. The nurses rapid assessment reveals that the patients abdomen is uncharacteristically rigid on palpation. What is the nurses best response? A) Administer a Fleet enema as ordered and remain with the patient. B) Contact the primary care provider promptly and report these signs of perforation. C) Position the patient supine and insert an NG tube. D) Page the primary care provider and report that the patient may be obstructed.

Ans: B Feedback: The patients change in status is suggestive of perforation, which is a surgical emergency. Obstruction does not have this presentation involving fever and abdominal rigidity. An enema would be strongly contraindicated. An order is needed for NG insertion and repositioning is not a priority.

A patient with a history of injection drug use has been diagnosed with hepatitis C. When collaborating with the care team to plan this patient's treatment, the nurse should anticipate what intervention? A) Administration of immune globulins B) A regimen of antiviral medications C) Rest and watchful waiting D) Administration of fresh-frozen plasma (FFP)

Ans: B Feedback: There is no benefit from rest, diet, or vitamin supplements in HCV treatment. Studies have demonstrated that a combination of two antiviral agents, Peg-interferon and ribavirin (Rebetol), is effective in producing improvement in patients with hepatitis C and in treating relapses. Immune globulins and FFP are not indicated.

The nurse is caring for a patient who is undergoing diagnostic testing for suspected malabsorption. When taking this patients health history and performing the physical assessment, the nurse should recognize what finding as most consistent with this diagnosis? A) Recurrent constipation coupled with weight loss B) Foul-smelling diarrhea that contains fat C) Fever accompanied by a rigid, tender abdomen D) Bloody bowel movements accompanied by fecal incontinence

Ans: B Test Bank - Brunner & Suddarth's Textbook of Medical-Surgical Nursing 14e (Hinkle 2017) 900 Feedback: The hallmarks of malabsorption syndrome from any cause are diarrhea or frequent, loose, bulky, foulsmelling stools that have increased fat content and are often grayish (steatorrhea). Constipation and bloody bowel movements are not suggestive of malabsorption syndromes. Fever and a tender, rigid abdomen are associated with peritonitis

An adult patient has been diagnosed with diverticular disease after ongoing challenges with constipation. The patient will be treated on an outpatient basis. What components of treatment should the nurse anticipate? Select all that apply. A) Anticholinergic medications B) Increased fiber intake C) Enemas on alternating days D) Reduced fat intake E) Fluid reduction

Ans: B, D Feedback: Patients whose diverticular disease does not warrant hospital treatment often benefit from a high-fiber, low-fat diet. Neither enemas nor anticholinergics are indicated, and fluid intake is encouraged

A nurse has entered the room of a patient with cirrhosis and found the patient on the floor. The patient states that she fell when transferring to the commode. The patient's vital signs are within reference ranges and the nurse observes no apparent injuries. What is the nurse's most appropriate action? A) Remove the patient's commode and supply a bedpan. B) Complete an incident report and submit it to the unit supervisor. C) Have the patient assessed by the physician due to the risk of internal bleeding. D) Perform a focused abdominal assessment in order to rule out injury.

Ans: C Feedback: A fall would necessitate thorough medical assessment due to the patient's risk of bleeding. The nurse's abdominal assessment is an appropriate action, but is not wholly sufficient to rule out internal injury. Medical assessment is a priority over removing the commode or filling out an incident report, even though these actions are appropriate.

A nurse is assessing a patients stoma on postoperative day 3. The nurse notes that the stoma has a shiny appearance and a bright red color. How should the nurse best respond to this assessment finding? A) Irrigate the ostomy to clear a possible obstruction. B) Contact the primary care provider to report this finding. C) Document that the stoma appears healthy and well perfused. D) Document a nursing diagnosis of Impaired Skin Integrity.

Ans: C Feedback: A healthy, viable stoma should be shiny and pink to bright red. This finding does not indicate that the stoma is blocked or that skin integrity is compromised.

A patients screening colonoscopy revealed the presence of numerous polyps in the large bowel. What principle should guide the subsequent treatment of this patients health problem? A) Adherence to a high-fiber diet will help the polyps resolve. B) The patient should be assured that these are a normal, age-related physiologic change. C) The patients polyps constitute a risk factor for cancer. D) The presence of polyps is associated with an increased risk of bowel obstruction.

Ans: C Feedback: Although most polyps do not develop into invasive neoplasms, they must be identified and followed closely. They are very common, but are not classified as a normal, age-related physiologic change. Diet will not help them resolve and they do not typically lead to obstructions.

An older adult has a diagnosis of Alzheimers disease and has recently been experiencing fecal incontinence. However, the nurse has observed no recent change in the character of the patients stools. What is the nurses most appropriate intervention? A) Keep a food diary to determine the foods that exacerbate the patients symptoms. B) Provide the patient with a bland, low-residue diet. C) Toilet the patient on a frequent, scheduled basis. D) Liaise with the primary care provider to obtain an order for loperamide

Ans: C Feedback: Because the patients fecal incontinence is most likely attributable to cognitive decline, frequent toileting is an appropriate intervention. Loperamide is unnecessary in the absence of diarrhea. Specific foods are not likely to be a cause of, or solution to, this patients health problem.

A patients health history is suggestive of inflammatory bowel disease. Which of the following would suggest Crohns disease, rather that ulcerative colitis, as the cause of the patients signs and symptoms? A) A pattern of distinct exacerbations and remissions B) Severe diarrhea C) An absence of blood in stool D) Involvement of the rectal mucosa

Ans: C Feedback: Bloody stool is far more common in cases of UC than in Crohns. Rectal involvement is nearly 100% in cases of UC (versus 20% in Crohns) and patients with UC typically experience severe diarrhea. UC is also characterized by a pattern of remissions and exacerbations, while Crohns often has a more prolonged and variable course.

A nurse is planning discharge teaching for a 21-year-old patient with a new diagnosis of ulcerative colitis. When planning family assessment, the nurse should recognize that which of the following factors will likely have the greatest impact on the patients coping after discharge? A) The familys ability to take care of the patients special diet needs B) The familys ability to monitor the patients changing health status C) The familys ability to provide emotional support D) The familys ability to manage the patients medication regimen

Ans: C Feedback: Emotional support from the family is key to the patients coping after discharge. A 21-year-old would be expected to self-manage the prescribed medication regimen and the family would not be primarily responsible for monitoring the patients health status. It is highly beneficial if the family is willing and able to accommodate the patients dietary needs, but emotional support is paramount and cannot be solely provided by the patient alone.

A nurse is working with a patient who has chronic constipation. What should be included in patient teaching to promote normal bowel function? A) Use glycerin suppositories on a regular basis. B) Limit physical activity in order to promote bowel peristalsis. C) Consume high-residue, high-fiber foods. D) Resist the urge to defecate until the urge becomes intense.

Ans: C Feedback: Goals for the patient include restoring or maintaining a regular pattern of elimination by responding to the urge to defecate, ensuring adequate intake of fluids and high-fiber foods, learning about methods to avoid constipation, relieving anxiety about bowel elimination patterns, and avoiding complications. Ongoing use of pharmacologic aids should not be promoted, due to the risk of dependence. Increased mobility helps to maintain a regular pattern of elimination. The urge to defecate should be heeded.

A patients colorectal cancer has necessitated a hemicolectomy with the creation of a colostomy. In the 4 days since the surgery, the patient has been unwilling to look at the ostomy or participate in any aspects of ostomy care. What is the nurses most appropriate response to this observation? A) Ensure that the patient knows that he or she will be responsible for care after discharge. B) Reassure the patient that many people are fearful after the creation of an ostomy. C) Acknowledge the patients reluctance and initiate discussion of the factors underlying it. D) Arrange for the patient to be seen by a social worker or spiritual advisor.

Ans: C Feedback: If the patient is reluctant to participate in ostomy care, the nurse should attempt to dialogue about this with the patient and explore the factors that underlie it. It is presumptive to assume that the patients behavior is motivated by fear. Assessment must precede referrals and emphasizing the patients responsibilities may or may not motivate the patient

A patient with portal hypertension has been admitted to the medical floor. The nurse should prioritize which of the following assessments related to the manifestations of this health problem? A) Assessment of blood pressure and assessment for headaches and visual changes B) Assessments for signs and symptoms of venous thromboembolism C) Daily weights and abdominal girth measurement D) Blood glucose monitoring q4h

Ans: C Feedback: Obstruction to blood flow through the damaged liver results in increased blood pressure (portal hypertension) throughout the portal venous system. This can result in varices and ascites in the abdominal cavity. Assessments related to ascites are daily weights and abdominal girths. Portal hypertension is not synonymous with cardiovascular hypertension and does not create a risk for unstable blood glucose or VTE.

A nurse is caring for a patient with hepatic encephalopathy. The nurse's assessment reveals that the patient exhibits episodes of confusion, is difficult to arouse from sleep and has rigid extremities. Based on these clinical findings, the nurse should document what stage of hepatic encephalopathy? A) Stage 1 B) Stage 2 C) Stage 3 D) Stage 4

Ans: C Feedback: Patients in the third stage of hepatic encephalopathy exhibit the following symptoms: stuporous, difficult to arouse, sleeps most of the time, exhibits marked confusion, incoherent in speech, asterixis, increased deep tendon reflexes, rigidity of extremities, marked EEG abnormalities. Patients in stages 1 and 2 exhibit clinical symptoms that are not as advanced as found in stage 3, and patients in stage 4 are comatose. In stage 4, there is an absence of asterixis, absence of deep tendon reflexes, flaccidity of extremities, and EEG abnormalities.

Diagnostic testing has revealed that a patient's hepatocellular carcinoma (HCC) is limited to one lobe. The nurse should anticipate that this patient's plan of care will focus on what intervention? A) Cryosurgery B) Liver transplantation C) Lobectomy D) Laser hyperthermia

Ans: C Feedback: Surgical resection is the treatment of choice when HCC is confined to one lobe of the liver and the function of the remaining liver is considered adequate for postoperative recovery. Removal of a lobe of the liver (lobectomy) is the most common surgical procedure for excising a liver tumor. While cryosurgery and liver transplantation are other surgical options for management of liver cancer, these procedures are not performed at the same frequency as a lobectomy. Laser hyperthermia is a nonsurgical treatment for liver cancer.

A nurse is working with a patient who is learning to care for a continent ileostomy (Kock pouch). Following the initial period of healing, the nurse is teaching the patient how to independently empty the ileostomy. The nurse should teach the patient to do which of the following actions? Test Bank - Brunner & Suddarth's Textbook of Medical-Surgical Nursing 14e (Hinkle 2017) 902 A) Aim to eventually empty the pouch every 90 minutes. B) Avoid emptying the pouch until it is visibly full. C) Insert the catheter approximately 5 cm into the pouch. D) Aspirate the contents of the pouch using a 60 mL piston syringe.

Ans: C Feedback: To empty a Kock pouch, the catheter is gently inserted approximately 5 cm to the point of the valve or nipple. The length of time between drainage periods is gradually increased until the reservoir needs to be drained only every 4 to 6 hours and irrigated once each day. It is not appropriate to wait until the pouch is full, and this would not be visible. The contents of the pouch are not aspirated.

A nurse is performing an admission assessment of a patient with a diagnosis of cirrhosis. What technique should the nurse use to palpate the patient's liver? A) Place hand under the right lower abdominal quadrant and press down lightly with the other hand. B) Place the left hand over the abdomen and behind the left side at the 11th rib. C) Place hand under right lower rib cage and press down lightly with the other hand. D) Hold hand 90 degrees to right side of the abdomen and push down firmly.

Ans: C Feedback: To palpate the liver, the examiner places one hand under the right lower rib cage and presses downward with light pressure with the other hand. The liver is not on the left side or in the right lower abdominal quadrant.

A nurse is talking with a patient who is scheduled to have a hemicolectomy with the creation of a colostomy. The patient admits to being anxious, and has many questions concerning the surgery, the care of a stoma, and necessary lifestyle changes. Which of the following nursing actions is most appropriate? A) Reassure the patient that the procedure is relatively low risk and that patients are usually successful in adjusting to an ostomy. B) Provide the patient with educational materials that match the patients learning style. C) Encourage the patient to write down these concerns and questions to bring forward to the surgeon. Test Bank - Brunner & Suddarth's Textbook of Medical-Surgical Nursing 14e (Hinkle 2017) 899 D) Maintain an open dialogue with the patient and facilitate a referral to the wound-ostomycontinence (WOC) nurse.

Ans: D Feedback: A wound-ostomy-continence (WOC) nurse is a registered nurse who has received advanced education in an accredited program to care for patients with stomas. The enterostomal nurse therapist can assist with the selection of an appropriate stoma site, teach about stoma care, and provide emotional support. The surgeon is less likely to address the patients psychosocial and learning needs. Reassurance does not address the patients questions, and education may or may not alleviate anxiety.

A nurse is caring for a patient with liver failure and is performing an assessment in the knowledge of the patient's increased risk of bleeding. The nurse recognizes that this risk is related to the patient's inability to synthesize prothrombin in the liver. What factor most likely contributes to this loss of function? A) Alterations in glucose metabolism B) Retention of bile salts C) Inadequate production of albumin by hepatocytes D) Inability of the liver to use vitamin K

Ans: D Feedback: Decreased production of several clotting factors may be partially due to deficient absorption of vitamin K from the GI tract. This probably is caused by the inability of liver cells to use vitamin K to make prothrombin. This bleeding risk is unrelated to the roles of glucose, bile salts, or albumin.

A patient with liver cancer is being discharged home with a biliary drainage system in place. The nurse should teach the patient's family how to safely perform which of the following actions? A) Aspirating bile from the catheter using a syringe B) Removing the catheter when output is 15 mL in 24 hours C) Instilling antibiotics into the catheter D) Assessing the patency of the drainage catheter

Ans: D Feedback: Families should be taught to provide basic catheter care, including assessment of patency. Antibiotics are not instilled into the catheter and aspiration using a syringe is contraindicated. The family would not independently remove the catheter; this would be done by a member of the care team when deemed necessary.

A nurse is caring for an older adult who has been experiencing severe Clostridium difficile-related diarrhea. When reviewing the patients most recent laboratory tests, the nurse should prioritize which of the following? A) White blood cell level Test Bank - Brunner & Suddarth's Textbook of Medical-Surgical Nursing 14e (Hinkle 2017) 907 B) Creatinine level C) Hemoglobin level D) Potassium level

Ans: D Feedback: In elderly patients, it is important to monitor the patients serum electrolyte levels closely. Diarrhea is less likely to cause an alteration in white blood cell, creatinine, and hemoglobin levels.

A patient with liver cancer is being discharged home with a hepatic artery catheter in place. The nurse should be aware that this catheter will facilitate which of the following? A) Continuous monitoring for portal hypertension B) Administration of immunosuppressive drugs during the first weeks after transplantation C) Real-time monitoring of vascular changes in the hepatic system D) Delivery of a continuous chemotherapeutic dose

Ans: D Feedback: In most cases, the hepatic artery catheter has been inserted surgically and has a prefilled infusion pump that delivers a continuous chemotherapeutic dose until completed. The hepatic artery catheter does not monitor portal hypertension, deliver immunosuppressive drugs, or monitor vascular changes in the hepatic system.

A patient has been experiencing disconcerting GI symptoms that have been worsening in severity. Following medical assessment, the patient has been diagnosed with lactose intolerance. The nurse should recognize an increased need for what form of health promotion? A) Annual screening colonoscopies B) Adherence to recommended immunization schedules C) Regular blood pressure monitoring Test Bank - Brunner & Suddarth's Textbook of Medical-Surgical Nursing 14e (Hinkle 2017) 903 D) Frequent screening for osteoporosis

Ans: D Feedback: Persons with lactose intolerance often experience hypocalcemia and a consequent risk of osteoporosis related to malabsorption of calcium. Lactose intolerance does not create an increased need for screening for colorectal cancer, immunizations, or blood pressure monitoring.

A patient has been experiencing occasional episodes of constipation and has been unable to achieve consistent relief by increasing physical activity and improving his diet. What pharmacologic intervention should the nurse recommend to the patient for ongoing use? A) Mineral oil enemas B) Bisacodyl (Dulcolax) C) Senna (Senokot) D) Psyllium hydrophilic mucilloid (Metamucil)

Ans: D Feedback: Psyllium hydrophilic mucilloid (Metamucil) is a bulk-forming laxative that is safe for ongoing use. None of the other listed laxatives should be used on an ongoing basis because of the risk of dependence.

A nurse is performing an admission assessment for an 81-year-old patient who generally enjoys good health. When considering normal, age-related changes to hepatic function, the nurse should anticipate what finding? A) Similar liver size and texture as in younger adults B) A nonpalpable liver C) A slightly enlarged liver with palpably hard edges D) A slightly decreased size of the liver

Ans: D Feedback: The most common age-related change in the liver is a decrease in size and weight. The liver is usually still palpable, however, and is not expected to have hardened edges.

A nurse is teaching a group of adults about screening and prevention of colorectal cancer. The nurse should describe which of the following as the most common sign of possible colon cancer? A) Development of new hemorrhoids B) Abdominal bloating and flank pain C) Unexplained weight gain D) Change in bowel habits

Ans: D Feedback: The most common presenting symptom associated with colorectal cancer is a change in bowel habits. The passage of blood is the second most common symptom. Symptoms may also include unexplained anemia, anorexia, weight loss, and fatigue. Hemorrhoids and bloating are atypical.

Which of the following is the most plausible nursing diagnosis for a patient whose treatment for colon cancer has necessitated a colonostomy? A) Risk for Unstable Blood Glucose Due to Changes in Digestion and Absorption B) Unilateral Neglect Related to Decreased Physical Mobility C) Risk for Excess Fluid Volume Related to Dietary Changes and Changes In Absorption D) Ineffective Sexuality Patterns Related to Changes in Self-Concept

Ans: D Feedback: The presence of an ostomy frequently has an effect on sexuality; this should be addressed thoughtfully in nursing care. None of the other listed diagnoses reflects the physiologic changes that result from colorectal surgery.

A patients large bowel obstruction has failed to resolve spontaneously and the patients worsening condition has warranted admission to the medical unit. Which of the following aspects of nursing care is most appropriate for this patient? A) Administering bowel stimulants as ordered B) Administering bulk-forming laxatives as ordered C) Performing deep palpation as ordered to promote peristalsis D) Preparing the patient for surgical bowel resection

Ans: D Feedback: The usual treatment for a large bowel obstruction is surgical resection to remove the obstructing lesion. Administration of laxatives or bowel stimulants are contraindicated if the bowel is obstructed. Palpation would be painful and has no therapeutic benefi

A nurse is amending a patient's plan of care in light of the fact that the patient has recently developed ascites. What should the nurse include in this patient's care plan? A) Mobilization with assistance at least 4 times daily B) Administration of beta-adrenergic blockers as ordered C) Vitamin B12 injections as ordered D) Administration of diuretics as ordered

Ans: D Feedback: Use of diuretics along with sodium restriction is successful in 90% of patients with ascites. Beta-blockers are not used to treat ascites and bed rest is often more beneficial than increased mobility. Vitamin B12 injections are not necessary.

What should the nurse suspect when hourly assessment of urine output on a patient postcraniotomy exhibits a urine output from a catheter of 1,500 mL for two consecutive hours? A) Cushing syndrome B) Syndrome of inappropriate antidiuretic hormone (SIADH) C) Adrenal crisis D) Diabetes insipidus

Ans: D Feedback:Diabetes insipidus is an abrupt onset of extreme polyuria that commonly occurs in patients after brain surgery. Cushing syndrome is excessive glucocorticoid secretion resulting in sodium and water retention. SIADH is the result of increased secretion of ADH; the patient becomes volume-overloaded, urine output diminishes, and serum sodium concentration becomes dilute. Adrenal crisis is undersecretion of glucocorticoids resulting in profound hypoglycemia, hypovolemia, and hypotension.

A patient has been diagnosed with a small bowel obstruction and has been admitted to the medical unit. The nurses care should prioritize which of the following outcomes? A) Preventing infection B) Maintaining skin and tissue integrity C) Preventing nausea and vomiting D) Maintaining fluid and electrolyte balance

Ans: D Test Bank - Brunner & Suddarth's Textbook of Medical-Surgical Nursing 14e (Hinkle 2017) 908 Feedback: All of the listed focuses of care are important for the patient with a small bowel obstruction. However, the patients risk of fluid and electrolyte imbalances is an immediate threat to safety, and is a priority in nursing assessment and interventions.

A patient has been diagnosed with glaucoma and the nurse is preparing health education regarding the patient's medication regimen. The patient states that she is eager to "beat this disease" and looks forward to the time that she will no longer require medication. How should the nurse best respond? A) "You have a great attitude. This will likely shorten the amount of time that you need medications." B) "In fact, glaucoma usually requires lifelong treatment with medications." C) "Most people are treated until their intraocular pressure goes below 50 mm Hg." D) "You can likely expect a minimum of 6 months of treatment."

B

A patient has just returned to the surgical floor after undergoing a retinal detachment repair. The postoperative orders specify that the patient should be kept in a prone position until otherwise ordered. What should the nurse do? A) Call the physician and ask for the order to be confirmed. B) Follow the order because this position will help keep the retinal repair intact. C) Instruct the patient to maintain this position to prevent bleeding. D) Reposition the patient after the first dressing change

B

A patient has lost most of her vision as a result of macular degeneration. When attempting to meet this patient's psychosocial needs, what nursing action is most appropriate? A) Encourage the patient to focus on her use of her other senses. B) Assess and promote the patient's coping skills during interactions with the patient. C) Emphasize that her lifestyle will be unchanged once she adapts to her vision loss. D) Promote the patient's hope for recovery.

B

A patient has recently begun mobilizing during the recovery from an ischemic stroke. To protect the patients safety during mobilization, the nurse should perform what action? A) Support the patients full body weight with a waist belt during ambulation. B) Have a colleague follow the patient closely with a wheelchair. C) Avoid mobilizing the patient in the early morning or late evening. D) Ensure that the patients family members do not participate in mobilization.

B

A patient who presents for an eye examination is diagnosed as having a visual acuity of 20/40. The patient asks the nurse what these numbers specifically mean. What is a correct response by the nurse? A) "A person whose vision is 20/40 can see an object from 40 feet away that a person with 20/20 vision can see from 20 feet away." B) "A person whose vision is 20/40 can see an object from 20 feet away that a person with 20/20 vision can see from 40 feet away." C) "A person whose vision is 20/40 can see an object from 40 inches away that a person with 20/20 vision can see from 20 inches away." D) "A person whose vision is 20/40 can see an object from 20 inches away that a person with 20/20 vision can see from 40 inches away."

B

A rehabilitation nurse caring for a patient who has had a stroke is approached by the patients family and asked why the patient has to do so much for herself when she is obviously struggling. What would be the nurses best answer? A) We are trying to help her be as useful as she possibly can. B) The focus on care in a rehabilitation facility is to help the patient to resume as much self-care as possible. C) We arent here to care for her the way the hospital staff did; we are here to help her get better so she can go home. D) Rehabilitation means helping patients do exactly what they did before their stroke.

B

During discharge teaching the nurse realizes that the patient is not able to read medication bottles accurately and has not been taking her medications consistently at home. How should the nurse intervene most appropriately in this situation? A) Ask the social worker to investigate alternative housing arrangements. B) Ask the social worker to investigate community support agencies. C) Encourage the patient to explore surgical corrections for the vision problem. D) Arrange for referral to a rehabilitation facility for vision training.

B

The nurse is providing discharge education to an adult patient who will begin a regimen of ocular medications for the treatment of glaucoma. How can the nurse best determine if the patient is able to self-administer these medications safely and effectively? A) Assess the patient for any previous inability to self-manage medications. B) Ask the patient to demonstrate the instillation of her medications. C) Determine whether the patient can accurately describe the appropriate method of administering her medications. D) Assess the patient's functional status.

B

The nurse on the medical-surgical unit is reviewing discharge instructions with a patient who has a history of glaucoma. The nurse should anticipate the use of what medications? A) Potassium-sparing diuretics B) Cholinergics C) Antibiotics D) Loop diuretics

B

The nurse's assessment of a patient with significant visual losses reveals that the patient cannot count fingers. How should the nurse proceed with assessment of the patient's visual acuity? A) Assess the patient's vision using a Snellen chart. B) Determine whether the patient is able to see the nurse's hand motion. C) Perform a detailed examination of the patient's external eye structures. D) Palpate the patient's periocular regions.

B

A patient with herpes simplex virus encephalitis (HSV) has been admitted to the ICU. What medication would the nurse expect the physician to order for the treatment of this disease process? A) Cyclosporine (Neoral) B) Acyclovir (Zovirax) C) Cyclobenzaprine (Flexeril) D) Ampicillin (Prinicpen)

B) Acyclovir (Zovirax)

A patient diagnosed with MS has been admitted to the medical unit for treatment of an MS exacerbation. Included in the admission orders is baclofen (Lioresal). What should the nurse identify as an expected outcome of this treatment? A) Reduction in the appearance of new lesions on the MRI B) Decreased muscle spasms in the lower extremities C) Increased muscle strength in the upper extremities D) Decreased severity and duration of exacerbations

B) Decreased muscle spasms in the lower extremities

A patient diagnosed with myasthenia gravis has been hospitalized to receive plasmapheresis for a myasthenic exacerbation. The nurse knows that the course of treatment for plasmapheresis in a patient with myasthenia gravis is what? A) Every day for 1 week B) Determined by the patient's response C) Alternate days for 10 days D) Determined by the patient's weight

B) Determined by the patient's response

A 69-year-old patient is brought to the ED by ambulance because a family member found him lying on the floor disoriented and lethargic. The physician suspects bacterial meningitis and admits the patient to the ICU. The nurse knows that risk factors for an unfavorable outcome include what? Select all that apply. A) Blood pressure greater than 140/90 mm Hg B) Heart rate greater than 120 bpm C) Older age D) Low Glasgow Coma Scale E) Lack of previous immunizations

B) Heart rate greater than 120 bpm C) Older age D) Low Glasgow Coma Scale

A patient with possible bacterial meningitis is admitted to the ICU. What assessment finding would the nurse expect for a patient with this diagnosis? A) Pain upon ankle dorsiflexion of the foot B) Neck flexion produces flexion of knees and hips C) Inability to stand with eyes closed and arms extended without swaying D) Numbness and tingling in the lower extremities

B) Neck flexion produces flexion of knees and hips

A male patient presents to the clinic complaining of a headache. The nurse notes that the patient is guarding his neck and tells the nurse that he has stiffness in the neck area. The nurse suspects the patient may have meningitis. What is another well-recognized sign of this infection? A) Negative Brudzinski's sign B) Positive Kernig's sign C) Hyperpatellar reflex D) Sluggish pupil reaction

B) Positive Kernig's sign

A patient with metastatic cancer has developed trigeminal neuralgia and is taking carbamazepine (Tegretol) for pain relief. What principle applies to the administration of this medication? A) Tegretol is not known to have serious adverse effects. B) The patient should be monitored for bone marrow depression. C) Side effects of the medication include renal dysfunction. D) The medication should be first taken in the maximum dosage form to be effective.

B) The patient should be monitored for bone marrow depression.

The nurse is caring for a client diagnosed with Parkinson disease. The client is having increasing problems with rising from the sitting to the standing position. What should the nurse suggest to the client to use that will aid in getting from the sitting to the standing position as well as aid in improving bowel elimination? A) Use of a bedpan B) Use of a raised toilet seat C) Sitting quietly on the toilet every 2 hours D) Following the outlined bowel program

B) Use of a raised toilet seat

The charge nurse in the neurological unit admits four new patients. The charge nurse should be MOST concerned about which patient? A. An 82-year-old patient with a history of dementia who is oriented to person only. B. A 53-year-old patient who sustained a head injury is talking very slowly. C. A 30-year-old patient who had a motor vehicle accident is complaining of pain in their arm. D. A 40-year-old patient with a history of heavy alcohol, whose last drink was 6 days ago, and reports an "odd tremor to his left hand".

B. A 53-year-old patient who sustained a head injury is talking very slowly.

Which statements regarding Glasgow Coma Scale are correct? Select all that apply. A. Patients are assessed for eye response, verbal response, and pain response. B. A patient that recognizes the nurse from the previous shift and opens his mouth for a temperature check has a GCS score of 15. C. A patient with a score of 13 has a better prognosis than a patient with a score of 8. D. A patient with a score of 0 is nearing death. E. A patient that opens the eyes only when receiving a painful stimulus has an eye response of 1.

B. A patient that recognizes the nurse from the previous shift and opens his mouth for a temperature check has a GCS score of 15. C. A patient with a score of 13 has a better prognosis than a patient with a score of 8.

A nurse notes in the patient's I/O flowsheet that their last bowel movement was 2 days ago. What would be an appropriate next step by the nurse? A. Contact the provider to request a bisacodyl (Dulcolax) suppository. B. Ask the patient how often they typically have a bowel movement. C. Administer ordered PRN sennakot tabs. D. Contact the dietitian to ensure a low fiber diet is ordered.

B. Ask the patient how often they typically have a bowel movement.

A nurse is planning discharge education for a patient who underwent a cervical diskectomy. What strategies would the nurse assess that would aid in planning discharge teaching? A. Techniques for restoring nerve function B. Care of the cervical collar C. Technique for performing neck ROM exercises D. Home assessment of ABGs

B. Care of the cervical collar

A 13-year-old is being admitted to the ED after falling from a roof and sustaining blunt abdominal injuries. To assess for internal injury in the patient's peritoneum, the nurse should anticipate what diagnostic test? A. Complete blood count (CBC) B. Computed tomography (CT) scan C. Radiograph D. Barium swallow

B. Computed tomography (CT) scan

A patient who has been diagnosed with cholecystitis is being discharged home from the ED to be scheduled for surgery later. The patient received morphine during the present ED admission and is visibly drowsy. When providing health education to the patient, what would be the most appropriate nursing action? A. Give verbal instructions to one of the patient's family members. B. Give verbal and written instructions to patient and a family member. C. Telephone the patient the next day with verbal instructions. D. Give written instructions to patient.

B. Give verbal and written instructions to patient and a family member.

A 6-month-old infant is brought to the ED by his parents for inconsolable crying and pulling at his right ear. When assessing this infant, the advanced practice nurse is aware that the tympanic membrane should be what color in a healthy ear? A) Yellowish-white B) Pink C) Gray D) Bluish-white

C

The nurse is assessing a patient with multiple sclerosis who is demonstrating involuntary, rhythmic eye movements. What term will the nurse use when documenting these eye movements? A) Vertigo B) Tinnitus C) Nystagmus D) Astigmatism

C

The nurse is caring for a patient diagnosed with an ischemic stroke and knows that effective positioning of the patient is important. Which of the following should be integrated into the patients plan of care? A) The patients hip joint should be maintained in a flexed position. B) The patient should be in a supine position unless ambulating. C) The patient should be placed in a prone position for 15 to 30 minutes several times a day. D) The patient should be placed in a Trendelenberg position two to three times daily to promote cerebral perfusion.

C

The nurse is planning the care of a patient with a diagnosis of vertigo. What nursing diagnosis risk should the nurse prioritize in this patient's care? A) Risk for disturbed sensory perception B) Risk for unilateral neglect C) Risk for falls D) Risk for ineffective health maintenance

C

Which of the following nursing interventions would most likely facilitate effective communication with a hearing-impaired patient? A) Ask the patient to repeat what was said in order to evaluate understanding. B) Stand directly in front of the patient to facilitate lip reading. C) Reduce environmental noise and distractions before communicating. D) Raise the voice to project sound at a higher frequency.

C

A nurse is collaborating with the interdisciplinary team to help manage a patient's recurrent headaches. What aspect of the patient's health history should the nurse identify as a potential contributor to the patient's headaches? A) The patient leads a sedentary lifestyle. B) The patient takes vitamin D and calcium supplements. C) The patient takes vasodilators for the treatment of angina. D) The patient has a pattern of weight loss followed by weight gain.

C Feedback: Vasodilators are known to contribute to headaches. Weight fluctuations, sedentary lifestyle, and vitamin supplements are not known to have this effect.

The nurse is discharging a patient home after surgery for trigeminal neuralgia. What advice should the nurse provide to this patient in order to reduce the risk of injury? A) Avoid watching television or using a computer for more than 1 hour at a time. B) Use OTC antibiotic eye drops for at least 14 days. C) Avoid rubbing the eye on the affected side of the face. D) Rinse the eye on the affected side with normal saline daily for 1 week.

C) Avoid rubbing the eye on the affected side of the face.

A 33-year-old patient presents at the clinic with complaints of weakness, incoordination, dizziness, and loss of balance. The patient is hospitalized and diagnosed with MS. What sign or symptom, revealed during the initial assessment, is typical of MS? A) Diplopia, history of increased fatigue, and decreased or absent deep tendon reflexes B) Flexor spasm, clonus, and negative Babinski's reflex C) Blurred vision, intention tremor, and urinary hesitancy D) Hyperactive abdominal reflexes and history of unsteady gait and episodic paresthesia in both legs

C) Blurred vision, intention tremor, and urinary hesitancy

The critical care nurse is caring for 25-year-old man admitted to the ICU with a brain abscess. What is a priority nursing responsibility in the care of this patient? A) Maintaining the patient's functional independence B) Providing health education C) Monitoring neurologic status closely D) Promoting mobility

C) Monitoring neurologic status closely

A 48-year-old patient has been diagnosed with trigeminal neuralgia following recent episodes of unilateral face pain. The nurse should recognize what implication of this diagnosis? A) The patient will likely require lifelong treatment with anticholinergic medications. B) The patient has a disproportionate risk of developing myasthenia gravis later in life. C) The patient needs to be assessed for MS. D) The disease is self-limiting and the patient will achieve pain relief over time.

C) The patient needs to be assessed for MS.

A nurse is providing education to a 65 year old male patient preparing for discharge. The patient asks the nurse why he must observe bleeding precautions. The appropriate response from the nurse would be: A. "Your prescribed lactulose increases your bleeding risk." B. "The high protein diet you are ordered will decrease production of clotting factors." C. "A cirrhotic liver is unable to produce clotting factors effectively." D. "Eating foods high in vitamin K will increase your bleeding risk."

C. "A cirrhotic liver is unable to produce clotting factors effectively."

A nurse is preparing to discharge a patient with multiple sclerosis; which of the following statements by the patient would indicate further teaching is required? A. "I will be sure to use the restroom at scheduled times." B. "I've arranged for a cane to be delivered to my home." C. "I can limit stress by taking hot baths to relax." D. "I will make sure to dress appropriately for the weather."

C. "I can limit stress by taking hot baths to relax." rationale: avoid extreme temperatures

The nurse preceptor reviews care during nightshift for a patient scheduled for an electroencephalogram (EEG) the following morning. Which of the following statements by the student nurse indicates teaching was successful? A. "I will remember to remove all pieces of jewelry and any metal objects from the patient." B. "I will withhold meals, and only offer water to the patient after 10 pm today." C. "I will keep the patient awake overnight." D. "I will instruct the patient to eat breakfast early in the morning with some tea or small coffee."

C. "I will keep the patient awake overnight."

A nurse is caring for a patient with a spinal cord injury at C6. She notices on the monitor that his heart rate has dropped into the low 50s, and he is newly hypertensive (168/79). What would be an appropriate intervention by the nurse? A. Lay the patient supine to assist with BP B. Administer the patient's scheduled propranolol C. Assess the patient's lower abdomen for bladder distension D. Hold the patient's requested PRN Tylenol

C. Assess the patient's lower abdomen for bladder distension

A nurse in the intensive care unit is caring for a patient currently ventilated due to respiratory failure secondary to myasthenic crisis. The patient is on a scheduled regimen of mestinon and receiving plasmapheresis every other day for a total of 5 sessions. You note that the patient is unable to tolerate a decrease in her ventilator settings due to copious secretions which require frequent suctioning. What is the most appropriate intervention by the RN? A. Request the provider decrease the patient's maintenance IV fluid rate B. Advocate for additional plasmapheresis treatment as the patient is still symptomatic C. Discuss decreasing the patient's mestinon dose with the provider D. Discuss increasing the patient's mestinon dose with the provider

C. Discuss decreasing the patient's mestinon dose with the provider rationale: you will see these increased secretions and other symptoms with too much of these types of drugs

A patient who suffered a spinal cord injury is experiencing an exaggerated autonomic response. What aspect of the patient's current health status is most likely to have precipitated this event? A. The patient was not repositioned during the night shift. B. The patient received a blood transfusion. C. The patient's urinary catheter became occluded. D. The patient's analgesia regimen was recent changed.

C. The patient's urinary catheter became occluded

The nurse preceptor reviews medication orders for a patient diagnosed with a stroke. Which of the following statements by the student nurse indicates further teaching is necessary? A. This patient will take aspirin and clopidogrel at home to prevent the formation of new clots. B. This patient will continue to take hydrochlorothiazide to manage hypertension. C. This patient should stop taking atorvastatin due to the increased risk for seizures. D. This patient will restart losartan at home to manage hypertension.

C. This patient should stop taking atorvastatin due to the increased risk for seizures.

A patient is being discharged home from the ambulatory surgical center after cataract surgery. In reviewing the discharge instructions with the patient, the nurse instructs the patient to immediately call the office if the patient experiences what? A) Slight morning discharge from the eye B) Any appearance of redness of the eye C) A "scratchy" feeling in the eye D) A new floater in vision

D

A patient is exploring treatment options after being diagnosed with age-related cataracts that affect her vision. What treatment is most likely to be used in this patient's care? A) Antioxidant supplements, vitamin C and E, beta-carotene, and selenium B) Eyeglasses or magnifying lenses C) Corticosteroid eye drops D) Surgical intervention

D

A patient is ready to be discharged home after a cataract extraction with intraocular lens implant and the nurse is reviewing signs and symptoms that need to be reported to the ophthalmologist immediately. Which of the patient's statements best demonstrates an adequate understanding? A) "I need to call the doctor if I get nauseated." B) "I need to call the doctor if I have a light morning discharge." C) "I need to call the doctor if I get a scratchy feeling." D) "I need to call the doctor if I see flashing lights."

D

A patient with a diagnosis of retinal detachment has undergone a vitreoretinal procedure on an outpatient basis. What subject should the nurse prioritize during discharge education? A) Risk factors for postoperative cytomegalovirus (CMV) B) Compensating for vision loss for the next several weeks C) Non-pharmacologic pain management strategies D) Signs and symptoms of increased intraocular pressure

D

A patient with chronic open-angle glaucoma is being taught to self-administer pilocarpine. After the patient administers the pilocarpine, the patient states that her vision is blurred. Which nursing action is most appropriate? A) Holding the next dose and notifying the physician B) Treating the patient for an allergic reaction C) Suggesting that the patient put on her glasses D) Explaining that this is an expected adverse effect

D

The public health nurse is addressing eye health and vision protection during an educational event. What statement by a participant best demonstrates an understanding of threats to vision? A) "I'm planning to avoid exposure to direct sunlight on my next vacation." B) "I've never exercised regularly, but I'm going to start working out at the gym daily." C) "I'm planning to talk with my pharmacist to review my current medications." D) "I'm certainly going to keep a close eye on my blood pressure from now on."

D

A 73-year-old man comes to the clinic complaining of weakness and loss of sensation in his feet and legs. Assessment of the patient shows decreased reflexes bilaterally. Why would it be a challenge to diagnose a peripheral neuropathy in this patient? A) Older adults are often vague historians. B) The elderly have fewer peripheral nerves than younger adults. C) Many older adults are hesitant to admit that their body is changing. D) Many symptoms can be the result of normal aging process.

D) Many symptoms can be the result of normal aging process.

The nurse caring for a patient diagnosed with Guillain-Barré syndrome is planning care with regard to the clinical manifestations associated this syndrome. The nurse's communication with the patient should reflect the possibility of what sign or symptom of the disease? A) Intermittent hearing loss B) Tinnitus C) Tongue enlargement D) Vocal paralysis

D) Vocal paralysis

A patient diagnosed with Bell's palsy is having decreased sensitivity to touch of the involved nerve. What should the nurse recommend to prevent atrophy of the muscles? A) Blowing up balloons B) Deliberately frowning C) Smiling repeatedly D) Whistling

D) Whistling

The nurse is caring for a patient who is hospitalized with an exacerbation of MS. To ensure the patient's safety, what nursing action should be performed? A. Provide several small meals each day. B. Pad the patient's bed rails. C. Maintain bed rest whenever possible. D. Ensure that suction apparatus is set up at the bedside.

D. Ensure that suction apparatus is set up at the bedside.

A nurse is caring for a 36 year old female patient who is 4 hours s/p cholecystectomy. Which of the following findings would necessitate escalation to the provider? A. Bowel sounds absent in all four quadrants on auscultation. B. Patient reports RUQ pain that is a 5/10 on the numeric scale. C. WBC count elevated on check of post-operative labwork. D. Patient's temperature is 39.0 C with diaphoresis and chills.

D. Patient's temperature is 39.0 C with diaphoresis and chills.

A 22 year old male patient presents to the ED c/o abdominal pain. On initial assessment, the nurse noted significant rebound tenderness in the RLQ, and an ultrasound was ordered by the provider. The nurse now notes that the patient's abdomen is "board like" on palpation; the nurse should be most concerned for: A. Paralytic Ileus B. Cholecystitis C. Pancreatitis D. Peritonitis

D. Peritonitis

A patient diagnosed with multiple sclerosis (MS) has been admitted to the medical unit for treatment of an MS exacerbation. Included in the admission orders is baclofen (Lioresal). What would be the expected outcome of this medication?

Decreased muscle spasms in the lower extremities

A nursing student is reviewing for an upcoming anatomy and physiology examination. Which of the following would the student correctly identify as a function of the liver? Select all that apply. a) Carbohydrate metabolism b) Glucose metabolism c) Zinc storage d) Ammonia conversion e) Protein metabolism

Glucose metabolism • Ammonia conversion • Protein metabolism Explanation: Functions of the liver include the metabolism of glucose, protein, fat, and drugs; conversion of ammonia; storage of vitamins and iron; formation of bile; and excretion of bilirubin. The liver is not responsible for the metabolism of carbohydrates or the storage of zinc. (less)

The nurse is caring for a patient with an altered LOC. What is the first priority of treatment for this patient?

Maintenance of a patent airway

A patient is admitted to the hospital for management of an extrapyramidal disorder. Included in the physician's admitting orders are the medications levodopa, benztropine, and selegiline. The nurse knows that most likely, the client has a diagnosis of:

Parkinson's disease.

A healthcare provider orders several drugs for a client with hemorrhagic stroke. Which drug order should the nurse question? a. Heparin sodium b. Dexamethasone c. Methyldopa d. Phenytoin

a Administering heparin, an anticoagulant, could increase the bleeding associated with hemorrhagic stroke. Therefore, the nurse should question this order to prevent additional hemorrhage in the brain. In a client with hemorrhagic stroke, the healthcare provider may use dexamethasone (Decadron) to decrease cerebral edema and pressure, methyldopa (Aldomet) to reduce blood pressure, and phenytoin (Dilantin) to prevent seizures.

A nurse is completing discharge teaching for the client who has left-sided hemiparesis following a stroke. When investigating the client's home environment, the nurse should focus on which nursing diagnosis? a. Risk for injury b. Ineffective coping c. Noncompliance d. Diarrhea

a Because of decreased physical mobility, a client with recent left-sided hemiparesis is at risk for falls in the home setting. His ability to cope with the stroke is important, but investigating the home environment doesn't provide information about this nursing diagnosis. Diarrhea and Noncompliance aren't related to the client's home environment.

An emergency department nurse is awaiting the arrival of a client with signs of an ischemic stroke that began 1 hour ago, as reported by emergency medical personnel. The treatment window for thrombolytic therapy is which of the following? a. Three hours b. One hour c. Two hours d. Six hours

a Rapid diagnosis of stroke and initiation of thrombolytic therapy (within 3 hours) in clients with ischemic stroke leads to a decrease in the size of the stroke and an overall improvement in functional outcome after 3 months.

A patient who just suffered a hemorrhagic stroke is brought to the emergency department by ambulance. What should be the nurse's primary assessment focus? a. Cardiac and respiratory status b. Seizure activity c. Urinary output d. Fluid and electrolyte balance

a Acute care begins with managing the ABC's. Patients may have difficulty keeping an open and clear airway secondary to decreased level of consciousness. Neurological assessment with close monitoring for signs of increased neurological deficit and seizure activity occurs next. Fluid and electrolyte balance must be controlled carefully, with the goal of adequate hydration to promote perfusion and decrease further brain damage.

Which of the following medication classifications is utilized preoperatively to decrease risk of postop seizures? a. Anticonvulsants b. Diuretics c. Corticosteroids d. Antianxiety

a Anticonvulsants are used to decrease the risk of postoperative seizures following cranial surgery. Diuretics, corticosteroids, and antianxiety medications may be used for the patient with increased intracranial pressure.

A client with a new diagnosis of amyotrophic lateral sclerosis (ALS) is overwhelmed by his diagnosis and the known complications of the disease. How can the client best make known his wishes for care as his disease progresses? a. Prepare an advance directive. b. Designate a most responsible health care provider (MRP) early in the course of the disease. c. Collaborate with representatives from the Amyotrophic Lateral Sclerosis Association. d. Ensure that witnesses are present when he provides instruction.

a Clients with ALS are encouraged to complete an advance directive or "living will" to preserve their autonomy in decision making. None of the other listed actions constitutes a legally binding statement of end-of-life care.

The nurse is providing diet-related advice to a client who experienced a cerebrovascular accident (CVA). The client wants to minimize his volume of food and yet meet all nutritional requirements. To control the volume of food intake, the nurse should suggest that the client consume: a. thickened commercial beverages and fortified cooked cereals. b. dry or crisp foods and chewy meats. c. hot or tepid foods. d. a high-fat diet.

a Clients with CVA or other cerebrovascular disorders should lose weight and, therefore, should minimize their volume of food consumption. To ensure this, the nurse may suggest thickened commercial beverages, fortified cooked cereals, or scrambled eggs.

A nurse is teaching a community class that those experiencing symptoms of ischemic stroke need to enter the medical system early. The primary reason for this is which of the following? a. Thrombolytic therapy has a time window of only 3 hours. b. A ruptured intracranial aneurysm must quickly be repaired. c. Intracranial pressure is increased by a space-occupying bleed. d. A ruptured arteriovenous malformation will cause deficits until it is stopped.

a Currently approved thrombolytic therapy for ischemic strokes has a treatment window of only 3 hours after the onset of symptoms. Urgency is needed on the part of the public for rapid entry into the medical system. The other three choices are related to hemorrhagic strokes.

A patient had a carotid endarterectomy yesterday and when the nurse arrived in the room to perform an assessment, the patient states, "All of a sudden, I am having trouble moving my right side." What concern should the nurse have about this complaint? a. A thrombus formation at the site of the endarterectomy b. This is a normal occurrence after an endarterectomy and would not be a concern. c. Bleeding from the endarterectomy site d. Surgical wound infection

a Formation of a thrombus at the site of the endarterectomy is suspected if there is a sudden new onset of neurologic deficits, such as weakness on one side of the body.

What is one of the earliest signs of increased ICP? a. decreased level of consciousness (LOC) b. headache c. Cushing's triad d. coma

a Headache is a symptom of increased ICP, but decreasing LOC is one of the earliest signs of increased ICP. Cushing's triad occurs late in increased ICP. If untreated, increasing ICP will lead to coma.

The nurse is caring for a client who sustained a moderate head injury following a bicycle accident. The nurse's most recent assessment reveals that the client's respiratory effort has increased. What is the nurse's most appropriate response? a. Inform the care team and assess for further signs of possible increased ICP. b. Administer bronchodilators as prescribed and monitor the client's LOC. c. Increase the client's bed height and reassess in 30 minutes. d. Administer a bolus of normal saline as prescribed.

a Increased respiratory effort can be suggestive of increasing ICP, and the care team should be promptly informed. A bolus of IV fluid will not address the problem. Repositioning the client and administering bronchodilators are insufficient responses, even though these actions may later be prescribed.

A nurse assesses the patient's LOC using the Glasgow Coma Scale. What score indicates severe impairment of neurologic function? a. 3 b. 6 c. 9 d. 12

a LOC, a sensitive indicator of neurologic function, is assessed based on the criteria in the Glasgow Coma Scale: eye opening, verbal response, and motor response (Barlow, 2012). The patient's responses are rated on a scale from 3 to 15. A score of 3 indicates severe impairment of neurologic function, brain death, or pharmacologic inhibition of the neurologic response. A score of 15 indicates that the patient is fully responsive (see Chapter 68).

When caring for a client with a head injury, a nurse must stay alert for signs and symptoms of increased intracranial pressure (ICP). Which cardiovascular findings are late indicators of increased ICP? a. Rising blood pressure and bradycardia b. Hypotension and bradycardia c. Hypotension and tachycardia d. Hypertension and narrowing pulse pressure

a Late cardiovascular indicators of increased ICP include rising blood pressure, bradycardia, and widening pulse pressure — known collectively as Cushing's triad. Increased ICP usually causes a bounding pulse; as death approaches, the pulse becomes irregular and thready.

A client whose diagnosis includes head trauma is being closely observed for signs and symptoms of increasing intracranial pressure. The client is exhibiting nonverbal indications of experiencing pain. Why should the nurse avoid the administration of narcotic analgesics in this case? a. Narcotic analgesics increase CSF pressure. b. Narcotic analgesics are ineffective against pain in head trauma. c. Narcotic analgesics decrease CSF pressure. d. Avoidance is inappropriate because narcotic analgesics are the drug of choice in treating pain associated with head trauma.

a Narcotic analgesics depress the respiratory center and raise CSF pressure. Their use is contraindicated in clients with head trauma or increased ICP, unless administration is an absolute necessity.

A client fell at home and sustained a head injury. The client exhibits signs and symptoms of head trauma with indications of increased ICP. What is the normal ventricular ICP? a. 5 to 15 mm Hg b. 16 to 20 mm Hg c. 21 to 30 mm Hg d. 31 to 40 mm Hg

a Normal ICP is 5 to 15 mm Hg.

Which of the following statements reflects nursing management of the patient with expressive aphasia? a. Encourage the patient to repeat sounds of the alphabet. b. Speak clearly and in simple sentences; use gestures or pictures when able. c. Speak slowly and clearly to assist the patient in forming the sounds. d. Frequently reorient the patient to time, place, and situation.

a Nursing management of the patient with expressive aphasia includes encouraging the patient to repeat sounds of the alphabet. Nursing management of the patient with global aphasia includes speaking clearly and in simple sentences and using gestures or pictures when able. Nursing management of the patient with receptive aphasia includes speaking slowing and clearly to assist the patient in forming the sounds. Nursing management of the patient with cognitive deficits, such as memory loss, includes frequently reorienting the patient to time, place, and situation.

A client has a 12-year history of migraine headaches and is frustrated over how these headaches impact lifestyle. The nurse discusses the potential triggers of the client's migraines. Which is not a potential trigger to migraines? a. seasonal changes b. reproductive hormone c. fluctuations d. specific food chemicals medications

a Researchers believe the contributing cofactors for the cause of migraines are from changes in serotonin receptors that promote dilation of cerebral blood vessels and pain intensification from neurochemicals released from the trigeminal nerve. It has been suggested that fluctuations in reproductive hormones, chemicals in certain foods, and medications can trigger migraines.

A 64-year-old client reports symptoms consistent with a transient ischemic attack (TIA) to the health care provider in the emergency department. Which is the origin of the client's symptoms? a. impaired cerebral circulation b. cardiac disease c. diabetes insipidus d. hypertension

a TIAs involve the same mechanism as in the ischemic cascade, but symptoms are transient (< 24 hours) and there is no evidence of cerebral tissue infarction. The ischemic cascade begins when cerebral blood flow decreases to less than 25 mL/100 g/min and neurons are no longer able to maintain aerobic respiration. Thus, a TIA results directly from impaired blood circulation in the brain. Atherosclerosis, cardiac disease, hypertension, or diabetes can be risk factors for a TIA but do not cause it.

The nurse is liaising with the physical therapist and occupational therapist to create an activity management plan for a patient who has multiple sclerosis. What principle should be integrated into guidelines for exercise and activity that the team will provide to this patient in anticipation of discharge? a. The patient should perform frequent physical activity but avoid becoming fatigued. b. The patient should perform exercises that are brief but high-intensity. c. The patient should prioritize energy conservation and remain on bed rest if possible. d. The patient should attempt to maintain prediagnosis levels of activity and mobility.

a The patient is encouraged to work and exercise to a point just short of fatigue. Very strenuous physical exercise is not advisable because it raises the body temperature and may aggravate symptoms. The patient is advised to take frequent short rest periods, preferably lying down. Extreme fatigue may contribute to the exacerbation of symptoms. It is unrealistic to expect the patient to maintain prediagnosis levels of activity.

A client is brought by ambulance to the ED after suffering what the family thinks is a stroke. The nurse caring for this client is aware that an absolute contraindication for thrombolytic therapy is what? a. Evidence of hemorrhagic stroke b. Blood pressure of ≥ 180/110 mm Hg c. Evidence of stroke evolution d. Previous thrombolytic therapy within the past 12 months

a Thrombolytic therapy would exacerbate a hemorrhagic stroke with potentially fatal consequences. Stroke evolution, high BP, or previous thrombolytic therapy does not contraindicate its safe and effective use.

A male patient with cerebrovascular accident (CVA) is prescribed medication to treat the disorder. The patient wants to know what other measures may help reduce CVA. Which of the following is an accurate suggestion for the patient? a. Reduce hypertension and high blood cholesterol levels. b. Increase body weight moderately. c. Increase the intake of proteins and carbohydrates. d. Increase the fluids and hydration.

a CVAs are prevented by reducing certain risk factors, such as hypertension, overweight, cardiac dysrhythmias (such as atrial fibrillation), and high blood cholesterol levels. Patients should not gain body weight. In addition, the increased intake of proteins, carbohydrates, or fluids does not help in reducing the risk of CVAs.

The nurse is called to attend to a patient having a seizure in the waiting area. What nursing care is provided for a patient who is experiencing a convulsive seizure? Select all that apply. a. Loosening constrictive clothing b. Opening the patient's jaw and inserting a mouth gag c. Positioning the patient on his or her side with head flexed forward d. Providing for privacy e. Restraining the patient to avoid self injury

a, c, d During a patient's seizure, the nurse should do the following. Loosen constrictive clothing. If possible, place the patient on one side with head flexed forward, which allows the tongue to fall forward and facilitates drainage of saliva and mucus. If suction is available, use it if necessary to clear secretions. Provide privacy, and protect the patient from curious onlookers. (The patient who has an aura [warning of an impending seizure] may have time to seek a safe, private place.) The nurse should not attempt to pry open jaws that are clenched in a spasm or attempt to insert anything. Broken teeth and injury to the lips and tongue may result from such an action. No attempt should be made to restrain the patient during the seizure, because muscular contractions are strong and restraint can produce injury.

Which nursing interventions might need to be considered in a care plan for a client with advanced multiple sclerosis? Select all that apply. a. Ensure access to a language board when communicating with the client. b. Obtain daily weights to monitor weight gain. c. Establish a voiding time schedule. d. Encourage the client to walk with feet wide apart.

a, c, d Language assistive devices may be needed if communication is severely affected. Occasional bladder incontinence may lead to total incontinence. A voiding time schedule will allow the client greater independence. If motor dysfunction causes problems of incoordination and clumsiness, the patient is at risk for falling. As the disease progresses, nutritional deficiencies may develop. Weight should be assessed to ensure that there is no significant weight loss. Weight gain should not be an issue.

A nurse is caring for a client with a cerebral aneurysm. Which nursing interventions would be most useful to the nurse to avoid bleeding in the brain? Select all that apply. a. Report changes in neurologic status as soon as a worsening trend is identified. b. Use a well-lighted room for assessments every 2 hours. c. Follow the healthcare provider's orders to increase fluid volume. d. Maintain the head of the bed at 30 degrees. e. Avoid any activities that cause a Valsalva maneuver.

a, d, e Cerebral aneurysm precautions are implemented for the patient with a diagnosis of aneurysm to provide a nonstimulating environment, prevent increases in intracranial pressure, and prevent further bleeding. The patient is placed on bed rest in a quiet, nonstressful environment, because activity, pain, and anxiety are thought to elevate the blood pressure, which may increase the risk for bleeding. The head of the bed is elevated 30 degrees to promote venous drainage and decrease intracranial pressure. Any activity that suddenly increases the blood pressure or obstructs venous return is avoided. This includes the Valsalva maneuver, straining, forceful sneezing, pushing oneself up in bed and acute flexion or rotation of the head and neck (which compromises the jugular veins). Stool softeners and mild laxatives are prescribed to prevent constipation, which can cause an increase in intracranial pressure. Dim lighting is helpful for photophobia. Increasing fluid volume does not affect brain bleeding.

While completing a health history on a client who has recently experienced a seizure, the nurse would assess for what characteristic associated with the postictal state? a. Epileptic cry b. Confusion c. Urinary incontinence d. Body rigidity

b In the postictal state (after the seizure), the client is often confused and hard to arouse and may sleep for hours. The epileptic cry occurs from the simultaneous contractions of the diaphragm and chest muscles that occur during the seizure. Urinary incontinence and intense rigidity of the entire body are followed by alternating muscle relaxation and contraction (generalized tonic-clonic contraction) during the seizure.

The nurse is participating in the care of a client with increased ICP. What diagnostic test is contraindicated in this client's treatment? a. Computed tomography (CT) scan b. Lumbar puncture c. Magnetic resonance imaging (MRI) d. Venous Doppler studies

b A lumbar puncture in a client with increased ICP may cause the brain to herniate from the withdrawal of fluid and change in pressure during the lumbar puncture. Herniation of the brain is a dire and frequently fatal event. CT, MRI, and venous Doppler are considered noninvasive procedures and they would not affect the ICP itself.

A clinic nurse is caring for a client diagnosed with migraine headaches. During the client teaching session, the client questions the nurse regarding alcohol consumption. What would the nurse be correct in telling the client about the effects of alcohol? a. Alcohol causes hormone fluctuations. b. Alcohol causes vasodilation of the blood vessels. c. Alcohol has an excitatory effect on the CNS. d. Alcohol diminishes endorphins in the brain.

b Alcohol causes vasodilation of the blood vessels and may exacerbate migraine headaches. Alcohol has a depressant effect on the CNS. Alcohol does not cause hormone fluctuations, nor does it decrease endorphins (morphine-like substances produced by the body) in the brain.

The nurse is caring for a client recovering from an ischemic stroke. What intervention best addresses a potential complication after an ischemic stroke? a. Providing frequent small meals rather than three larger meals b. Teaching the client to perform deep breathing and coughing exercises c. Keeping a urinary catheter in situ for the full duration of recovery d. Limiting intake of insoluble fiber

b Because pneumonia is a potential complication of stroke, deep breathing and coughing exercises should be encouraged unless contraindicated. No particular need exists to provide frequent meals and normally fiber intake should not be restricted. Urinary catheters should be discontinued as soon as possible.

Which activity should be avoided in clients with increased intracranial pressure (ICP)? a. Suctioning b. Enemas c. Position changes d. Minimal environmental stimuli

b Enemas should be avoided in clients with increased ICP. The Valsalva maneuver causes increased ICP. Suctioning should not last longer than 15 seconds. Environmental stimuli should be minimal. If monitoring reveals that turning the client increases the ICP, rotating beds, turning sheets, and holding the client's head during turning may minimize the stimuli that cause increased ICP.

If warfarin is contraindicated as a treatment for stroke, which medication is the best option? a. Dipyridamole b. Aspirin c. Clopidogrel d. Ticlodipine

b If warfarin is contraindicated, aspirin is the best option, although other medications may be used if both are contraindicated.

Which clinical manifestation would be exhibited by a client following a hemorrhagic stroke of the right hemisphere? a. Inability to move the right arm b. Neglect of the left side c. Neglect of the right side d. Expressive aphasia

b This client would have deficits on the left side. Expressive aphasia typically occurs with left-hemisphere stroke.

The nurse is reviewing the medication administration record of a female client who possesses numerous risk factors for stroke. Which of the woman's medications carries the greatest potential for reducing her risk of stroke? a. Naproxen 250 PO b.i.d. b. Calcium carbonate 1,000 mg PO b.i.d. c. Aspirin 81 mg PO o.d. d. Lorazepam 1 mg SL b.i.d. PRN

c Research findings suggest that low-dose aspirin may lower the risk of stroke in women who are at risk. Naproxen, lorazepam, and calcium supplements do not have this effect.

A patient with Parkinson's disease is undergoing a swallowing assessment because she is experiencing difficulties when swallowing. What consistency is most appropriate for this patient, to reduce the risk of aspiration? a. Solid food with thin liquids b. Pureed food with water c. Semisolid food with thick liquids d. Thin liquids only

c A semisolid diet with thick liquids is easier to swallow for a patient with swallowing difficulties than a solid diet. Thin liquids should be avoided. Pureed foods with water are not indicated for this patient.

A client with a new diagnosis of ischemic stroke is deemed to be a candidate for treatment with tissue plasminogen activator (t-PA) and has been admitted to the ICU. In addition to closely monitoring the client's cardiac and neurologic status, the nurse monitors the client for signs of what complication? a. Acute pain b. Septicemia c. Bleeding d. Seizures

c Bleeding is the most common side effect of t-PA administration, and the client is closely monitored for any bleeding. Septicemia, pain, and seizures are much less likely to result from thrombolytic therapy.

A client with increased ICP has a ventriculostomy for monitoring ICP. The nurse's most recent assessment reveals that the client is now exhibiting nuchal rigidity and photophobia. The nurse would be correct in suspecting the presence of what complication? a. Encephalitis b. CSF leak c. Meningitis d. Catheter occlusion

c Complications of a ventriculostomy include ventricular infectious meningitis and problems with the monitoring system. Nuchal rigidity and photophobia are clinical manifestations of meningitis, but are not suggestive of encephalitis, a CSF leak, or an occluded catheter.

A client with neurologic infection develops cerebral edema from syndrome of inappropriate antidiuretic hormone (SIADH). Which is an important nursing action for this client? a. Maintaining adequate hydration b. Administering prescribed antipyretics c. Restricting fluid intake and hydration d. Hyperoxygenation before and after tracheal suctioning

c Fluid restriction may be necessary if the client develops cerebral edema and hypervolemia from SIADH. Antipyretics are administered to clients who develop hyperthermia. In addition, it is important to maintain adequate hydration in such clients. A client with neurologic infection should be given tracheal suctioning and hyperoxygenation only when the respiratory distress develops.

A client with neurological infection develops cerebral edema from syndrome of inappropriate antidiuretic hormone (SIADH). Which is an important nursing action for this client? a. Maintaining adequate hydration b. Administering prescribed antipyretics c. Restricting fluid intake and hydration d. Hyperoxygenation before and after tracheal suctioning

c Fluid restriction may be necessary if the client develops cerebral edema and hypervolemia from SIADH. Antipyretics are administered to clients who develop hyperthermia. In addition, it is important to maintain adequate hydration in such clients. A client with neurological infection should be given tracheal suctioning and hyperoxygenation only when respiratory distress develops.

A client who has been on long-term phenytoin therapy is admitted to the unit. In light of the adverse of effects of this medication, the nurse should prioritize which of the following in the client's plan of care? a. Monitoring of pulse oximetry b. Administration of a low-protein diet c. Administration of thorough oral hygiene d. Fluid restriction as prescribed

c Gingival hyperplasia (swollen and tender gums) can be associated with long-term phenytoin use. Thorough oral hygiene should be provided consistently and encouraged after discharge. Fluid and protein restriction are contraindicated and there is no particular need for constant oxygen saturation monitoring.

The nurse is caring for a client with a brain tumor. What drug would the nurse expect to be prescribed to reduce the edema surrounding the tumor? a. Solumedrol b. Dextromethorphan c. Dexamethasone d. Furosemide

c If a brain tumor is the cause of the increased ICP, corticosteroids (e.g., dexamethasone) help reduce the edema surrounding the tumor. Solumedrol, a steroid, and furosemide, a loop diuretic, are not the drugs of choice in this instance. Dextromethorphan is used in cough medicines.

A client with amyotrophic lateral sclerosis (ALS) is being visited by the home health nurse who is creating a care plan. Which of the following nursing diagnoses is most likely for a client with this condition? a. Chronic confusion b. Impaired urinary elimination c. Impaired verbal communication d. Bowel incontinence

c Impaired communication is an appropriate nursing diagnosis; the voice in clients with ALS assumes a nasal sound and articulation becomes so disrupted that speech is unintelligible. Intellectual function is marginally impaired in clients with late ALS. Usually, the anal and bladder sphincters are intact because the spinal nerves that control muscles of the rectum and urinary bladder are not affected.

A nurse is reading a journal article about stroke and the underlying causes associated with this condition. The nurse demonstrates understanding of the information when identifying which subtype of stroke as being due to atrial fibrillation? a. large-artery thrombotic b. small, penetrating artery thrombotic c. cardio embolic d. cryptogenic

c Ischemic strokes are further divided into five subtypes, according to a mechanism-based classification system: large-artery thrombotic strokes (representing 20% of ischemic strokes); small, penetrating artery thrombotic strokes (25%); cardio embolic strokes (20%); cryptogenic strokes (strokes that cannot be attributed to any specific cause) (30%); and "other" (5%). Large-artery thrombotic strokes are caused by atherosclerotic plaques in the large blood vessels of the brain. Thrombus formation and occlusion can occur at the site of the atherosclerosis and result in ischemia and infarction (tissue death). Small, penetrating artery thrombotic strokes that affect one or more vessels and cause reduced blood flow are the most common type of ischemic stroke, typically caused by longstanding hypertension, hyperlipidemia, or diabetes. Cardio embolic strokes are associated with cardiac dysrhythmias, such as atrial fibrillation, but can also be associated with valvular heart disease or left ventricular thrombus. The last two classifications of ischemic strokes are cryptogenic strokes, which have no identified cause, and strokes from other causes, such as illicit drug use (cocaine), coagulopathies, migraine, or spontaneous dissection of the carotid or vertebral arteries.

A client with a traumatic brain injury is showing early signs of increasing intracranial pressure (ICP). While planning care for this client, what would be the priority expected outcome? a. Attains desired fluid balance b. Displays no signs or symptoms of infection c. Maintains a patent airway d. Demonstrates optimal cerebral tissue perfusion

c Maintenance of a patent airway is always a first priority. Loss of airway is a possible complication of increasing ICP, as well as aspiration from vomiting.

A patient is diagnosed with amyotrophic lateral sclerosis, also known as ALS or Lou Gehrig's disease. The nurse understands that the symptoms of the disease will begin in what way? a. Ascending paralysis b. Numbness and tingling in the lower extremities c. Weakness starting in the muscles supplied by the cranial nerves d. Jerky, uncontrolled movements in the extremities

c The chief symptoms are fatigue, progressive muscle weakness, cramps, fasciculations (twitching), and incoordination. In about 25% of patients, weakness starts in the muscles supplied by the cranial nerves, and difficulty in talking, swallowing, and ultimately breathing occurs.

A client is being admitted to the neurologic ICU following an acute head injury that has resulted in cerebral edema. When planning this client's care, the nurse would expect to administer what priority medication? a. Hydrochlorothiazide b. Furosemide c. Mannitol d. Spironlactone

c The osmotic diuretic mannitol is given to dehydrate the brain tissue and reduce cerebral edema. This drug acts by reducing the volume of brain and extracellular fluid. Spironlactone, furosemide, and hydrochlorothiazide are diuretics that are not typically used in the treatment of increased ICP resulting from cerebral edema.

The nurse is preparing discharge teaching for a patient who is being discharged after hospitalization for a hemorrhagic stroke. What should be included in the discharge teaching for this patient? a. Intermittent seizures can be expected. b. Take ibuprofen for complaints of a serious headache. c. Take antihypertensive medication as ordered. d. Drowsiness is normal for the first week after discharge.

c The patient and family are provided with information that will enable them to cooperate with the care and restrictions required during the acute phase of hemorrhagic stroke and to prepare them to return home. Patient and family teaching includes information about the causes of hemorrhagic stroke and its possible consequences. Symptoms of hydrocephalus include gradual onset of drowsiness and behavioral changes. Hypertension is the most serious risk factor, suggesting that appropriate antihypertensive treatment is essential for a patient being discharged. Seizure activity is not normal; complaints of a serious headache should be called to the health care provider before any medication is taken. Drowsiness is not normal.

A patient 3 days postoperative from a craniotomy informs the nurse, "I feel something trickling down the back of my throat and I taste something salty." What priority intervention does the nurse initiate? a. Give the patient some mouthwash to gargle with. b. Request an antihistamine for the postnasal drip. c. Ask the patient to cough to observe the sputum color and consistency. d. Notify the physician of a possible cerebrospinal fluid leak.

d Any sudden discharge of fluid from a cranial incision is reported at once, because a large leak requires surgical repair. Attention should be paid to the patient who complains of a salty taste or "postnasal drip," because this can be caused by cerebrospinal fluid trickling down the throat.

The nurse practitioner advises a patient who is at high risk for a stroke to be vigilant in his medication regimen, to maintain a healthy weight, and to adopt a reasonable exercise program. This advice is based on research data that shows the most important risk factor for stroke is: a. Obesity b. Dyslipidemia c. Smoking d. Hypertension

d Hypertension is the most modifiable risk factor for either ischemic or hemorrhagic stroke. Unfortunately, it remains under-recognized and undertreated in most communities.

A client has a poor prognosis after being involved in a motor vehicle accident resulting in a head injury. As the client's ICP increases and condition worsens, the nurse knows to assess for indications of approaching death. These indications include which of the following? a. Hemiplegia b. Dry mucous membranes c. Signs of internal bleeding d. Loss of brain stem reflexes

d Loss of brain stem reflexes, including pupillary, corneal, gag, and swallowing reflexes, is an ominous sign of approaching death. Dry mucous membranes, hemiplegia, and bleeding must be promptly addressed, but none of these is a common sign of impending death.

The nurse is participating in a health fair for stroke prevention. Which will the nurse say is a modifiable risk factor for ischemic stroke? a. Thyroid disease b. Social drinking c. Advanced age d. Smoking

d Modifiable risk factors for transient ischemic attack (TIA) and ischemic stroke include hypertension, type 1 diabetes, cardiac disease, smoking, and chronic alcoholism. Advanced age, gender, and race are nonmodifiable risk factors for stroke.

During the examination of an unconscious client, the nurse observes that the client's pupils are fixed and dilated. What is the most plausible clinical significance of the nurse's finding? a. It suggests onset of metabolic problems. b. It indicates paralysis on the right side of the body. c. It indicates paralysis of cranial nerve X (CN X). d. It indicates an injury at the midbrain level.

d Pupils that are fixed and dilated indicate injury at the midbrain level. This finding is not suggestive of unilateral paralysis, metabolic deficits, or damage to CN X.

The public health nurse is planning a health promotion campaign that reflects current epidemiologic trends. The nurse should know that hemorrhagic stroke currently accounts for what percentage of total strokes? a. 43% b. 33% c. 23% d. 13%

d Strokes can be divided into two major categories: ischemic (87%), in which vascular occlusion and significant hypoperfusion occur, and hemorrhagic (13%), in which there is extravasation of blood into the brain or subarachnoid space.

After a subarachnoid hemorrhage, the client's laboratory results indicate a serum sodium level of less than 126 mEq/L. What is the nurse's most appropriate action? a. Administer a bolus of normal saline as prescribed. b. Prepare the client for thrombolytic therapy as prescribed. c. Facilitate testing for hypothalamic dysfunction. d. Prepare to administer 3% NaCl by IV as prescribed.

d The client may be experiencing syndrome of inappropriate antidiuretic hormone (SIADH) or cerebral salt-wasting syndrome. The treatment most often is the use of IV hypertonic 3% saline. A normal saline bolus would exacerbate the problem and there is no indication for tests of hypothalamic function or thrombolytic therapy.

Which is the initial diagnostic test for a stroke? a. Carotid Doppler b. Electrocardiography c. Transcranial Doppler studies d. Noncontrast computed tomography

d The initial diagnostic test for a stroke is nonconstrast computed tomography performed emergently to determine whether the event is ischemic or hemorrhagic. Further diagnostics include a carotid Doppler, electrocardiogram, and transcranial Doppler.

A client is admitted to an acute care facility after an episode of status epilepticus. After the client is stabilized, which factor is most beneficial in determining the potential cause of the episode? a. The type of anticonvulsant prescribed to manage the epileptic condition b. Recent stress level c. Recent weight gain and loss d. Compliance with the prescribed medication regimen

d The most common cause of status epilepticus is sudden withdraw of anticonvulsant therapy. The type of medication prescribed, the client's stress level, and weight change don't contribute to this condition.

A client recovering from a stroke has severe shoulder pain from subluxation of the shoulder. To prevent further injury and pain, the nurse caring for this client is aware of what principle of care? a. The client should be fitted with a cast because use of a sling should be avoided due to adduction of the affected shoulder. b. Elevation of the arm and hand can lead to further complications associated with edema. c. Passively exercising the affected extremity is avoided in order to minimize pain. d. The client should be taught to interlace fingers, place palms together, and slowly bring scapulae forward to avoid excessive force to shoulder.

d To prevent shoulder pain, the nurse should never lift a client by the flaccid shoulder or pull on the affected arm or shoulder. The client is taught how to move and exercise the affected arm/shoulder through proper movement and positioning. The client is instructed to interlace the fingers, place the palms together, and push the clasped hands slowly forward to bring the scapulae forward; he or she then raises both hands above the head. This is repeated throughout the day. The use of a properly worn sling when the client is out of bed prevents the paralyzed upper extremity from dangling without support. Range-of-motion exercises are still vitally important in preventing a frozen shoulder and ultimately atrophy of subcutaneous tissues, which can cause more pain. Elevation of the arm and hand is also important in preventing dependent edema of the hand.


संबंधित स्टडी सेट्स

NT512a Epistles to Revelations Midterm (Blomberg)

View Set

CH 97 Postoperative Nursing care

View Set

Chapter 7 : Cellular Respiration (Glycolysis, Kreb's, and ETC)

View Set

History of Urban Planning Test #2

View Set

Pre-Exam Quiz #1: Chapters 1, 3, 29, and 4

View Set

Adult Health: Fluids, Electrolytes & Acid-Base Balance

View Set

Behavioral Challenges of Autism reliias

View Set

Kentucky Fact and Info Study Guide

View Set